Transcript
Page 1: MATEMÀTIQUES - · PDF fileMatemàtiques 2 · Batxillerat · Solucionari No és permesa la reproducció total o parcial d’aquest llibre, ni el seu tractament informàtic, ni la

BARCELONA - MADRID - BUENOS AIRES - CARACASGUATEMALA - LISBOA - MÈXIC - NOVA YORKPANAMÀ - SAN JUAN - BOGOTÀ - SÃO PAULOAUCKLAND - HAMBURG - LONDRES - MILÀ - MONT-REALNOVA DELHI - PARÍS - SAN FRANCISCO - SYDNEY - SINGAPURSAINT LOUIS - TÒQUIO - TORONTO

2S O L U C I O N A R I

Autors del llibre de l’alumneÀngela Jané

Jordi BesoraJosep M. Guiteras

Revisió tècnicaAntoni Giménez

MATEMÀTIQUES

Page 2: MATEMÀTIQUES - · PDF fileMatemàtiques 2 · Batxillerat · Solucionari No és permesa la reproducció total o parcial d’aquest llibre, ni el seu tractament informàtic, ni la

Matemàtiques 2 · Batxillerat · Solucionari

No és permesa la reproducció total o parcial d’aquest llibre, ni el seu tractament informàtic, ni la transmissió de cap forma o per qualsevol mitjà, ja sigui electrònic, mecànic, per fotocòpia, per registre o d’altres mitjans. Adreceu-vos a CEDRO (Centro Español de Derechos Reprográficos, www.cedro.org) si necessiteu fotocopiar o esca-nejar algun fragment d’aquesta obra.

Drets reservats © 2009, respecte a la primera edició en català per:

McGraw-Hill/Interamericana de España, S.A.U. Edificio Valrealty, 1.ª planta Basauri, 17 28023 Aravaca (Madrid)

ISBN: 978-84-481-7026-4Depósito legal:

Editora de projecte: Alícia AlmonacidTècnic editorial: Conrad AgustíDisseny de coberta: Quin Team!Disseny d’interiors: McGraw-HillIl·lustracions: Jordi SotoComposició: Servei Gràfic NJR, S.L.

IMPRÈS A - PRINTED IN

Page 3: MATEMÀTIQUES - · PDF fileMatemàtiques 2 · Batxillerat · Solucionari No és permesa la reproducció total o parcial d’aquest llibre, ni el seu tractament informàtic, ni la

3ÍNDEX

Guies didàctiques interactives McGraw-Hill ........................................................ 4

Unitat 0. Comencem

Activitats finals .................................................... 9

Bloc 1. Funcions

Unitat 1. Derivades

Activitats ............................................................ 14

Activitats finals .................................................. 27

Avaluació .......................................................... 32

Unitat 2. Funcions contínues i derivables

Activitats .......................................................... 33

Activitats finals .................................................. 36

Avaluació .......................................................... 40

Unitat 3. Aplicacions de la derivada

Activitats .......................................................... 41

Activitats finals .................................................. 50

Avaluació .......................................................... 61

Unitat 4. Primitives

Activitats .......................................................... 63

Activitats finals .................................................. 74

Avaluació .......................................................... 82

Unitat 5. La integral

Activitats .......................................................... 83

Activitats finals .................................................. 87

Avaluació .......................................................... 92

Bloc 2. Matrius i sistemes

Unitat 6. Vectors a l’espai

Activitats .......................................................... 94

Activitats finals .................................................. 101

Avaluació .......................................................... 104

Unitat 7. Matrius i determinants

Activitats .......................................................... 105

Activitats finals .................................................. 113

Avaluació .......................................................... 114

Unitat 8. Sistemes d’equacions

Activitats .......................................................... 115

Activitats finals .................................................. 121

Avaluació .......................................................... 125

Bloc 3. Geometria

Unitat 9. Equacions de rectes i plans

Activitats .......................................................... 126

Activitats finals .................................................. 129

Avaluació .......................................................... 132

Unitat 10. Posició relativa de rectes i plans

Activitats .......................................................... 133

Activitats finals .................................................. 138

Avaluació .......................................................... 142

Unitat 11. Distàncies i angles

Activitats .......................................................... 144

Activitats finals .................................................. 153

Avaluació .......................................................... 160

Page 4: MATEMÀTIQUES - · PDF fileMatemàtiques 2 · Batxillerat · Solucionari No és permesa la reproducció total o parcial d’aquest llibre, ni el seu tractament informàtic, ni la

4 GUIES DIDÀCTIQUES INTERACTIVES MCGRAW-HILL

j Guies didàctiques interactives McGraw-Hill

A fi de facilitar-vos la tasca docent, hem complementat l’oferta del nostre llibre de Matemàtiques 2 amb una guia didàctica interactiva, que estem convençuts que us serà de gran ajut.

A continuació us en presentem els trets principals, tot i que, sens dubte, serà a mesura que l’aneu fent servir que en descobrireu totes les potencialitats. A més, incorpora una adreça de correu electrònic, on ens podeu fer arribar les vostres observacions i suggeriments.

Com veureu, és fàcil de fer anar, molt visual i intuïtiva, i no requereix cap mena d’instal·lació prèvia.

McGraw-Hill, avui, com sempre, qualitat al servei de l’educador.

A la pantalla principal apareix la barra de menú amb les opcions de navegació i de visualització de les guies digitals.

El vídeo de presentació explica com s’ha de treballar amb les guies didàctiques interactives de McGraw-Hill.

Prement en els ítems de l’índex de con-tinguts podeu accedir a material genèric de la matèria amb més informació i ac-tivitats extres.

Menú amb les accions disponibles per als professors

Continguts addicionals

Page 5: MATEMÀTIQUES - · PDF fileMatemàtiques 2 · Batxillerat · Solucionari No és permesa la reproducció total o parcial d’aquest llibre, ni el seu tractament informàtic, ni la

5GUIES DIDÀCTIQUES INTERACTIVES MCGRAW-HILL

El menú us mostra totes les opcions genèriques per navegar dins de les guies i per visualitzar les pàgines i els continguts.

Pàgina anterior: prement aquest botó podeu navegar fi ns la pàgina anterior. Pàgina següent: amb aquesta opció podeu avançar fi ns a la pàgina següent.Inici de la guia: prement aquesta opció podeu anar al començament de la guia.Fi de la guia: podeu navegar fi ns a la darrera pàgina de la guia.Opció de lectura recoman da: permet ampliar el text i les imatges de la pàgina que s’està llegint.Opció cerca ràpida: aquesta opció us mostra en versió reduïda totes les pàgines de la guia.Cerca: us serveix per cercar paraules dins del text de la publicació.Ajuda: en qualsevol moment podeu visualitzar l’ajuda per fer servir adequadament la guia digital.Índex de continguts: l’índex de continguts està sempre accessible per navegar pels continguts addicionals més ràpida-ment.Sumari: índex de les unitats i dels continguts del llibre de l’alumne.

recomanadaOpció de cerca

Cerca

Tancar la

a

Page 6: MATEMÀTIQUES - · PDF fileMatemàtiques 2 · Batxillerat · Solucionari No és permesa la reproducció total o parcial d’aquest llibre, ni el seu tractament informàtic, ni la

6 GUIES DIDÀCTIQUES INTERACTIVES MCGRAW-HILL

del llibre.

j Zones senyalitzades

Page 7: MATEMÀTIQUES - · PDF fileMatemàtiques 2 · Batxillerat · Solucionari No és permesa la reproducció total o parcial d’aquest llibre, ni el seu tractament informàtic, ni la

7GUIES DIDÀCTIQUES INTERACTIVES MCGRAW-HILL

Visualització de doble pàgina:

Requeriments tècnics:L’ús d’aquestes guies interactives no requereix cap instal·lació especial, ja que funcionen amb el navegador d’Internet.

Tot i que no és necessari estar connectat a la Xarxa per fer-les anar, sí que hi ha continguts, com l’accés a pàgines web, que només es po-dran aprofitar al 100% si s’està on-line.

La major part d’equips ja incorporen el Flash Player, però si no fos el vostre cas, us el podeu descarregar gratuïtament des del web d’Adobe.

Opció de lectura recomanada:

Opció cerca ràpida:

j Opcions de visualització

Page 8: MATEMÀTIQUES - · PDF fileMatemàtiques 2 · Batxillerat · Solucionari No és permesa la reproducció total o parcial d’aquest llibre, ni el seu tractament informàtic, ni la
Page 9: MATEMÀTIQUES - · PDF fileMatemàtiques 2 · Batxillerat · Solucionari No és permesa la reproducció total o parcial d’aquest llibre, ni el seu tractament informàtic, ni la

9MATEMÀTIQUES 2 0

j Unitat 0. Comencem

Activitats finals 1. Calcula:

a)

b)

c)

d)

2. Expressa de la manera més senzilla possible el resultat de les operacions següents:

a)

7

b)

c) ·

·

d)

3. Expressa en forma d’una sola arrel:

a)

b)

c)

d)

e)

f)

g)

h)

i)

4. Si A(x) 6x4 2x2 4x 6 i B(x) x3 2x 1, calcula:

a) 2 . A(x)

Multipliquem els coeficients per 2:

b) 3x . B(x)

Multipliquem els coeficients per 3x:

c) A(x) : B(x)

Quocient: c(x) 6x

Residu: r(x) 14x2 10x 6

Page 10: MATEMÀTIQUES - · PDF fileMatemàtiques 2 · Batxillerat · Solucionari No és permesa la reproducció total o parcial d’aquest llibre, ni el seu tractament informàtic, ni la

10 SOLUCIONARI DEL LLIBRE DE L’ALUMNE0

d) B(x) : (x 1)

Apliquem la regla de Ruffini. Com que B(x) no té terme de grau dos, en el seu lloc hi posem un zero. El primer nombre de la segona fila és 1, perquè dividim entre x 1 canviant de signe el terme independent del binomi. El quocient queda determinat pels tres primers termes de la tercera fila: 1, 1, 1 x2 x 1. El residu és 2.

5. Efectua aquestes divisions. Aplica la regla de Ruffini quan que sigui possible.

a) (x4 2x2 1) : (x 2)

Per Ruffini:

Quocient: x3 2x2 2x 4

Residu: 9

b) (x6 x3 x 1) : (x 1)

Per Ruffini:

Quocient: x5 x4 x3 2x2 2x 1

Residu: 2

6. Factoritza els polinomis següents:

a) A(x) 6x3 20x2 6x

A(x) 6x3 20x2 6x x (6x2 20x 6) 6x (x 3)

b) B(x) x4 3x3 3x2 11 x 6

B(x) x4 3x3 3x2 11x 6 (x 1)2(x 2)(x 3)

7. Determina el m.c.d. i el m.c.m. dels polinomis:

A(x) 2x5 6x4 8x2

A(x) 2x2 (x 1)(x 2)2

B(x) x3 x

B(x) x (x 1)(x 1)

C(x) x4 x3 x2 x

C(x) x (x 1)2(x 1)

m.c.d. (x 1)x

m.c.m. 2x2 (x 1)2(x 1)(x 2)2

8. Calcula:

Cal tenir en compte que 1 x (x 1). m.c.m. x2 1

xx

xx

xx

x xx

x

11

11

11

1 11

2

2

2

( )( ) ( 1 11

11

112

2

2

2

2

)( )xx

xx

xx

9. Donades les fraccions algèbriques següents:

A(x) i B(x)

calcula: A(x) . B(x), A(x) : B(x) i B(x) : A(x).

A(x) . B(x)

A(x) : B(x)

B(x) : A(x)

10. Resol els sistemes d’equacions lineals següents pel mètode que s’indica:

a) per reducció.

Multipliquem la primera equació per 2. D’aquesta manera, la x tindrà el mateix coeficient en les dues equacions:

Restem les dues equacions per reduir-ne el nombre d’incògni-tes:

Page 11: MATEMÀTIQUES - · PDF fileMatemàtiques 2 · Batxillerat · Solucionari No és permesa la reproducció total o parcial d’aquest llibre, ni el seu tractament informàtic, ni la

11MATEMÀTIQUES 2 0

Substituïm aquest valor en qualsevol de les dues equacions (per exemple, en la primera) per trobar el valor de l’altra in-cògnita.

La solució del sistema és: (x, y) (4, 5)

b) per substitució.

Aïllem una incògnita d’una de les equacions, per exemple la y de la segona equació:

x y 1 y 1 x

Substituïm en la primera equació la y per 1 x:

2x 3 ( 1 x) 1

Resolem la equació que apareix, que té una única incògnita:

2x 3 ( 1 x) 1 2x 3 3x 1

2x 3x 1 3 x 4 x 4

Substituïm aquest valor en la igualtat en la qual hem aïllat la incògnita y:

y 1 ( 4) y 1 4 y 3

La solució del sistema és: (x, y) ( 4, 3).

c) per igualació.

Aïllem una mateixa incògnita de les dues equacions, per exem-ple la x:

Igualem els membres de la dreta de les equacions:

Resolem l’equació que apareix, que té una única incògnita:

Substituïm aquest valor en qualsevol equació en la qual la x estigui aïllada, per exemple en la primera:

La solució del sistema és: (x, y) (1, 1)

11. Resol els sistemes d’equacions següents:

a)

Substituïm en la segona, i resolem l’equació:

Substituïm en la 1a equació: x 10 16 26.

Solució: (x, y) (26, 10).

b)

Aïllem la y de la 2a, y 7 x, substituïm en la 1a i resolem:

a 1, b 7 i c 10

Substituïm en l’equació aïllada:

Solucions: (x, y) (2, 5)

(x, y) (5, 2)

c)

Per reducció. Restem les dues equacions:

Resolem l’equació:

Page 12: MATEMÀTIQUES - · PDF fileMatemàtiques 2 · Batxillerat · Solucionari No és permesa la reproducció total o parcial d’aquest llibre, ni el seu tractament informàtic, ni la

12 SOLUCIONARI DEL LLIBRE DE L’ALUMNE0

Arribem a una identitat, per la qual cosa les dues equacions són equivalents (gairebé són la mateixa). El sistema és compatible indeterminat: té infinites solucions. Si aïllem una de les incòg-nites d’una equació obtindrem una fórmula per trobar totes les solucions. Per exemple, la x de la primera equació:

x 5 y Solució: (x, y) (5 y, y)

Per a cada valor de y tindrem una solució del sistema. Exemples:

y 1 (4, 1) y 0 (5, 0) y 3 (8, 3)

12. a) Per a quins valors de m l’equació x2 mx 4 0 té una solució?

El discriminant de la equació ( b2 4ac) ha de ser igual a zero:

a 1, b m i c 4

b) Per a quins valors de m el sistema

té una solució única?

El sistema ha de ser compatible determinat, és a dir, :

,

,

1 m m 1

Per a tots els valors diferents de 1.

13. Sense resoldre’ls, classifica els sistemes següents:

a)

Sistema compatible determinat té una solució.

b) 2 3

4 2 6x y

x y

24

12

36

0 5 0 5 0 5, , ,

Sistema compatible indeterminat infinites solucions.

c) 2 3

4 2 6x y

x y

Sistema incompatible no té solució.

14. Determina el domini de cadascuna de les funcions se-güents:

a) g(x)

R

b) k(x)

R

15. Siguin f(x) i g(x)

a) Troba les funcions: ( )( )f g x , ( )( )f g x , ( )fg x ,

( )( )f f x , ( )( )g g x , f x1( ).

(f g)(x) f(x) g(x)

(f . g)(x) f(x) . g(x)

(x)

Page 13: MATEMÀTIQUES - · PDF fileMatemàtiques 2 · Batxillerat · Solucionari No és permesa la reproducció total o parcial d’aquest llibre, ni el seu tractament informàtic, ni la

13MATEMÀTIQUES 2 0

(f ° f)(x) f(f(x))

(g ° g)(x) g(g(x))

y xy y 2x 1

2x xy y 1 x (2 y) y 1

x f 1(x)

b) Troba el domini d’aquestes funcions.

R

R

R

R

c) Comprova que f 1 (x) és la funció inversa de f (x).

(f 1 o f) (x) f 1 ( f(x)) f 1

(f o f 1) (x)

Page 14: MATEMÀTIQUES - · PDF fileMatemàtiques 2 · Batxillerat · Solucionari No és permesa la reproducció total o parcial d’aquest llibre, ni el seu tractament informàtic, ni la

14 SOLUCIONARI DEL LLIBRE DE L’ALUMNE1

j Bloc 1. Funcions

j Unitat 1. Derivades

Activitats

1. La funció f(x) x3 2 sempre és creixent. Calcula’n la variació mitjana a cadascun dels intervals següents: [ 3, 1], [0, 2] i [5, 7].

En quin dels tres intervals té un creixement més ràpid?

Interval [ 3, 1]: 13

Interval [0, 2]: 4

Interval [5, 7]: 109

La funció f(x) x3 2 té el creixement més ràpid en l’interval[5, 7].

2. Considera la funció f(x) 3x 1. Demostra que la variació mitjana sempre és la mateixa, independentment de l’inter-val [x1, x2] considerat.

3

En qualsevol interval [x1, x2] la variació mitjana de la funció és 3.

3. Quant val la variació mitjana de la funció f(x) 5 en qual-sevol interval [x1, x2]?

Val zero, ja que es tracta d’una funció constant.

4. Calcula la variació mitjana de la funció f(x) x2 4x a l’in-terval [2,9, 3,1]. Creix o decreix aquesta funció al voltant de x 3?

2

Fes-ne la representació gràfica i comprova després la teva resposta.

Podem esperar que la funció f(x) x2 4x decreixi al voltantde x 3. Ho comprovem a la gràfica de la funció.

5. La funció ƒ(x) x2 6x és decreixent al voltant de x 4.

Quantifica aquest decreixement calculant .

Interpreta’n el resultat obtingut.

(2 x) 2

Per a valors de x pròxims a 4, la funció f(x) disminueix de l’ordre de dues vegades el que augmenta x.

6. Fes el mateix estudi, de l’activitat 5, per a x 3.

Fixa’t en la gràfica de la funció i interpreta el resultat que has obtingut.

(3 x) 0

7. Representa gràficament la funció f(x) 2x 3. Calcula f ’( 2), f ’(0) i f ’(3). Interpreta’n els resultats.

3

f ’( 2)

2 2

Page 15: MATEMÀTIQUES - · PDF fileMatemàtiques 2 · Batxillerat · Solucionari No és permesa la reproducció total o parcial d’aquest llibre, ni el seu tractament informàtic, ni la

15MATEMÀTIQUES 2 1

També es verifica: f ’(0) f ’(3) 2.

La funció f (x) 2x 3 decreix sempre de la mateixa manera,és a dir, presenta un decreixement uniforme. En general,f ’(x0) 2, x0 R.

8. Donada la funció f(x) ax b, demostra que f ’(x0) a, independentment del valor x0 considerat.

ƒ’(x0)

a a

9. Calcula, si és possible:

a) f ’(8) si f(x)

f’(8)

b) f ’ si f(x) 4 x2

ƒ’

( 1 h) 1

c) f’(0) si f(x)

No existeix f’(0), ja que x 0 no pertany al domini de la fun-ció.

f xx

( )1

no existeix f(0).

d) ƒ’( 2) si ƒ(x) x 2

ƒ’( 2) ∞

La funció f(x) no és derivable en x 2.

10. Representa gràficament la funció f(x) x2 2x 4 i in-dica’n, a partir de la gràfica, els intervals de creixement i decreixement. Comprova que f’(1) 0.

Decreixent: ( ∞, 1)Creixent: (1, ∞)

(1)1 (1)

1 2 1 4 3

1 2 2 2 4 3

0

f hf h f

hh h

hh h h

hh h

0

0

2

0

2

0

2

0

h

h

h

h h

lim

lim

lim

lim lim

"

"

"

" "

l]] ]

gg g

11. Sense fer-ne la representació gràfica, indica si la funció ƒ(x) (2 x)2 és creixent o decreixent en x 6. Fes el ma-teix estudi en x 1.

ƒ’(6)

(h 8) 8

Page 16: MATEMÀTIQUES - · PDF fileMatemàtiques 2 · Batxillerat · Solucionari No és permesa la reproducció total o parcial d’aquest llibre, ni el seu tractament informàtic, ni la

16 SOLUCIONARI DEL LLIBRE DE L’ALUMNE1

ƒ’(6) 0 creixent en x 6.

ƒ’( 1)

(h 6) 6

ƒ’( 1) 0 decreixent en x 1.

12. Com ha de ser una funció perquè la derivada sigui nul·la en tots i cadascun dels punts del seu domini? Per què?

La funció ha de ser constant, f(x) K, K R. És així perquè siuna funció és constant, la seva variació és zero per a qualsevolvalor de x Dƒ R.

13. Calcula, si és possible:

a) f’( 4) si f(x)

No és possible, ja que no existeix f( 4).

f( 4) R

b) f’(1) si f(x) 2x

ƒ’(1)

2

c) f’(0) si f(x) 2x2 1

ƒ’(0)

2h 2 . 0 0

d) f’( 2) si f(x) 10x 3

ƒ’( 2)

10 10

14. Troba l’equació de la recta tangent a la gràfica de la funció f(x) 3x2 10x 3 en x 2.

x 2 f(2) 3 · 22 10 · 2 3 5 (2, 5)

f’(x) 6x 10

mtg f’(2) 6 · 2 10 12 10 2

y 5 2 (x 2) y 5 2x 4 y 2x 9

15. Considera la funció f(x) x3 2. En quins punts de la gràfi-ca d’aquesta funció la recta tangent és paral·lela a la recta y 3x 5?

Es tracta de buscar els valors de x per als quals es compleix que f’(x) 3.

f(x) x3 2 f’(x) 3x2 3x2 3 x ± 1

x 1 f(1) 13 2 3

x 1 f( 1) ( 1)3 2 1

Els punts són (1, 3) i ( 1, 1).

16. Dibuixa la recta tangent a la corba representada a la gràfica en els punts d’abscisses x 3, x 0 i x 2.

Page 17: MATEMÀTIQUES - · PDF fileMatemàtiques 2 · Batxillerat · Solucionari No és permesa la reproducció total o parcial d’aquest llibre, ni el seu tractament informàtic, ni la

17MATEMÀTIQUES 2 1

a) Quin és el signe del pendent de cadascuna d’aquestes tangents?

En x 3, pendent positiu; en x 0, pendent negatiu; en x 2, pendent positiu.

b) Quin signe tenen f’( 3), f’(0) i f’(2)?

f’( 3) 0f’(0) 0f’(2) 0

17. A partir de la gràfica, fes una estimació dels valors de f’(2), g’( 1) i h’(0).

f’(2) 0,7

g( 1)

g’( 1) 0

h’(0) 0,4

18. Considera la funció f(x) x2 3x 5. Digues en quin punt de la seva gràfica la recta tangent forma un angle de 45º amb el sentit positiu de l’eix X. Aquesta funció, és creixent o decreixent en aquest punt? Per què?

Com que tg 45º 1, es tracta de determinar el valor o valors de x per als quals es verifica que f’(x) 1.

f(x) x2 3x 5 f’(x) 2x 3 2x 3 1 x 2

f(2) 22 3 · 2 5 3 (2, 3)

En el punt P(2, 3) la funció és creixent, ja que f’(2) 1 0.

19. Esbrina quins són els punts de la gràfica de la funció f(x) x3 6x2 4 que tenen tangent paral·lela a l’eix d’abs-cisses.

f(x) x3 6x2 4 f’(x) 3x2 12x

Si la recta tangent és paral·lela a l’eix 0X, mtg tg 0º 0.

Per tant, es tracta de trobar quins són els valors de x que com-pleixen l’equació: f’(x) 0.

3x2 12x 0 x (3x 12) 0 x 0 i x 4

x 0 f(0) 4

x 4 f(4) 28

Els punts són P(0, 4) i Q(4, 28)

20. Indica raonadament per què la funció f(x) 1x

és decrei-xent en tots els punts del seu domini.

Perquè f(x) 1x2

i, per tant, f’(x) 0 per a qualsevol x R, x 0.

21. La gràfica de la funció f(x) x2 bx c presenta un mínim en el punt (3, 1).

a) Calcula b i c.

Es compleix:

f(3) 1 1 9 3b c 3b c 10

f’(3) 0 amb f’(x) 2x b 0 6 b b 6

3 · ( 6) c 10 c 8

La funció és f(x) x2 6x 8.

b) Representa-la gràficament i verifica el resultat de la teva resposta.

22. Les gràfiques de les funcions polinòmiques de segon grau f(x) ax2 bx c sempre tenen un màxim o un mí-nim. Demostra que es troba localitzat en el punt d’abscissa

x0 b2a

.

Cal que f’(x) 0, ja que en un màxim o en un mínim la gràfica de la funció presenta sempre tangent horitzontal.

f’(x) 2ax b 0 2ax b x ba2

Page 18: MATEMÀTIQUES - · PDF fileMatemàtiques 2 · Batxillerat · Solucionari No és permesa la reproducció total o parcial d’aquest llibre, ni el seu tractament informàtic, ni la

18 SOLUCIONARI DEL LLIBRE DE L’ALUMNE1

23. Digues en quins punts no són derivables cadascuna de les funcions següents i indica’n en cada cas el motiu:

a) f(x) 2x2

x 0, perquè no pertany al Df .

b) g(x) 1

x2 9

x 3 i x 3, perquè no pertanyen al Dg.

c) h(x) x

4 3x

x 43

, perquè no pertany al Dh.

d) i(x) x5

x 0, perquè la recta tangent és perpendicular a l’eix 0X.

24. Representa gràficament la funció:

f(x)

4 x2 si x 02x 4 si x 0

És contínua en x 0? I derivable?

És contínua en x 0, ja que lim f(x) lim f(x) f(0) 4x 0 x 0

f’(x) –2x si x 0 2 si x 0

Per tant, f’(0 ) 2 i f’(0 ) 0. Com que f’(0 ) f’(0 ), la funció no és derivable en x 0.

25. Donada la funció:

f(x) ax b si x 2

2x2 3 si x 2 Troba a i b perquè sigui derivable en x 2.

limx 2

f(x) 2a b

limx 2

f(x) 11

f(2) 11

La funció ha de ser contínua en x 2 2a b 11

f’(x) a si x 24x si x 2

En conseqüència: f’(2 ) f’(2 ) a 8

2a b 11a 8

16 b 11 b –5

26. La funció:

ƒ(x)

és derivable en x 2? Per què?

ƒ(2) 3

f(x) (x 2) 4

No és derivable en x 2 ja que f(x) f(2), i, per tant f(x) no és contínua en x 2.

27. La funció f(x) |x2 6x 8| és, en realitat, una funció de-finida a trossos:

f(x)

x2 6x 8 si x 2 o x 4x2 6x 8 si 2 x 4

La gràfica de la funció es pot obtenir fàcilment a partir de la gràfica de la funció g(x) x2 6x 8. Dibuixa les gràfiques de les dues funcions. Estudia la continuïtat i la derivabilitat de la funció f(x) en x 2 i en x 4.

2 4 6 8 10

2

2

4

6

8

10

y = x2 − 6x + 8

Page 19: MATEMÀTIQUES - · PDF fileMatemàtiques 2 · Batxillerat · Solucionari No és permesa la reproducció total o parcial d’aquest llibre, ni el seu tractament informàtic, ni la

19MATEMÀTIQUES 2 1

f(x) és contínua en x 2 i en x 4, ja que es compleix:

lim f(x) lim f(x) f(2) 0x 2 x 2

lim f(x) lim f(x) f(4) 0x 4 x 4

En canvi, la funció no és derivable ni en x 2 ni en x 4.

f’(x) 2x 6 si x 2 o x 4 2x 6 si 2 x 4

f’(2 ) 2; f’(2 ) 2 f’(2 ) f’(2 )

f’(4 ) 2; f’(4 ) 2 f’(x)(4 ) f’(4 )

28. Sabem que la funció f(x) 3

4 bx no és derivable en x 2.

Calcula b.

L’expressió 4 bx s’anul·la per a x 2:

4 2b 0 b 2

29. Defineix a trossos la funció f(x) |x 2|. Representa-la grà-ficament i indica raonadament en quin punt no és derivable.

f(x) x 2 si x 2

x 2 si x 2

No és derivable en x 2, ja que f’( 2 ) 1 i, en canvi, f’( 2 ) 1. Per tant, f’( 2 ) f’( 2 )

30. Calcula la funció derivada de cadascuna de les funcions se-güents:

a) f(x) x 7

ƒ’(x)

1 1

b) f(x) 1 2x2

ƒ’(x)

2(2x h) 4x

c) f(x) , x 0

f’(x)

d) f(x) , x 0

f’(x)

e) f(x) , x 0

f’(x)

f) f(x) 3

f’(x) 0

Page 20: MATEMÀTIQUES - · PDF fileMatemàtiques 2 · Batxillerat · Solucionari No és permesa la reproducció total o parcial d’aquest llibre, ni el seu tractament informàtic, ni la

20 SOLUCIONARI DEL LLIBRE DE L’ALUMNE1

g) f(x) 3x2 2x 1

f’(x)

(3h 6x 2) 6x 2

h) f(x)

f ’(x) 0

31. Sense fer-ne la representació gràfica, determina els inter-vals de creixement i decreixement de la funció f(x) x2 6x 8. Quant val f ’(3)?

32. Donada la funció f(x) 6 x2, calcula f’( 2) i f’(4). Indica si la funció és creixent o decreixent en x 2 i en x 4.

La funció és creixent en x 2.

La funció és creixent en x 4.

33. Donada la funció f(x) x4, calcula f’(x) de dues maneres diferents:

a) Aplicant la definició de funció derivada.

b) A partir de la segona regla que acabem de veure.

34. Calcula la funció derivada de cadascuna de les funcions se-güents:

Page 21: MATEMÀTIQUES - · PDF fileMatemàtiques 2 · Batxillerat · Solucionari No és permesa la reproducció total o parcial d’aquest llibre, ni el seu tractament informàtic, ni la

21MATEMÀTIQUES 2 1

35. Considera la funció ƒ(x) . Calcula ƒ’(4); ƒ’(16). Interpreta’n els resultats obtinguts.

f x x( )12 f x x

x( )

12

1

2

12

f

f

( )

( )

414

1618

La funció és decreixent en x 4 i en x 16, ja que ƒ’(4) 0 i ƒ’(16) 0. Com que |ƒ’(4)| |ƒ’(16)|, la funció decreix amb més rapidesa prop de x 4 que prop de x 16.

De fet, la funció és decreixent en tot el seu domini, excepte en x 0, on no és derivable.

36. Donada la funció f(x) x3, calcula f ’( 1) i f ’(1). Indica si la funció és creixent o decreixent en aquests dos punts, i en cas que hi presenti el mateix tipus de variació, digues on és més ràpida aquesta variació.

la funció és creixent en x 1 i x 1 i en ambdós punts creix amb la mateixa rapidesa.

37. Pot decréixer en algun punt la funció de l’activitat anterior? Per què?

No, perquè ƒ’(x) 3x2 ≥ 0 per a qualsevol x R.

38. Indica raonadament per què la funció ƒ(x) és decrei-xent en tots els punts del seu domini.

ƒ(x) x 1 ƒ’(x) 1 x 2

Si x R {0}, x2 0, i, per tant, f’(x) 0. Aleshores, f’(x) 0 per a qualsevol valor de x real i diferent de zero. Cal tenir en compte que Df R {0}.

39. Troba la funció derivada de la funció f x x( ) 3 i comprova que aquesta funció no és derivable en x 0.

f x x x f x xx

( ) ( )313

23

23

13

1

3

La funció f(x) no és derivable en x 0, ja que aquest valor anul.la el denominador de la funció f’(x).

40. Calcula la funció derivada de cadascuna de les funcions se-güents:

41. Per a quins valors de x s’anul·la la derivada de la funció f(x) x3 5x2 3x 4?

42. Demostra que la derivada de la funció polinòmica de segon grau f(x) ax2 bx c s’anul·la per al valor de x correspo-nent al vèrtex de la paràbola que en resulta de representar-la gràficament.

43. Quina és l’equació de la recta tangent a la gràfica de la funció ƒ(x) x2 6x en els punts en què talla l’eix d’abscisses?

ƒ(x) 0 x2 6x 0 x1 0, x2 6

Els punts són (0, 0) i (6, 0).

Page 22: MATEMÀTIQUES - · PDF fileMatemàtiques 2 · Batxillerat · Solucionari No és permesa la reproducció total o parcial d’aquest llibre, ni el seu tractament informàtic, ni la

22 SOLUCIONARI DEL LLIBRE DE L’ALUMNE1

ƒ’(x) 2x 6

Punt (0, 0):

mtg ƒ’(0) 6 y 6x

Punt (6, 0):

mtg ƒ’(6) 6 y 6(x 6) y 6x 36

44. Troba l’equació d’una funció f(x) que tingui per derivada la funció f’(x) representada en la gràfica. Pots trobar-ne més d’una? Per què?

Compleixen la condició que s’estableix a l’enunciat totes les fun-cions del tipus f(x) x k, amb k R.

45. Troba la funció derivada de cadascuna de les funcions següents:

a) f(x) 3 sin x 5

f’(x) 3 cos x

b) f(x) 4 cos x 2 sin x 1

f’(x) 4 sin x 2 cos x

c) f(x) ln x7

2x

f’(x) 17x

2x2

d) f(x) log3 x 3x ln 9

f’(x) 1

ln 3 · x 3

46. Determina els punts d’abscisses compreses entre 0 i 2 en els quals la recta tangent a la gràfica de la funció f(x) sin x és paral·lela a l’eix OX. Escriu les equacions d’aquestes rectes tangents.

mtg 0 f’(x) 0 cos x 0 x 2

i x 32

x 2

f 2

sin 2

1 2

, 1

x 32

f 32

sin 32

1 32

, 1

L’equació de la recta tangent a la gràfica de f(x) sin x en el

punt 2

, 1 és y 1; en el punt 32

, 1 , la recta tangent té

per equació y 1.

47. Determina l’equació de la recta perpendicular a la recta tan-gent a la gràfica de la funció f(x) 2 cos x en

6. Aquesta

recta s’anomena recta normal a la gràfica de la funció en aquest punt.

f(x) 2 cos x f’(x) 2 sin x

x 6

f 6

2 cos 6

36

, 3

mtg f’ 6

2 sin 6

2 · 12

1 mnormal 1

Equació de la normal:

y 3 1 x 6

x y 3 6

0

48. Hi ha algun punt de la gràfica de la funció f(x) log2 x que tingui recta tangent paral·lela a la bisectriu del primer qua-drant i del tercer? Si la resposta és afirmativa, troba l’equa-ció d’aquesta recta tangent.

f(x) log2 x f’(x) 1 · 1 ln2 x

Bisectriu primer i tercer quadrants: y x m 1.

f’(x) 1 1 · 1 1 x 1 ln2 x ln2

x 1 f 1 log2 1

ln2 ln2 ln2

El punt és 1 , log2 1 ~ (1,44, 0,53)

ln2 ln2

Equació de la recta tangent:

y 0,53 x 1,44 x y 0,91 0

49. Justifica per què la gràfica de la funció f(x) ln x no pot tenir ni màxims ni mínims.

Perqué la funció derivada, f’(x) 1x

no s’anul·la per a cap valor real de x:

1x

0,

A

x R

50. Calcula la derivada de les funcions següents:

a) ( ) 1f x x23

Page 23: MATEMÀTIQUES - · PDF fileMatemàtiques 2 · Batxillerat · Solucionari No és permesa la reproducció total o parcial d’aquest llibre, ni el seu tractament informàtic, ni la

23MATEMÀTIQUES 2 1

b) f(x) sin (3x 5)

f’(x) 3 cos (3x 5)

c) f(x) ln (cos x)

f’(x) sin x tg x cos x

d) f(x) (1 x2)3

f’(x) 3 (1 x2)2 · ( 2x) 6x (1 x2)2

e) f(x) cos3 x 2

f’(x) 3 cos2 x sin x

f) f(x) sin (ln x)

f’(x) cos(ln x)

x

g) f(x) sin2 x sin x2

f’(x) 2 sin x cos x 2x cos x2 2 (sin x cos x x cos x2)

h) f(x) log

i) f(x) 1

(x2 4)2

f’(x) –2(x2 4) · 2x –4x (x2 4)4 (x2 4)3

j) f(x) sin [cos (ln x)]

f’(x) cos(cos(ln x))sin(ln x)

x

k) f(x) ln [sin (1 x)]

f’(x) cos(1 x) cotg(1 x) sin(1 x)

l) f(x) cos2 (1 3x)

f’(x) 6 cos (1 3x) sin (1 3x)

51. Calcula la derivada de la funció f(x) sin2 x cos2 x. Inter-preta’n el resultat obtingut.

f’(x) 0, perquè f(x) sin2 x cos2 x 1

52. Troba l’equació de la recta tangent al gràfic de la funció

f(x) 2 sin2 x en x 4

.

x f 2 sin2 2 · 1 1 , 14 4 4 2 4

f ’(x) 4 sin x cos x

mtg f’ 4 sin cos 2 4 4 4

Equació de la recta tangent:

y 1 2 x 2 x y 1 04 2

53. Indica per a quins valors de x és creixent la funció f(x) ln (x2 1). Per què?

Té la gràfica d’aquesta funció algun punt en el qual la recta tangent tingui pendent nul? Si la resposta és afirmativa, de quin punt es tracta?

f’(x) 1 · 2x 2x x2 1 x2 1

La funció és creixent per a x R , ja que si x 0 es verifica f’(x) 0.

mtg 0 f’(x) 0 2x 0 x2 1

2x 0 x 0 f (0) ln1 0

La recta tangent a la gràfica de la funció té pendent nul·la en el punt (0, 0).

54. Calcula la funció derivada de les funcions següents:

a) f(x) x3 · cos x

f’(x) 3x2 cos x x3 ( sin x) x2 (3 cos x x sin x)

b) f(x) x · ln x ln2 x 1

f’(x) ln x x ∙ 1 2 ln x ∙ 1 2 ln x ln x 1 x x x

c) f(x) 7 cotg x 3

f(x) 7 cos x 3 f’(x) 7 –sin2 x cos2 x –7 sin x sin2 x sin2 x

d) f(x) 1 x2 3 x 2

f’(x) 2x 3 (x2 3x 2)2

e) f(x) sin x cos x 1 sin x

f xx x x x x

( )(cos – sin )( – sin ) – (sin cos ) (–cos1 xx

x

x xx

)( sin )

cos – sin( sin )

1

11

2

2

Page 24: MATEMÀTIQUES - · PDF fileMatemàtiques 2 · Batxillerat · Solucionari No és permesa la reproducció total o parcial d’aquest llibre, ni el seu tractament informàtic, ni la

24 SOLUCIONARI DEL LLIBRE DE L’ALUMNE1

f) f(x) ∙ (1 x)

g) f(x) 1 2 cos 3x 5

f xxx

xx( )

sincos cos3 3

33 3

32

tg

h) f(x) sin x x2

f’(x) cos x ∙ x2 sin x ∙2x xcos x 2sin x x4 x3

i) f(x)

j) f (x) ln x 1 x

k) f(x)

l) f(x) (1 x)3 · log2 x

f’(x) 3(1 x)2 log2 x (1 x)3 ∙ 1 · 1 ln2 x

(1 x)2 3log2 x 1 x x ln2

55. Utilitzant la derivada logarítmica, demostra que la derivada

de la funció f(x)

kg(x) és f ’(x) .

f ’(x) k ln f(x) ln k lnk ln g(x) g(x) g(x)

Per tant:

f’(x) k · g’(x) f’(x) kg’(x) g(x) g(x) [g(x)]2

56. La derivada de la funció f(x) tg x es pot expressar f’(x) 1 tg2 x. Per què?

Perquè f’(x) 1 i, 1 tg2 x 1 sin2 x 1

cos2 x cos2 x cos2 x

En conseqüència:

f’(x) 1 1 tg2 x cos2 x

57. Troba l’equació de la recta tangent al gràfic de la funció

f(x) x2

x2 1 en x 1. En quin punt del gràfic d’aquesta

funció la recta tangent és paral·lela a l’eix d’abscisses?

f(1) 12

El punt és 112

,

mtg f’(1) 2 1 4 2

Equació recta tangent:

y 1 1 (x 1) x 2y 0 2 2

Recta tangent paral·lela a l’eix 0X mtg 0 f’(x) 0

2x 0 2x 0 x 0 f(0) 0 (x2 1)2

La recta tangent és paral·lela a l’eix d’abscisses en el punt (0, 0).

Page 25: MATEMÀTIQUES - · PDF fileMatemàtiques 2 · Batxillerat · Solucionari No és permesa la reproducció total o parcial d’aquest llibre, ni el seu tractament informàtic, ni la

25MATEMÀTIQUES 2 1

58. Comprova que la derivada de la funció:

f(x) ln és f’(x)

f’(x) 1 cos x cos x 1 · 2cos x 1 2 1 sin x 1 sin x 2 cos2 x cos x

59. Calcula la derivada de les funcions:

a) f(x) e3x · ln (x2 4)

f’(x) 3e3x ln(x2 4) e3x 2x x2 4

e3x [3ln(x2 4) 2x ] x2 4

b) f(x) sin [cos (ex 2)]

f’(x) ex cos(cos(ex 2)) sin(ex 2)

c) f(x) ex 1 ex

f’(x) ex · ex ex(ex 1) –ex 1

(ex)2 ex ·ex ex

d) f(x) tg (3x 7)

f’(x) 3x ln 3 cos2(3x 7)

e) f(x) (x2 1)cos x

ln f(x) cos x · ln (x2 1)

f’(x) sin x ·ln(x2 1) cos x · 2x f(x) x2 1

f’(x) (x2 1)cos x [ sin x ·ln(x2 1) 2x cos x] x2 1

f) f(x) ln ex 3 e x

f(x) ln(ex 3) lnex ln(ex 3) x

f’(x) ex

1 ex – ex –3

3 ex 3 ex 3 ex 3

60. Donada la funció f(x) x · ex, calcula f’(x). Escriu l’equació de la recta tangent a la gràfica de f(x) en el punt on s’anul-la la seva derivada. Indica raonadament si aquesta funció és creixent o decreixent en x 0.

f’(x) ex x · ex ex(1 x)

f’(x) 0 ex (1 x) 0 1 x 0 x 1

f( 1) 1 · e 1 1

e

Punt: 1, 1e ; pendent: m 0 equació tangent: y

–1e

f’(0) 1 0 la funció és creixent en x 0.

61. Una petita mostra de material radioactiu conté 1 bilió d’àtoms. A conseqüència de la desintegració, el nombre N d’àtoms de la mostra va disminuint a mesura que passa el temps t. La funció N f(t) que descriu aquesta situació és:

N(t) N0 · e 2t

on N0 és el nombre inicial d’àtoms que hi ha a la mostra i t, el temps transcorregut en anys. Es demana:

a) Quants àtoms hi haurà a la mostra quan hagin passat 5 anys? I quan n’hagin passat 10?

t 5 anys N f(5) 1012 · e 10 45 399 930 àtoms

t 10 anys N f(10) 1012 · e 20 2 061 àtoms

b) Quan és més ràpida la desintegració, als 5 anys o als 10 anys?

N’(t) 2N0 · e 2t

La desintegració és més ràpida per a t 5 anys, ja que:

N’(5) 2 · 1012 · e 10 9,08 · 107 àtoms/s

N’(10) 2 · 1012 · e 20 4 122 àtoms/s

i, per tant, es compleix:

N ’(5) N ’(10)

62. Comprova que la derivada de la funció f(x) x2 · 5x s’anul·la

en els punts x

2ln5

i x 0.

f’(x) 2x · 5x x2 · 5x · ln5 x · 5x (2 x · ln5)

f’(x) 0 x · 5x (2 x · ln5) x 0

2 x · ln5 0

x 2 ln5

Page 26: MATEMÀTIQUES - · PDF fileMatemàtiques 2 · Batxillerat · Solucionari No és permesa la reproducció total o parcial d’aquest llibre, ni el seu tractament informàtic, ni la

26 SOLUCIONARI DEL LLIBRE DE L’ALUMNE1

63. Calcula la derivada de les funcions:

a) f(x) arc sin 1 x 1 x

b) f(x) earc tg x

f’(x) earc tg x · 1 earc

tg

x

1 x2 1 x2

c) f(x) ln [arc cos (x 1)]

f’(x) 1 · ( 1) arc cos(x 1) 1 – (x 1)2

1 arc cos (x 1) 2x x2

d) f(x) arc tg 1 x arc tg x 1 x

f’(x) 1 · 1 x (1 x) · ( 1) 1 1 1 x 2 (1 x)2 1 x2

1 x

(1 x)2 · 2 1

(1 x)2 (1 x)2 (1 x)2 1 x2

2 1 1 1 0 2(1 x2) 1 x2 1 x2 1 x2

e) f(x) · arc sin x

f) f(x) arc cos (cos x)

f(x) arc cos(cos x) x f’(x) 1

64. Dedueix la derivada de la funció g(x) ax sabent que la seva funció inversa és f(x) loga x i suposant coneguda f’(x).

f(x) loga x f’(x) 1 · 1 lna x

g’(x) 1 1 ax lna f’(f 1(x)) 1 · 1 lna ax

65. Donada la funció f(x) x2

x2 4, calcula f’(x), f’’(x) i f’’’(x).

f’(x) 2x(x2 –4) x2 ·2x 8x (x2 4)2 (x2 4)2

f’’(x) –8(x2 –4)2 8x ·2 ·2x(x2 –4) (x2 4)4

–8(x2 –4) 32x2 24x2 32

(x2 4)3 (x2 4)3

f’’’(x) 48x(x2 4)3 (24x2 32)(x2 4)2 ·3 ·2x (x2 4)6

48x(x2 –4) (24x2 32)6x (x2 4)4

48x3 192x 144x3 192x (x2 4)4

–96x3 384x –96x(x2 4) (x2 4)4 (x2 4)4

66. Troba f(66)(x) i g(94)(x) per a les funcions f(x) sin x i g(x) cos x.

f(66)(x) f’’(x) sin x

g(95)(x) g(3)(x) sin x

67. Per a la funció f(x) 2x, calcula:

f’(x), f’’(x), f’’’(x) i f(4)(x)

Observa amb detall les funcions que has obtingut i dedueix l’expressió de la derivada f(n)(x).

Page 27: MATEMÀTIQUES - · PDF fileMatemàtiques 2 · Batxillerat · Solucionari No és permesa la reproducció total o parcial d’aquest llibre, ni el seu tractament informàtic, ni la

27MATEMÀTIQUES 2 1

Activitats finals

1. En quins punts de la gràfica de la funció f(x) 1 x

la recta

tangent és perpendicular a la recta 4x y 2 0? Escriu les equacions d’aquestes rectes tangents.

4x y 2 0 m 4 m’ 1 4

f(x) 1 x ƒ’ (x) –1

x2

m’ ƒ’(x) 1 4 –1

x2 No té solució.

En cap punt la gràfica de f(x) té una tangent perpendicular a 4x y 2 0.

2. Dibuixa en un paper mil·limetrat la gràfica de la funció f(x) x2 8x. Tot seguit, fes una estimació a partir d’aques-ta gràfica dels valors de f’(1) i f’(5). Calcula analíticament f’(1) i f’(5) i compara els resultats amb els anteriors.

f(x) x2 8x f’(x) 2x 8f’(1) 2 8 6; f’(5) 10 8 2

Cal comparar aquests valors amb els valors obtinguts de manera experimental, a partir de la gràfica de la funció.

3. Representa gràficament la funció:

f(x)

0 si x 0

x si x 0

Aquesta funció és contínua en x 0? I derivable? Justifica les respostes.

lim f(x) 0; lim f(x) 0; f(0) 0x 0 x 0

Es compleix: lim f(x) f(0) 0 x 0

Per tant, la funció és contínua en x 0.

f ’(x) {

0 si x 0

1 si x 0

És a dir, f’(0 ) 0 i f’(0 ) 1 f’(0 ) f’(0 )la funció no és derivable en x 0.

4. Indica en quins punts és derivable la funció:

És derivable en tot R excepte en x 0. Es compleix que f’(x) 0 per a x R, x 0.

5. La gràfica d’una funció f(x) és la de la figura. Sense calcular-ne l’expressió analítica, representa gràficament la funció f’(x).

6. Troba les derivades laterals en x 5 de la funció f(x) |2x 10|. És derivable en aquest punt? Per què?

2x 10 si x 5f(x) |2x 10| { 2x 10 si x 5

f’(5 ) 2; f’(5 ) 2 f’(5 ) f’(5 )

La funció no és derivable en x 5.

7. Indica els intervals de creixement i decreixement i els punts estacionaris de la funció f(x)

x2

x2 4.

f’(x) 2x(x2 – 4) x2 ·2x 8x

(x2 4)2 (x2 4)2

Df R { 2, 2}; f’(x) 0 x 0, f(0) 0 (0, 0)

Com que f’(x) 0 per a x 0, x 2 i f’(x) 0per a x 0, x 2, es compleix que:

La funció és creixent en els intervals ( , 2) i ( 2, 0).La funció és decreixent en els intervals (0, 2) i (2, ).La funció presenta un punt estacionari a l’origen de coordenades.

Page 28: MATEMÀTIQUES - · PDF fileMatemàtiques 2 · Batxillerat · Solucionari No és permesa la reproducció total o parcial d’aquest llibre, ni el seu tractament informàtic, ni la

28 SOLUCIONARI DEL LLIBRE DE L’ALUMNE1

8. La funció f(x) x2

(x 1)2 és creixent o decreixent en x 2?

Justifica la resposta.

f’(2) 4 0 f(x) és decreixent en x 2.

9. Donada la paràbola d’equació f(x) x2 2x 5, es considera la recta r que uneix els punts d’aquesta paràbola, les abscis-ses dels quals són x1 1 i x2 3. Troba l’equació de la recta tangent a la paràbola que és paral·lela a la recta r.

x1 1 f(x1) f(1) 4 (1, 4)

x2 3 f(x2) f(3) 8 (3, 8)

La recta r conté els punts (1, 4) i (3, 8)

mr 2

f’(x) 2x 2 i f’(x) mr 2 2x 2 2 x 2f(2) 5 El punt de tangències és (2, 5)

mtg mr 2

Equació de la recta tangent:y 5 2 (x 2) y 5 2x 4 2x y 1 0

10. Aquesta és la representació gràfica de la derivada f’(x) d’una funció polinòmica f(x).

a) Quin és el grau d’aquesta funció polinòmica? Per què?

Grau 2. Perquè f’(x) és una funció polinòmica de primer grau, ja que la seva representació gràfica és una recta.

b) Indica els intervals de creixement i decreixement de la funció f(x).

f’(x) 0 x 2 Creixent: (2, )

f’(x) 0 x 2 Decreixent: ( , 2)

c) Té f(x) algun punt estacionari? Quin és?

Sí, x 2, ja que ƒ’(2) 0.

11. Donada la funció f(x) x · ex, resol les equacions f’(x) 0 i f’’(x) 0.

f’(x) ex x · ex ex (1 x)

f’’(x) ex (1 x) ex ex (2 x)

f’(x) 0 ex (1 x) 0 1 x 0 x 1

f’’(x) 0 ex (2 x) 0 2 x 0 x 2

12. Troba per a quin valor de a i b és contínua i derivable la funció:

Contínua:

lim f(x) lim f(x) f(1)x 1 x 1

3 a a 3

Derivable:

f’(1 ) f’(1 )

3 2a b b 3 2a 3 6 3

13. Calcula la derivada de les funcions:

a) f(x) sin4 [ln (x2 5)]

b)

Page 29: MATEMÀTIQUES - · PDF fileMatemàtiques 2 · Batxillerat · Solucionari No és permesa la reproducció total o parcial d’aquest llibre, ni el seu tractament informàtic, ni la

29MATEMÀTIQUES 2 1

c)

d)

e) f(x) [1 cos2 (1 3x)]2

f) f(x) log2

g) f(x) sec2 (x3 2)

h) f(x) arc sin

i) f(x) arc tg

Page 30: MATEMÀTIQUES - · PDF fileMatemàtiques 2 · Batxillerat · Solucionari No és permesa la reproducció total o parcial d’aquest llibre, ni el seu tractament informàtic, ni la

30 SOLUCIONARI DEL LLIBRE DE L’ALUMNE1

j) f(x) 2arc sin x ·

k) f(x) (x2 3)x 5

l) f(x) ln

m) f(x) 1

3sin x

n) f(x)

o) f(x)

p) f(x) etg 3x ·

14. El nombre N de bacteris d’un determinat cultiu varia en fun-ció del temps t expressat en hores, d’acord amb l’equació:

a) Quin és el nombre inicial de bacteris en el cultiu?

t 0 N(0) 10 bacteris.

b) En quin moment creix més de pressa el nombre d’aquests bacteris, quan t 2 h o quan t 4 h? Per què?

t 2 h N’(2) 5e

t 4 h N’(4) 5e2

N’(4) N’(2) el nombre de bacteris creix més de pressa

per a t 4 h.

15. Calcula les tres primeres derivades de la funció f(x) e3x. Dedueix l’expressió de la derivada enèsima f(n)(x) d’aquesta funció.

f’(x) 3e3x

f’’(x) 9e3x

f’’’(x) 27e3x

f (n)(x) 3ne3x

16. Tenint en compte que arc sec x arc cos , calcula la deri-

vada de la funció f(x) arc sec x. De manera similar, pots calcular les derivades de les funcions g(x) arc cosec x i h(x) arc cotg x. Fes-ho.

arc cosec x arc sin

Page 31: MATEMÀTIQUES - · PDF fileMatemàtiques 2 · Batxillerat · Solucionari No és permesa la reproducció total o parcial d’aquest llibre, ni el seu tractament informàtic, ni la

31MATEMÀTIQUES 2 1

h(x) arc cotg x arc tg

17. Troba l’equació de la recta normal a la gràfica de la funció f(x) x2 7x 10 en els punts d’ordenada nul·la.

f(x) 0 x2 7x 10 0 x1 2, x2 5 (2, 0) i (5, 0)f ’(x) 2x 7

Punt (2, 0):

Equació recta normal:

Punt (5, 0):

Equació recta normal:

18. Representa gràficament la funció f(x) x2 2. Hi ha algun punt en el qual aquesta funció no sigui derivable? Justifi-ca’n la resposta.

No. La funció és contínua i derivable a tot R.

La seva gràfica és la mateixa que la de la funció g(x) x2 2, ja que f(x) 0 per a tot x R.

19. Determina l’expressió algèbrica de la funció f(x) que verifica les condicions següents:

a) f’(x) 3

b) El seu gràfic passa pel punt P(2, 10)

f(x) 3x n

f(2) 10 10 6 n n 10 6 4

Per tant, f(x) 3x 4

20. Indica raonadament per què la funció f(x) , on a i b

són nombres reals, no pot tenir punts estacionaris.

La funció no pot tenir punts estacionaris, ja que f’(x) no s’anul·la per a cap valor de x real.

21. Se sap que la funció f(x) ax2 bx 12 presenta un mínim en el punt P(4, 4). Calcula a i b.

f(4) 4 16a 4b 12 4 4a b 4

f’(x) 2ax b; f’(4) 0 8a b 0

22. Dibuixa de manera aproximada la gràfica de la funció f(x) ln |x|. Indica raonadament si hi ha algun punt en què aquesta funció no sigui derivable.

La funció no és derivable en x 0 perquè no existeix f(0) i, per tant, no pot ser-hi contínua.

23. Justifica el motiu pel qual la funció f(x) no és deri-vable en x 0.

Perquè no existeix f’(0).

De fet, com que no existeix limx 0

f(x), la funció no és contínua en

x 0 i, per tant, no pot existir f’(x).

24. Representa gràficament la funció f(x) log2 x i, a partir d’aquesta gràfica, dibuixa la funció g(x) |log2 x|. Per a quins valors de x no existeix g’(x)? Per què?

Page 32: MATEMÀTIQUES - · PDF fileMatemàtiques 2 · Batxillerat · Solucionari No és permesa la reproducció total o parcial d’aquest llibre, ni el seu tractament informàtic, ni la

32 SOLUCIONARI DEL LLIBRE DE L’ALUMNE1

La funció g(x) no és derivable en x 1.Observa a partir de la gràfica que g’(1 ) g’(1 ).

25. La funció: f(x)

és derivable en x 2?

Per què?

f(2) 3

Com que limx 2

f(x) f(2), la funció no és contínua en x 2. Ales-

hores, tampoc pot ser derivable en aquest punt.

Avaluació

1. Calcula les derivades de les funcions següents, simplificant al màxim:

a) f

f

b) f

f

c) f

f

d) f

f

2. Donada la funció x, resol l’equació

f ’(x) 0 en .

f(x) x

f’(x) 1

ƒ’(x) 0 1 0

Per tant,

x x

x x

3. Calcula les equacions de les dues rectes del pla que passen pel punt P(1, 1) i que són tangents a la corba d’equació y (x 1)2.

Els punts de la gràfica de y (x 1)2 són de la forma Q (x, (x 1)2). El pendent de la recta tangent en cada un d’aquests punts val y’(x) 2(x 1). El pendent de la recta que

uneix P amb un d’aquests punts Q serà .

Si es vol que una de les rectes PQ sigui tangent a la gràfica de

y (x 1)2 s’ha de complir l’equació que

té com a úniques solucions: a) x 0 i b) x 2.

En el cas a) el pendent de la recta ha de ser y’(0) 2 i en el cas b) és y’(2) 2. Les rectes corresponents, que passen per P i tenen pendents 2 i 2, tindran com a equacions:

(y 1) 2(x 1) 2x y 1 0

(y 1) 2(x 1) 2x y 3 0

4. Considera la funció f(x) x3 3x2 2x 2. Calcula l’equació de la recta tangent a la gràfica de f(x) en el punt d’abscissa x 3.

La imatge del punt d’abscissa x 3 de la funció és f(3) 33 3 . 32 2 . 3 2 8 P(3, 8) i la derivada de la funció: f’(x) 3x 2 6x 2.

El pendent de la recta tangent en aquest punt és: m f(3) 3 . 32 6 . 3 2 11

i l’equació de la recta: y 8 11(x 3) y 11x 25

Page 33: MATEMÀTIQUES - · PDF fileMatemàtiques 2 · Batxillerat · Solucionari No és permesa la reproducció total o parcial d’aquest llibre, ni el seu tractament informàtic, ni la

33MATEMÀTIQUES 2 2

j Unitat 2. Funcions contínues i derivables

Activitats

1. A partir de les funcions f(x) x2 1 i g(x) x3 1, escriu

les funcions (f g)(x), (f · g)(x), i

fg

x( ). Són contínues?

Raona la teva resposta.

(f g)(x) x2 1 x3 1 x3 x2

(f · g)(x) (x2 1)(x3 1) x5 x3 x2 1

Les dues funcions són contínues per ser polinomis.

presenta una discontinuïtat asimptòtica a

x 1, valor que anul·la el denominador.

2. Descompon la funció f(x) 5x4 · ex en tres factors que siguin funcions contínues.

Es poden donar diferents resultats. Per exemple: m(x) 5x, g(x) x3 i h(x) ex.

3. La funció f(x) tg x és contínua? Recorda que tg x .

f(x) tg x no és contínua en els valors de x que fan

cos x 0 x (2k 1) , amb k un nombre enter.

4. Considera les funcions f(x) 2x 1 i g(x) x2 1. Escriu les

funcions fg

x( ), gf

x( ), (f ° g)(x) i (g ° f)(x). Raona

si les funcions obtingudes són contínues.

és contínua.

presenta una discontinuïtat a x 0, valor que

anul·la el denominador.

(f ° g)(x) 2x2 1 1 és contínua.

(g ° f)(x) (2x 1)2 1 és contínua.

5. Explica un fet quotidià que posi de manifest el teorema dels valors intermedis.

Per exemple, en una etapa ciclista els corredors passen per un quilòmetre determinat.

6. Considera la funció f(x) 2x4 14x2 14x 1. Explica per què es pot aplicar el teorema de Bolzano en l’interval [0, 1]. Troba un valor aproximat a les centèsimes de c tal que f(c) 0 en aquest interval.

La funció f(x) és contínua i verifica: f(0) 1 i f(1) 1. Es verifica el teorema de Bolzano en l’interval [0,1].

Utilitzant la calculadora per trobar valors numèrics tenim que f(0,1) 0,26, per tant, el valor c buscat es troba entre 0 i 0,1. El valor de c 0,08 dóna f(0,08) 0.

7. Separa les quatre arrels reals de la funció següent:

f(x) 2x4 13x2 15

En la funció tenim: f(1) 4, f(2) 5 i f(3) 60 igualment per la paritat de les potències de x tenim: f( 1) 4, f( 2) 5 i f( 3) 60. Els intervals que separen les quatre arrels són: [1,2], [ 2, 1], [2,3] i [ 3, 2].

8. Calcula els valors de f(x) x7 3x 3 a x 0 i x 1. Pots determinar si la gràfica de la funció talla l’eix de les abscis-ses en algun punt entre 1 i 0? Troba aquest punt amb una aproximació fins a les centèsimes.

f(0) 3 i f( 1) 1. Pel teorema de Bolzano en l’interval [ 1,0] la gràfica de la funció talla en un punt l’eix de les abscisses.

Calculant valors numèrics de la funció per a diferents valors de x de l’interval, s’obté c 0,87.

9. Considera la funció f(x) x2 2x 1. És una funció contínua que té com a gràfica una paràbola. Existeix un punt c tal que f(c) 0? Explica si en aquesta funció es pot aplicar el teorema de Bolzano en l’interval [0, 2].

La funció verifica f(1) 0 c 1. No es pot aplicar el teorema de Bolzano en l’interval [0, 2] ja que f(0) 1 f(2).

10. Troba el màxim i el mínim absoluts de la funció f(x) x2 2x en l’interval [ 1, 2]. Representa gràficament la funció per ajudar-te a trobar la solució.

En l’interval [ 1, 2] es verifica: f( 1) 3, f(2) 0 i f(1) 1 que és el màxim absolut i vèrtex de la paràbola. El mínim absolut es troba a x 1, un dels extrems de l’interval. La gràfica és:

3

Page 34: MATEMÀTIQUES - · PDF fileMatemàtiques 2 · Batxillerat · Solucionari No és permesa la reproducció total o parcial d’aquest llibre, ni el seu tractament informàtic, ni la

34 SOLUCIONARI DEL LLIBRE DE L’ALUMNE2

11. Verifica si la funció f(x) tg x té màxim i mínim absoluts en

l’interval . Raona la teva resposta.

La funció f(x) tg x no és contínua a x . Té mínim absolut

a x 0 f(0) 0. No té màxim absolut a x per la dis-

continuïtat.

12. Troba els punts de la funció en els quals no sigui derivable.

La funció no és contínua a x 1 i x 1, valors que no són del domini; per tant, no és derivable en aquests punts.

13. Considera la funció f(x) 3x4 8x3 6x2. Troba’n els punts estacionaris i classifica’ls.

Calculem la derivada de la funció i la igualem a 0.

f’(x) 12x3 24x2 12x 12x3 24x2 12x 0

12x(x2 2x 1) 0

Per a x < 0 f’(x) < 0, i per a x > 0 f’(x) > 0; per tant, a x 0 hi ha un mínim relatiu.

Per a x < 1 f’(x) > 0, i per a x > 1 f’(x) > 0; per tant, a x 1 hi ha un punt d’inflexió de tangent horitzontal.

En ambdós casos es consideren valors de l’entorn de 0 i 1, res-pectivament.

14. Interpreta el valor de la derivada de la funció y x3 1 en el punt x 0.

La derivada y’ 3x2. En el punt x 0 s’anul·la la derivada i per a valors anteriors i posteriors de l’entorn de x 0, la derivada és positiva. A x 0 hi ha un punt d’inflexió de tangent horitzontal.

15. Troba la derivada de les funcions f(x) e2x i de g(x) ln x. Tenen punts estacionaris aquestes funcions? Raona’n la resposta.

f’(x) e2x · 2 i la funció no té punts estacionaris ja que la deri-vada no s’anul·la per a cap valor de x. Igualment passa amb la

funció g(x) ln x, ja que la derivada g’(x) 1x

no s’anul·la.

16. Considera la funció f(x) 2 sin x en l’interval [0, ]. Aplica-li el teorema de Rolle per trobar un punt c tal que f’(c) 0.

f(0) 0 i f( ) 0. El teorema de Rolle afirma que hi ha un punt de l’interval (0, ) en el qual la derivada s’anul·la.

17. Esbrina si la funció verifica les condicions

del terorema de Rolle a l’interval [0, 2].

La funció presenta una discontinuïtat en el punt x 1; per tant, la funció no és contínua en l’interval [0, 2] i no es pot aplicar el teorema de Rolle.

18. Considera la funció f(x) x3 3x2 en l’interval [0, 3] i apli-ca el teorema de Rolle en aquest interval. Quin és el punt c que prediu el teorema?

Hi ha algun altre punt que no pertany a (0, 3) en què també s’anul·li la derivada?

f(0) 0 i f(3) 0 es pot aplicar el teorema de Rolle.

f’(x) 3x2 6x 3x2 6x 0

3x(x 2) 0

c 2 (0, 3) i 0 (0, 3)

19. Demostra que a la funció f(x) se li pot aplicar el teo-rema del valor mitjà en l’interval (0, 1). Troba el punt c de l’interval en què f’(c) 1. Troba l’equació de la recta tangent a la corba en aquest punt.

f’(x) és contínua en l’interval (0, 1) i f(0) 0 ≠ f(1) 1; per tant, es pot aplicar el teorema del valor mitjà.

Equació de la recta tangent: punt , m 1

20. Demostra que la funció f(x) és decreixent en tot el seu domini.

. L’expressió de la derivada per a qualsevol

valor de x; per tant, la funció és decreixent.

21. Comprova que el punt és el punt on es verifica el te-

orema de Cauchy per les funcions següents f(x) 3x 2 i g(x) x2 1 en l’interval [1, 4].

Page 35: MATEMÀTIQUES - · PDF fileMatemàtiques 2 · Batxillerat · Solucionari No és permesa la reproducció total o parcial d’aquest llibre, ni el seu tractament informàtic, ni la

35MATEMÀTIQUES 2 2

22. Troba els punts de la funció f(x) x3 4x 1 que verifiquen f’(x) 0.

Classifica’ls i expressa els intervals de monotonia i concavi-tat.

f’’(x) 6x Mínim relatiu a

Màxim relatiu a

f’’(0) 0 x 0 és un punt d’inflexió.

f(x) és creixent: i

decreixent:

convexa: ( ∞,0); còncava: (0, ∞)

23. Estudia la primera i la segona derivada de la funció f(x) ln (x2 1) per trobar possibles màxims o mínims rela-tius i punts d’inflexió. Vés amb compte a l’hora d’interpretar els valors que anul·len la segona derivada.

f’’(0) > 0 a x 0 hi ha un mínim relatiu.

Els punts x ±1 són punts d’inflexió, encara que no de tangent horitzontal.

24. Troba els extrems relatius i els punts d’inflexió de les funcions:

a) f(x)

f’(x) 1 1 0 x2 1 0 x ±1

f’’(x)

f’’(1) > 0, a x 1 hi ha un mínim relatiu.

f’’( 1) < 0, a x 1 hi ha un màxim relatiu.

No hi ha punts d’inflexió ja que:

b) f(x) x (x 1)2 (x 2)3

La funció és: f(x) x(x 1)2 (x 2)3

f’(x) (x 2)2 (x 1) (6x2 10x 2)

f’(x) 0 x 2 x 1

x 5 13

6

A x 5 13

6 hi ha un mínim relatiu i absolut; a x 1, un

màxim relatiu; a x 5 13

6, un mínim relatiu; i a x 2,

un punt d’inflexió.

c) f(x) ex · x

f’(x) ex x ex ex(x 1) x 1 0 x 1

f’’(x) ex(x 1) ex ex(x 2) x 2 0 x 2

f’’( 1) e 1 > 0 a x 1 hi ha un mínim relatiu i a x 2 hi ha un punt d’inflexió.

d) f(x) cos x amb x [0, 2 ]

f’(x) sin x sin x 0 x 0

2f’’(x) cos x

f’’(0) < 0 a x 0 hi ha un màxim relatiu.

f’’( ) > 0 a x hi ha un mínim relatiu.

f’’(2 ) < 0 a x 2 hi ha un màxim relatiu.

25. Calcula els límits següents:

a)

b)

tg x

c)

Page 36: MATEMÀTIQUES - · PDF fileMatemàtiques 2 · Batxillerat · Solucionari No és permesa la reproducció total o parcial d’aquest llibre, ni el seu tractament informàtic, ni la

36 SOLUCIONARI DEL LLIBRE DE L’ALUMNE2

d)

26. Calcula els límits següents aplicant la regla de l’Hôpital sa-bent que són una potència del nombre e.

Cal trobar l’exponent k de ek en cada cas.

a)

Derivant numerador i denominador tot aplicant la regla de L’Hôpital, s’obté k 2. Per tant, e2 és el resultat.

b)

e3

c)

e1 e

d)

e6

Activitats finals

1. Raona la continuïtat de les funcions:

a) f(x) ln (x2 1)

f(x) ln(x2 1) és contínua per a tot x del domini, ja que x2 1 > 0.

b) f(x) sin x · ex 1

f(x) (sin x) · ex 1 és contínua ja que és el producte de dues funcions contínues.

c) f(x)

f(x) és contínua per a tot x ≠ 1. Per a x 1 pre-

senta una discontinuïtat asimptòtica.

d) f(x) cos2 x cos x 1

f(x) cos2 x cos x 1 és contínua per ser suma de tres funcions contínues.

2. La funció f(x) x2 x 1 és contínua. Explica si es pot apli-car el teorema de Bolzano en algun interval. Té alguna arrel l’equació f(x) 0?

L’expressió x2 x 1 > 0 per a tot x i, per tant, no es pot aplicar el teorema de Bolzano i l’equació f(x) 0 no té cap arrel.

3. Raona si la funció f(x) x6 6x2 3 té alguna arrel entre 0 i 1. Troba aquest valor amb una aproximació fins a les centèsimes.

Apliquem el teorema de Bolzano ja que:

f(0) 3; f(1) 2 i la funció és contínua.

Existeix un c [0, 1] tal que f(c) 0.

Utilitzant la calculadora per trobar valors numèrics de la funció per a valors de x de l’interval, s’obté com a valor aproximat c 0,72.

4. Troba els extrems absoluts de f(x) ex 1 en l’interval [ 1, 1].

f( 1) e 1 1 0,63; f(1) 1,718

f’(x) ex f(x) no té extrems relatius ja que ex ≠ 0, per tant,

els extrems absoluts es troben en els extrems de l’interval: a

x 1 hi ha el mínim absolut i a x 1 el màxim absolut.

5. Estudia la derivabilitat de la funció f(x) en el punt x 0.

f(x) és contínua per a tot x del domini: Df [ 1, ).

no està definida a x 1, per tant, no és de-

rivable en aquest punt. A x 0 no és derivable.

Page 37: MATEMÀTIQUES - · PDF fileMatemàtiques 2 · Batxillerat · Solucionari No és permesa la reproducció total o parcial d’aquest llibre, ni el seu tractament informàtic, ni la

37MATEMÀTIQUES 2 2

6. Demostra que f(x) és decreixent en tot el seu domini.

per a tot x del domini.

Si la derivada és negativa, la funció és decreixent.

7. Calcula les tres primeres derivades de f(x) . Troba una expressió per a la derivada enèsima.

x13

x

23

x

53

x

83 ;

a a

xnn

an1 3

1

3

...

amb an 3n 4

8. Troba l’equació de la recta tangent a la corba següent: y x3 3x en el punt d’abscissa 1.

Punt de tangència: P( 1, 2); pendent: m y’( 1). Equació de la recta: y 2.

9. Esbrina si f(x) és creixent en tot el seu domini.

Què passa en el punt x 1?

. La derivada és positiva i la funció és creixent

per a x > 1; és negativa i la funció és decreixent per a x < 1. En el punt x 1 hi ha una discontinuïtat asimptòtica.

10. Calcula la derivada de les funcions següents:

a) y

b) y sin 3x · tg 3x

y sin 3x · tg 3x

y’ 3 · cos 3x · tg 3x sin 3x ·

3 · sin 3x ·

11. Raona per què la funció f(x) 2x cos x no pot tenir mà-xims ni mínims relatius.

f’(x) 2 sin x > 0 ja que 1 sin x 1 i la derivada no s’anul·la per a cap valor de x.

12. Estudia la monotonia i dóna els intervals de creixement i decreixement de les funcions:

a) f(x) 1 2x 3x2

f’(x) 2 6x 2 6x > 0

Per a x < la funció és creixent; per a x > , la funció és

decreixent: ( , ) i ( , ), respectivament.

b) f(x) x sin x

f’(x) 1 cos x 0, la funció és creixent per a tot x ≠ en [0, 2 ].

c) f(x) x2 ln x2

f’(x) 2x

f(x) és creixent en ( 1, 0) i (1, ) i decreixent en ( , 1) i (0, 1).

d) f(x) x4 x2

És la funció de l’activitat 14 d). Aprofitant els extrems rela-tius establim que f(x) és:

Creixent: i

Decreixent: i

13. Dóna un raonament per tal de justificar que la funció f(x) x5 5x3 2x talla l’eix de les abscisses en un sol punt.

f(x) x · (x4 5x2 2) f(x) 0 x 0, que és el punt on talla l’eix de les abscisses; x4 5x2 2 > 0 per a tot x i, per tant, la gràfica no talla a cap altre punt l’eix de les abscisses.

14. Classifica els possibles extrems relatius de les funcions:

a) f(x) 2sin x cos 2x

f’(x) 2 · cos x 2 · sin 2x cos x sin 2x 0 cos x 2 · sin x · cos x 0

cos x · (1 2 · sin x) 0

x1

x2

x3

x4

f’’(x) 2 · sin x 4 · cos 2x

hi ha un mínim relatiu.

Page 38: MATEMÀTIQUES - · PDF fileMatemàtiques 2 · Batxillerat · Solucionari No és permesa la reproducció total o parcial d’aquest llibre, ni el seu tractament informàtic, ni la

38 SOLUCIONARI DEL LLIBRE DE L’ALUMNE2

hi ha un mínim relatiu.

hi ha un màxim relatiu.

hi ha un màxim relatiu.

b) f(x) x4 e x

f’(x) 4x3 ·e x x4 · e x( 1) x3 · e x(4 x)

x3 · e x(4 x) 0

f’’(x) 3x2 · e x(4 x) x3 · e x(4 x) x3 · e x

f’’(0) 0, a x 0 hi ha un punt d’inflexió.

f’’(4) < 0, a x 4 hi ha un màxim relatiu.

c) f(x) x3 5x2 6x

f’(x) 3x2 10x 6 3x2 10x 6 0 x

f’’(x) 6x 10

hi ha un mínim relatiu.

hi ha un màxim relatiu.

d) f(x) x4 x2

f’(x) 4x3 2x 4x3 2x 0

2x(2x2 1) 0

x1 0 x2 x3

f’’(x) 12x2 2

f’’(0) < 0, a x 0 hi ha un màxim relatiu.

; a x i x

hi ha mínims relatius.

15. Determina la concavitat i els punts d’inflexió de les funcions:

En cada cas cal trobar els punts en els que s’anul·len les deri-vades primera i segona.

a) f(x) x3 2x2 4x 8

f ’(x) 3x2 4x 4 3x2 4x 4 0 x1 2 i x2

f ’’(x) 6x 4 0 x és un punt d’inflexió.

f ’’( 2) < 0 hi ha un màxim relaitu a x 2.

f ’’ > 0 hi ha un mínim relatiu a x .

f(x) és convexa en , i còncava en , .

b) f(x) x3 2

f ’(x) 3x2; f ’’(x) 6x x 0 és un punt d’inflexió de tan-

gent horitzontal i de canvi de concavitat.

f(x) és convexa en ( , 0) i còncava en (0, ).

c) f(x) x cos x

f ’(x) 1 sin x i f ’’(x) cos x

cos x 0

f(x) és convexa a (0, ) i ( , 2 ) i còncava a ( , ).

d) f(x) x4 x2

f’(x) 4x3 2x

f’’(x) 12x2 2 12x2 2 0 x

tenint en compte els extremes relatius de la funció trobats a l’activitat 14 d) podem establir:

f(x) és còncava a , i , ,

i convexa a .

Page 39: MATEMÀTIQUES - · PDF fileMatemàtiques 2 · Batxillerat · Solucionari No és permesa la reproducció total o parcial d’aquest llibre, ni el seu tractament informàtic, ni la

39MATEMÀTIQUES 2 2

16. Determina els punts d’inflexió de la funció següent:

6x2 2 0 x ±

Hi ha dos punts d’inflexió.

17. Calcula la primera i la segona derivada de la funció f(x) (x 1)3. S’anul·len les dues derivades en un mateix punt? Troba aquest punt i explica de quin tipus és.

f ’(x) 3(x 1)2

f ’’(x) 6(x 1)

Les dues derivades s’anul·len per a x 1. En aquest punt hi ha una inflexió de tangent horitzontal i de canvi de concavitat.

18. Considera la funció f(x) x3 ax2 bx 7. Troba a i b de manera que la gràfica de la funció tingui a x 1 un punt d’inflexió de tangent horitzontal.

Els punts estacionaris anul·len la primera derivada:

f’(x) 3x2 2ax b

f’(1) 3 2a b 0 2a b 3

Els punts d’inflexió anul·len la segona derivada:

f ’’(x) 6x 2a

f’’(1) 6 2a 0 a 3 b 3

La funció té un punt d’inflexió a x 1 per a b 3.

19. Determina f(x) sabent que la derivada tercera és f’’’(x) 24x, f(0) 0, f’(0) 1 i f’’(0) 2.

f(x) és un polinomi de quart grau ja que la tercera derivada és de primer grau:

f(x) ax4 bx3 cx2 dx e

f(0) 0 e 10

f’(x) 4ax3 3bx2 2cx d

f’(0) 1 d 1

f’’(x) 12ax2 6bx 2c

f’’(0) 2 2c 2 c 1

f’’’(x) 24ax 6b 24x a 1 i b 0

Substituint: f(x) x4 x2 x

20. Analitza la continuïtat i la derivabilitat de la funció

f(x) si x ≠ 0 i f(0) 0.

La funció presenta una discontinuïtat de salt a x 0.No és derivable a x 0 ja que no és contínua en aquest punt.

21. Calcula:

a) (1 cos x)2x

És un limit del tipus eK.

lim( cos )x

xx0

2 01 1e

b) (sin x)tg x

És del tipus eK.

lim(sin )x

xx2

0 1tg e

c) limln( )x

x x

x0

3

1e e

limln( )

lim

li

x

x x

x

x x

xx

0

3

0

3

131

1

e e e e

m( )( )x

x xx0

31 3 4e e

d) (x4 ln x)

(x4 ln x)

Page 40: MATEMÀTIQUES - · PDF fileMatemàtiques 2 · Batxillerat · Solucionari No és permesa la reproducció total o parcial d’aquest llibre, ni el seu tractament informàtic, ni la

40 SOLUCIONARI DEL LLIBRE DE L’ALUMNE2

Avaluació

1. (Curs 2003–04) Considera la funció on a és un nombre real.

a) Calcula el valor del nombre real a sabent que f(x) té un extrem relatiu en el punt d’abscissa x 3.

Hem de tenir f’(3) 0. Ara, f’(x) . O sigui que

f’(3) . Igualant a 0, tenim a -- 4.

b) Aquest extrem relatiu, es tracta d’un màxim o d’un mí-nim? Raona la resposta.

Per veure el caràcter de l’extrem, calculem la derivada segona,

f’’(x)

f’’(3) > 0, per tant, l’extrem és un mínim relatiu.

2. La gràfica següent correspon a una funció f :[2, 6] de-rivable i amb derivada contínua. Fes un esbós de la gràfica de f ’:(2, 6) i justifica’n el per què.

És la gràfica d’una funció més o menys en forma de paràbola decreixent a i creixent a ; que s’anul la a 3 i a 5 i amb un mínim a 4.

3. Considera la funció definida per

sisi

on a és un nombre real.

a) Calcula f(x) i comprova que f(x) és contínua en x 0.

Com que f(0) 1 i eax 1 independentment del

valor que tingui a, la funció f(x) és contínua.

b) Per a quin valor del nombre a, la funció f(x) és derivable en x 0 ?

Com que la derivada de eax és aeax, que en x 0 val a, i la deri-vada de 2x 1 és constant i igual a 2, la funció f(x) és derivable només quan a 2.

4. (Curs 2002–03) Com a resultat del test efectuat amb un nou model d’automòbil per determinar-ne el consum de benzina, s’ha observat que, per a velocitats compreses entre 25 i 175 km/h, el consum C(x) de gasolina, ex-pressat en litres consumits en 100 km, fets a la velo-citat constant de x km/h, es pot aproximar per la funció C(x) 7,5 0,05x 0,00025x2.

a) Determina el consum a les velocitats de 50 km/h i de 150 km/h.

C(50) 7,5 0,05 · 50 0,00025·502 5,625 litres en 100 km

C(150) 7,5 0,05 · 150 0,00025·1502 5,625 litres en 100 km

b) A quina velocitat s’obté el mínim consum? Quin és aquest consum mínim?

La gràfica de C(x) és una paràbola que té un mínim absolut. Per determinar-lo igualem la derivada C’(x) 0,05 0,0005x a 0 i obtenim x 100 km/h. Per a aquesta velocitat el consum és C(100) 5 litres en 100 km, que serà el consum mínim.

c) Fes un estudi del creixement i decreixement de la funció C(x) a l’interval [25,175]. Determina les velocitats que corresponen a consum màxim, així com aquest consum.

La funció derivada C ’(x) s’anul la per a x 100, on té el mínim. Per a valors de x inferiors, esdevé negativa, ja que el coeficient de la x és positiu i per tant, al disminuir la x a partir del valor que anul·la la derivada, aquesta esdevin-drà negativa. A l’inrevés passa quan x s’incrementa a partir d’aquest valor. Per tant la funció és decreixent en l’interval ( , 100) i creixent en l’interval (100, ), i assoleix el mínim absolut i relatiu en el punt (100, 5). El màxim absolut en l’interval [25, 175] s’assolirà en un dels dos extrems de l’interval (o en tots dos).

Obtenim els valors C(25) C(175) 6,41 en 100 km, que és el consum màxim que s’assolirà per a les dues velocitats de 25 i 175 km/h.

Page 41: MATEMÀTIQUES - · PDF fileMatemàtiques 2 · Batxillerat · Solucionari No és permesa la reproducció total o parcial d’aquest llibre, ni el seu tractament informàtic, ni la

41MATEMÀTIQUES 2 3

j Unitat 3. Aplicacions de la derivada

Activitats

1. Estudia les simetries i indica els punts de tall amb els eixos

de la funció:

Com que f( x) f(x), la funció és impa-

rella, per tant, és simètrica respecte de l’origen de coordenades.

f(x) 0 0 x3 0 x 0,

talla els eixos en l’origen.

2. Donada la funció dedueix-ne:

a) El domini i els tipus de discontinuïtats.

Df R { 1}, , presenta una discontinuïtat

asimptòtica en x 1.

b) Les simetries.

En ser f( x) , vol dir que f( x) f(x) i f( x) f(x),

per tant, la funció no és parella ni imparella, la gràfica no és simètrica ni respecte de l’eix d’ordenades ni respecte de l’origen.

c) Els punts de tall amb els eixos de coordenades.

f(x) 0 x 1 0 x 1, talla l’eix d’abscisses en el punt (1, 0).

f(0) 1, talla l’eix d’ordenades en el punt (0, 1).

3. Troba el recorregut de la funció de l’activitat anterior a par-tir del domini de la funció inversa.

f 1(x) Rf Df 1 R {1}.

4. a) Per què una funció no pot ser simètrica respecte de l’eix d’abscisses?

Hi hauria valors de x que tindrien dues imatges.

b) Per què la gràfica d’una funció pot tallar com a màxim per un punt l’eix OY?

Si tallés en més d’un punt l’eix OY, el valor x 0 tindria més d’una imatge.

5. Justifica de manera raonada per què la gràfica d’una funció no talla en cap punt una asímptota vertical.

Si la gràfica tallés una asímptota vertical, el valor d’x correspo-nent hauria de pertànyer al domini, però no hi pertany.

6. Demostra que les funcions polinòmiques no tenen asímpto-tes de cap tipus.

En ser p(x) , fa que no tingui asímptotes horitzontals,

com que m , tampoc en té d’obliqües i com que

Dp R, tampoc en té verticals.

7. Troba, si n’hi ha, els punts de tall de l’asímptota obliqua i la gràfica de la funció de l’exemple 3 apartat b.

No es tallen en cap punt, ja que l’equació 1

no té solució.

8. Troba les asímptotes de les funcions següents:

a)

Df R { 2, 2}

asímptotes verticals: x 2 i x 2, horitzontal: y 0, no en té d’obliqües.

b)

Df R, no té asímptotes verticals.

, tampoc no en té d’horitzontals.

asímptota obliqua: y 2x.

c)

Df R, no té asímptotes verticals.

asímptota horitzontal:

y 0 per a x .

Page 42: MATEMÀTIQUES - · PDF fileMatemàtiques 2 · Batxillerat · Solucionari No és permesa la reproducció total o parcial d’aquest llibre, ni el seu tractament informàtic, ni la

42 SOLUCIONARI DEL LLIBRE DE L’ALUMNE3

no té asímptota horitzontal per a

x .

no té asímptotes obliqües.

d)

Df R { 3}; la recta x 3 és una

asímptota vertical.

la recta y 2 és una asímptota horit-

zontal. No en té d’obliqües.

e)

Df R {5}; la recta x 5 és una asímp-

tota vertical.

no té cap asímptota horitzontal.

no té cap asímptota

obliqua.

f)

Df R { 1}; la recta x 1 és una

asímptota vertical.

la recta y 0 és una asímptota horit-

zontal. No en té d’obliqües.

9. Justifica la validesa o falsedat de les afirmacions següents:

a) Si f(x) és creixent en el punt x x0, aleshores f’(x0) 0.

Fals. Per exemple la funció f(x) x3 és creixent en tots els reals i en canvi f ’(0) 0.

b) Si f(x) és decreixent en el punt x x0, aleshores f’(x0) 0.

Veritat, ja que si la funció és decreixent la funció derivada no és positiva.

c) Si f(x) és creixent a l’esquerra del punt x a i decreixent a la dreta del mateix punt, aleshores x a és un màxim.

Fals. En el punt x a, pot haver-hi una discontinuïtat asimptòtica.

10. Estudia els intervals de creixement i decreixement de les funcions següents:

a)

R {1}, la funció és decrei-

xent en tot el seu domini.

b)

Df R, f’(x)

f’(x) 0 2 4x2 0 x 1

2

f’( 1) 0 és decreixent en – , –1

2, f’(0) 0 és

creixent en 1

2

1

2, i f’(1) 0 és decreixent en

1

2, ∞ .

c)

Df R {1}

f’(x) , f’(x) 0 ln x 1 0 ln x 1 x e

f’(0,5) 0 decreixent a (0, 1), f’(2) 0 decreixent a (1, e), i f’(3) 0 creixent a (e, ).

d)

Df R {0}, f’(x)

f’(x) 0 x 1

f’( 1) 0, ƒ’(1/2) 0 i f’(2) 0 decreix en ( , 0) i (0, 1) i creix en (1, )

11. Contesta raonadament les preguntes següents:

a) Per què igualem a zero la funció derivada, i resolem l’equació obtinguda, per trobar els punts estacionaris?

En els punts estacionaris, la recta tangent a la gràfica de la funció és horitzontal, per tant el seu pendent ha de ser zero, i com que per definició el pendent de la recta tangent en un punt és la derivada de la funció en aquest punt, tenim que la derivada ha de ser zero.

b) Per què una funció és creixent en un punt del seu domini, quan la derivada en aquest punt és positiva?

Si f’(x0) 0 mt 0 la recta t és creixent f(x) és creixent.

Page 43: MATEMÀTIQUES - · PDF fileMatemàtiques 2 · Batxillerat · Solucionari No és permesa la reproducció total o parcial d’aquest llibre, ni el seu tractament informàtic, ni la

43MATEMÀTIQUES 2 3

c) Per què en un punt d’inflexió de tangent horitzontal la funció és creixent o decreixent, i en els màxims i mínims relatius no?

En els màxims i en els mínims canvia el creixement de la fun-ció, aquesta passa de creixent a decreixent, o a l’inrevés, en canvi en els punts d’inflexió de tangent horitzontal no varia el creixement de la funció.

12. Esbrina els màxims, els mínims i els punts d’inflexió de tan-gent horitzontal de les funcions:

a) ƒ(x) 3x4 6x2

ƒ’(x) 12x3 12x

12x3 12x 0 x 0, x ± 1

ƒ’

En x 1 hi ha un mínim, en x 0 hi ha un màxim i en x 1 hi ha un altre mínim.

b) f(x) x4 2x3

ƒ’(x) 4x3 6x2

4x3 6x2 0 x 0, x

ƒ’

En el punt x la funció presenta un mínim i en el punt

x 0 hi ha un punt d’inflexió de tangent horitzontal.

c)

no presenta cap punt estacionari.

d) ƒ(x)

ƒ’(x)

f ’(x) 0 4x 0 x 0

ƒ’

En x 0 la funció presenta un màxim.

13. Contesta raonadament les preguntes següents:

a) Una funció f(x) és còncava en l’interval obert (a, b) i convexa en l’interval obert (b, c); vol dir això que en el punt d’abscissa x b hi ha un punt d’inflexió?

No, pot ser que en el punt x b hi hagi una discontinuïtat.

b) Si f’(x0) f’’(x0) 0, podem estar segurs que en x x0 hi ha un punt d’inflexió de tangent horitzontal?

No, ja que no n’hi ha prou que f’(x0) f”(x0) 0, si a més en

x x0 canvia la concavitat de la funció, aleshores sí que és

un punt d’inflexió de tangent horitzontal.

14. Determina els punts d’inflexió en general i els intervals de concavitat i convexitat de les funcions següents:

a) f(x) x3 2x2 4

La funció és convexa en ( , ), i és còncava en ( , ), per tant x és un punt d’inflexió.

b) f(x) x3(x 4)

En ( , 0) i (2, ) la funció és còncava, en (0, 2) és con-vexa, en x 0 i x 2 hi ha punts d’inflexió.

c)

En els intervals ( , 2 ) i (0, 2 ) la funció és convexa,

en ( 2 , 0) i (2 , ) és còncava, x 2 , x 0 i

x 2 són punts d’inflexió.

Page 44: MATEMÀTIQUES - · PDF fileMatemàtiques 2 · Batxillerat · Solucionari No és permesa la reproducció total o parcial d’aquest llibre, ni el seu tractament informàtic, ni la

44 SOLUCIONARI DEL LLIBRE DE L’ALUMNE3

d)

En ( , 2) i ( 2, 0) és convexa, en (0, 2) i (2, ) és còn-cava, té un punt d’inflexió en x 0.

15. Troba l’equació de la recta tangent al gràfic de les funcions següents en cada un dels seus punts d’inflexió:

a) f(x) x3 3x2 2x

f’(x) 3x2 6x 2 f’’(x) 6x 6

f’’(x) 0 6x 6 0 x 1

m f’(1) 3 6 2 1y0 f(1) 1 3 2 0 P(1, 0)

y (x 1) y x 1

b) f(x) x(x 1)3

f’(x) 4x3 9x2 6x 1

f’’(x) 12x2 18x 6

f’’(x) 0 12x2 18x 6 0

m1 f’(1) 0

y1 f(1) 0 P1(1, 0) y 0

c)

f’’(x) 0 3ex(3 ex) 0 3 ex 0 ex 3 x ln3

d) f(x) e x

no presenta cap punt d’inflexió.

16. Determina els intervals de concavitat i convexitat de cadas-cuna de les funcions de l’activitat anterior.

a) punt d’inflexió: x 1; Df R.

( , 1) f ’’(0) 6 0 f(x) és convexa.

(1, ) f ’’(2) 6 0 f(x) és còncava.

b) punts d’inflexió: x1 1, x2 ; Df R.

, f’’(0) 6 0 f(x) és còncava.

, 1 f’’(0, 6) 0 f(x) és convexa.

(1, ) f’’(2) 18 0 f(x) és còncava.

c) punt d’inflexió: x ln3; Df R.

( , ln3) f’’(0) 0 f(x) és còncava.

(ln3, ) f’’(2) 0 f(x) és convexa.

d) no hi ha punts d’inflexió, Df R.

( , ) f’’(0) 1 0 f(x) és còncava.

17. Dibuixa la gràfica d’una funció que tingui un punt d’inflexió de tangent horitzontal en el punt x 1, de manera que en aquest punt la funció passi de còncava a convexa. Justifica el creixement o decreixement de la funció en el punt x 1.

Resposta oberta. Per exemple:

En el punt d’abscissa x 1 la funció és decreixent.

Page 45: MATEMÀTIQUES - · PDF fileMatemàtiques 2 · Batxillerat · Solucionari No és permesa la reproducció total o parcial d’aquest llibre, ni el seu tractament informàtic, ni la

45MATEMÀTIQUES 2 3

18. Dibuixa les gràfiques de f’(x) i de f’’(x) a partir de la gràfica de f(x).

19. Si f’’(x0) 0, aleshores f(x) és còncava en x x0, però no recíprocament. Justifica-ho.

Si f(x) és còncava en x x0 f’(x) és creixent en x x0

f’’(x0) 0.

Recíprocament no és cert. Donada la funció f(x) x4, f’’(0) 0;

però en x 0, f(x) és còncava.

20. Donada la funció f(x) x3, dibuixa, mitjançant una taula de valors, les gràfiques de f(x), f’(x) i f’’(x). Explica de manera raonada què passa en el punt x 0.

En el punt x 0 hi ha un punt d’inflexió de tangent horitzontal, on la funció és creixent, i passa de convexa a còncava.

21. Considera les funcions dels apartats a) i b) de l’activitat 12: de cadascuna d’aquestes, dedueix-ne els màxims i els mínims aplicant el test de la segona derivada. Compara’n els resultats.

a) f’(x) 12x3 12x f’’(x) 36x2 12

f’(x)

f’’(x)

Page 46: MATEMÀTIQUES - · PDF fileMatemàtiques 2 · Batxillerat · Solucionari No és permesa la reproducció total o parcial d’aquest llibre, ni el seu tractament informàtic, ni la

46 SOLUCIONARI DEL LLIBRE DE L’ALUMNE3

f ’(x) 0 12x3 12x 0 x1 0, x2 1, x3 1.

f ’’(0) 12 0 en x 0 hi ha un màxim.

f ’’(1) 24 0 en x 1 hi ha un mínim.

f ’’( 1) 24 0 en x 1 hi ha un mínim.

b) f’(x) 4x3 6x2 f ’’(x) 12x2 12x

f ’(x) 0 4x3 6x2 0 x1 0, x2

f ’’(0) 0 en x 0 hi ha un punt d’inflexió de tangent horitzontal.

9 0 en x hi ha un mínim.

22. Dibuixa la gràfica d’una funció tal que:

a) Tingui un màxim i un mínim relatius, i no tingui cap punt d’inflexió.

b) Tingui un punt d’inflexió i no tingui cap màxim ni cap mínim relatius.

2 3. a) A partir de les gràfiques dels exemples 12 i 13, justifica les simetries, el recorregut i els màxims i mínims abso-luts de cada una de les funcions.

Exemple 12: La gràfica és simètrica respecte de l’origen. Rf R i no té ni màxim ni mínim absoluts.

Exemple 13: La gràfica de la funció no és simètrica ni respec-te de l’eix d’ordenades ni respecte de l’origen. Rf (0, ), el punt ( 1, 0) és un mínim absolut i no hi ha cap màxim absolut.

b) Justifica els punts d’inflexió de cadascuna de les gràfi-ques dels exemples anteriors.

Exemple 10: Hi ha dos punts d’inflexió, un en x a , 0 ,

ja que la gràfica en aquest punt passa de convexa a còncava,

i un altre en x b 0, on la gràfica passa de còncava a convexa.

Exemple 11: Hi ha un punt d’inflexió en un punt x 2, on la gràfica passa de convexa a còncava.

Exemple 13: En un punt x c ( 1, 0) la gràfica de la funció passa de còncava a convexa, per tant x c és un punt d’in-flexió, i en x 1 n’hi ha un altre, donat que la gràfica passa de convexa a còncava.

24. Dibuixa la gràfica de les funcions següents.

A partir de les gràfiques dibuixades, informa sobre la conca-vitat, els punts d’inflexió, les simetries, el recorregut i els màxims i mínims absoluts de cada funció.

a) f(x) 6x2 2x3

Df R. En ser una funció polinòmica no té cap tipus

d’asímptota.

f(x) 0 6x2 2x3 0 2x2 (3 x) 0 x 0, x 3; talla els eixos en (0, 0) i (3, 0).

f’(x) 12x 6x2

f’(x) 0 12x 6x2 0 6x (2 x) 0 x 0, x 2.

f’’(x) 12 12x

f’’(0) 12 0 a l’origen hi ha un mínim,

f’’(2) 12 0 en el punt (2, 8) hi ha un màxim.

Page 47: MATEMÀTIQUES - · PDF fileMatemàtiques 2 · Batxillerat · Solucionari No és permesa la reproducció total o parcial d’aquest llibre, ni el seu tractament informàtic, ni la

47MATEMÀTIQUES 2 3

No presenta cap tipus de simetria. Rf R, no hi ha cap punt de la gràfica que sigui un màxim o un mínim absoluts.En l’interval ( , 1) és còncava, en (1, ) és convexa. El punt (1, 4) és un punt d’inflexió, no hi ha cap simetria. Rf R, no hi ha cap punt que sigui un màxim o un mínim absoluts.

b) f(x)

Df R {0}, x 0 és una asímptota

vertical.

0 la recta y 0 és una asímptota horitzontal.

No en té d’obliqües.

f(x) 0 x 1 0 x 1, talla l’eix d’abscisses en el punt (1, 0).

x 0 Df no talla l’eix d’ordenades.

f’(x) , f’(x) 0 x 2 0 x 2

En el punt 2, la funció presenta un màxim.

No presenta simetries. Rf , ], el punt 2, és un

màxim absolut, no té mínim absolut.

És convexa en els intervals ( , 0) i (0, 3), és còncava en l’interval (3, ), en el punt d’abscissa x 3 hi ha un punt

d’inflexió, no hi ha simetries.

Rf , ], el punt 2, és un màxim absolut, no té

mínim absolut.

c)

Df R no té asímptotes verticals.

la recta y és una asímptota ho-

ritzontal.

No té asímptotes obliqües.

f(x) 0 x2 x 0 x(x 1) 0 x 0, x 1; passa pels punts (0, 0), (1, 0).

f’(x)

f’(x) 0 8x2 2x 1 0 x , x

En , hi ha un màxim, en , hi ha un mínim.

La gràfica no és simètrica ni respecte de l’eix d’ordenades, ni respecte de l’origen.

Rf [ , ], el màxim i el mínim relatius són també

absoluts.

Considerem els punts d’abscissa a, b i c tals que:

a , b 0 i c 1, on la funció canvia la conca-

vitat. És còncava en ( , a) i (b, c) i és convexa en (a, b) i (c, ), els punts a, b i c són punts d’inflexió. La gràfica no és simètrica ni respecte de l’eix d’ordenades ni respecte de l’origen.

Page 48: MATEMÀTIQUES - · PDF fileMatemàtiques 2 · Batxillerat · Solucionari No és permesa la reproducció total o parcial d’aquest llibre, ni el seu tractament informàtic, ni la

48 SOLUCIONARI DEL LLIBRE DE L’ALUMNE3

Rf [ , ], el punt , és un màxim absolut i

el punt , és un mínim absolut.

d)

Df R {1}

la recta x 1 és una asímptota vertical.

no hi ha asímptotes horitzontals.

m 1

n 4

La recta y x 4 és una asímptota obliqua.

f(x) 0 x2 3x 0 x 3, x 0; talla els eixos en els punts ( 3, 0) i (0, 0).

f’(x) , f’(x) 0 x2 2x 3 0

Hi ha un màxim en ( 1, 1) i un mínim en (3, 9).

No és simètrica ni respecte de l’eix d’ordenades, ni respecte de l’origen.

Rf R (1, 9), no té ni màxims ni mínims absoluts.

En l’interval ( , 1) és convexa i és còncava en (1, ), no té cap punt d’inflexió.

El recorregut és Rf R (1, 9), no és simètrica ni respecte de l’eix d’ordenades ni respecte de l’origen. No hi ha ni màxim ni mínim absoluts.

25. Dibuixa la gràfica de les funcions següents:

a) f(x) x4 4x

Df R. És una funció polinòmica, per tant no té cap tipus

d’asímptota.

x4 4x 0 x 0, x ; talla els eixos en l’origen i en el punt ( , 0).

f’(x) 4x3 4 f’(x) 0 4x3 4 0 x 1

f“(x) 12x2 ƒ“(1) 12 0 en el punt (1, 3) hi ha un mínim.

Page 49: MATEMÀTIQUES - · PDF fileMatemàtiques 2 · Batxillerat · Solucionari No és permesa la reproducció total o parcial d’aquest llibre, ni el seu tractament informàtic, ni la

49MATEMÀTIQUES 2 3

b) f(x)

Df R {0}

l’eix d’ordenades és una asímptota vertical.

la recta y 1 és una asímptota horitzontal.

No en té d’oblíqües.

f(x) 0 x 1, talla l’eix d’abscisses en el punt (1, 0).

x 0 Df no talla l’eix d’ordenades.

f’(x) 1

2x, no té cap punt estacionari. Com que f ’(x) 0,

x Df; la funció és creixent en tot el seu domini.

26. Observa la gràfica de la funció i dóna tota la informació pos-sible de la funció.

La recta y x és una asímptota obliqua, les rectes x a i x a són asímptotes verticals.

En el punt x b hi ha un màxim relatiu, en x 0 hi ha un punt d’inflexió de tangent horitzontal i en x b hi ha un mínim relatiu.

Talla els eixos de coordenades en l’origen, el domini i el recorre-gut són Df R { a, a} i Rf R respectivament.

És una funció imparella, ja que la gràfica és simètrica respecte de l’origen, no té ni màxim ni mínim absoluts.

És creixent en ( , b), ( a, a) i (b, ), i és decreixent en ( b, a) i (a, b), és convexa en ( , a) i ( a, 0) i és còncava en (0, b) i (b, ).

27. En l’exemple 14, troba el màxim a partir de la variable y.

x 30 (3/2)y f(y) ( 3/2)y2 20y 17 000

f’(y) 3y 20

f’(y) 0 y 20/3 [0, 20)

Per a y 0 f(0) 17 000 cm2.

Per a y 20, f(20) 16 000 cm2.

La solució és y 0 x 30 cm.

28. Resol l’exemple 15 a partir de la variable r.

A r2 c2

2 r 4c 1 c

polinòmica de 2n grau on

a 0, per tant tindrà un mínim.

Page 50: MATEMÀTIQUES - · PDF fileMatemàtiques 2 · Batxillerat · Solucionari No és permesa la reproducció total o parcial d’aquest llibre, ni el seu tractament informàtic, ni la

50 SOLUCIONARI DEL LLIBRE DE L’ALUMNE3

Per construir la circumferència es necessita:

0,44 m 44 cm

La resta de filferro 1 0,44 0,56 m 56 cm serà per construir el quadrat.

29. Descompon el nombre 36 en dos sumands, tals que el seu producte sigui màxim.

f(x) x(36 x) 36x x2, funció que té un màxim.

f’(x) 36 2x f’(x) 0 x 18.

30. De tots els triangles rectangles amb hipotenusa igual a 9 cm, calcula el d’àrea més gran.

f’(x) 0 81 2x2 0 x2 x cm

El valor x cm maximitza l’àrea del triangle.

ƒ cm2

31. Demostra que de tots els rectangles de perímetre 4p, el que té àrea màxima és el quadrat de costat p.

L’àrea del quadrat de costat p és Sc p2.

L’àrea d’un rectangle de perímetre 4p és

Sr (2p q) q 2pq q2 com que (q p)2 0

q2 2pq p2 0 p2 2pq q2 Sc Sr.

32. El perímetre d’un rectangle és de 4 m. Els seus costats se substitueixen per semicircumferències exteriors, tal com in-dica el dibuix. Troba les dimensions del rectangle que facin que la superfície de la nova figura sigui mínima. Calcula aquesta superfície mínima.

2x: costat gran del rectangle

2y: costat petit del rectangle

4x 4y 4 x y 1 y 1 x

S 2x · 2y x2 y2 4xy (x2 y2) 4x(1 x) [x2 (1 x)2] 4x 4x2 (x2 1 2x x2) 4x 4x2 (2x2 2x 1) (2 4)x2 (4 2 )x

f(x) (2 4)x2 (4 2 )x

és una funció polinòmica de 2n grau, on a 0 tindrà un mínim.

f’(x) 2(2 4)x 4 2

f’(x) 0 2(2 4)x 4 2 0

2(2 4) x 2 4 x m

y 1 x 1 m

S 4xy (x2 y2) 4 · ·

1 m2

Activitats finals

1. Dibuixa la gràfica de la funció f(x) |x2 x|. Estudia la continuïtat i la derivabilitat en els punts x 0 i x 1.

Page 51: MATEMÀTIQUES - · PDF fileMatemàtiques 2 · Batxillerat · Solucionari No és permesa la reproducció total o parcial d’aquest llibre, ni el seu tractament informàtic, ni la

51MATEMÀTIQUES 2 3

En els punts x 0 i x 1 la funció és contínua, però no deri-vable.

2. Dibuixa, en cada cas, la gràfica d’una funció que tingui una discontinuïtat asimptòtica en un punt, i que en aquest punt:

a) En canviï el creixement, però no la concavitat.

Respostes obertes, per exemple:

b) En canviï la concavitat, però no el creixement.

Respostes obertes, per exemple:

3. Dibuixa la gràfica d’una funció que sigui contínua en tots els reals i que tingui un punt x x0 on canviï el creixement i la concavitat de la funció. En aquest punt és derivable la funció? Raona la resposta.

Resposta oberta, per exemple:

En el punt x x0 la funció no és derivable, ja que en aquest punt no existeix una única recta tangent a la gràfica.

4. Raona la certesa o la falsedat de les afirmacions següents:

a) Dues funcions amb idèntica funció derivada són necessà-riament idèntiques.

Fals, per exemple f(x) x2 i g(x) x2 2 tenen la mateixa funció derivada i en canvi són diferents.

b) La funció f(x) 2x cos x és sempre creixent.

Veritat, ja que f’(x) 2 sin x és sempre positiva; perquè sin x 1 2 2 sin x 0

c) La funció f(x) 4x sin x no té cap punt estacionari.

Veritat, ja que f’(x) 4 cos x no s’anul·la per a cap valor de x, perquè cos R.

5. Donada la funció , indica’n el domini, els límits

per a x 0 i x , i les asímptotes. Raona detalladament

tot el que fas.

Df R {1} la recta x 1 és una asímptota vertical.

6. Sigui f(x) una funció derivable en tots els reals.

a) Si sabem que f’(a) 0, pots afirmar que f(x) té necessà-riament un màxim o un mínim relatiu en el punt x a?

No, ja que també podria ser un punt d’inflexió de tangent horitzontal.

Page 52: MATEMÀTIQUES - · PDF fileMatemàtiques 2 · Batxillerat · Solucionari No és permesa la reproducció total o parcial d’aquest llibre, ni el seu tractament informàtic, ni la

52 SOLUCIONARI DEL LLIBRE DE L’ALUMNE3

b) Si sabem que la derivada de f(x) és negativa en tots els punts x a i positiva en tots els punts x a, pots afir-mar que f(x) té necessàriament un mínim relatiu en el punt x a?

Sí, ja que en ser derivable també és contínua, i això fa que necessàriament en x a hi hagi un mínim relatiu.

7. Calcula el valor de k per tal que:

a) La funció f(x) x e–kx tingui un màxim o un mínim relatiu en el punt x 1.

f’(x) e–kx(1 kx)f’(1) 0 e–k(1 k) 0 1 k 0 k 1

b) La funció tingui límit 2 quan x .

c) La funció f(x) ln (kx2 1) sigui creixent en x 1.

k 1 o k 0

8. Determina els coeficients a i b de la funció següent f(x) ax2 bx 2, sabent que la recta tangent a la gràfica en el punt x 1 és la recta y 2x.

f’(x) 2ax b

f’(1) 2 2a b 2

f(1) 2 a b 2 2 a b 4

a 2, b 6 f(x) 2x2 6x 2

9. Determina quins són els coeficients a, b i c de la funció f(x) ax3 bx2 cx per tal que aquesta funció tingui un màxim relatiu en x 0, un mínim relatiu en x 1 i compleixi la condició f(1) 1/2.

f’(x) 3ax2 2bx c

f’(0) 0 c 0

f’(1) 0 3a 2b c 0 3a 2b 0

f(1) 12

a b c 12

a b 12

D’on s’obté que a 1, b 32

f(x) x3 32

x2

10. Determina els coeficients a i b de la funció següent f(x) x3 ax2 bx, sabent que canvia de còncava a convexa en el punt x 1 i que la recta tangent a la gràfica de la fun-ció en aquest mateix punt és horitzontal.

f’(x) 3x2 2ax b f’’(x) 6x 2a

f’’(1) 0 6 2a 0 a 3

f’(1) 0 3 2a b 0 3 6 b 0 b 3

f(x) x3 3x2 3x

11. Considerant la funció f(x) x3 3x2, troba l’equació de la recta tangent a la gràfica en el punt d’inflexió.

f’(x) 3x2 6x f’’(x) 6x 6

f’’(x) 0 6x 6 0 x 1

y 3x 1

12. Tenim una funció f(x) de la qual sabem que la seva derivada és positiva en tot x 2 i s’anul·la en x 2, tal com indica la figura. Què pots dir de la funció f(x) en el punt x 2? Tindrà en aquest punt un màxim, un mínim, un punt d’inflexió? Raona detalladament la resposta.

En el punt x 2 hi ha un punt d’inflexió de tangent horitzontal; ja que f’(x) 0, i x 2, per tant no canvia el creixement en el punt x 2, tot i que f’(2) 0.

13. Sabem que la gràfica de la derivada f’(x) d’una funció f(x) és el que mostra el dibuix, s’anul·la en x 1, x 2 i x 3. Digues quins valors de x corresponen a mínims relatius de f(x). Explica el perquè de la teva resposta.

En el punt x 1 la funció f’’(x) s’anul·la i passa de positiva a negativa, per tant la funció f(x) passa de creixent a decreixent, aleshores en x 1 hi ha un màxim.

En el punt x 2 també s’anul·la f’’(x), però no canvia de signe, continua essent negativa, per tant la funció f(x) és decreixent en aquest punt, aleshores en x 2 hi ha un punt d’inflexió de tangent horitzontal.

Page 53: MATEMÀTIQUES - · PDF fileMatemàtiques 2 · Batxillerat · Solucionari No és permesa la reproducció total o parcial d’aquest llibre, ni el seu tractament informàtic, ni la

53MATEMÀTIQUES 2 3

En el punt x 3 tenim que f’(3) 0, i la derivada passa de ne-gativa a positiva, la funció f(x) passa de decreixent a creixent, aleshores en x 3 hi ha un mínim.

14. Tenim una funció derivable f(x) definida per a x 0, de la qual sabem que la seva gràfica és la que s’indica, l’eix d’or-denades és asímptota vertical, la recta y x és asímptota obliqua i té un mínim en el punt x 1. Fes un esquema sen-zill de la gràfica de la funció f’(x) tot explicant raonadament la resposta.

15. Dibuixa la gràfica de les funcions següents:

a) f(x)

Df R {2}, les rectes x 2 i y 0 són una asímptota ver-tical i una asímptota horitzontal, respectivament. Talla els eixos en els punts (3, 0) i (0, 3), i té un màxim en el punt

(4, 1), ja que és decreixent en ( , 2) i (4, ), i creixent

en (2, 4).

b) f(x)

Df R { 1}, la recta x 1 és una asímptota vertical, no en té d’horitzontals i la recta y x 2 n’és una d’obliqua. Passa per l’origen de coordenades, els punts estacionaris són x 3 i x 0, i com que és creixent en els intervals ( , 3) i ( 1, ), i decreixent en ( 3, 1), fa que en ( 3, 27/4) hi hagi un mínim i en l’origen un punt d’inflexió de tangent horitzontal.

Page 54: MATEMÀTIQUES - · PDF fileMatemàtiques 2 · Batxillerat · Solucionari No és permesa la reproducció total o parcial d’aquest llibre, ni el seu tractament informàtic, ni la

54 SOLUCIONARI DEL LLIBRE DE L’ALUMNE3

c) f(x)

En ser una funció polinòmica, no té cap tipus d’asímptotes. Talla els eixos en els punts (0, 0) i (3, 0). Els valors que anul·len la derivada són x 0 i x 2, en el punt (0, 0) la funció presenta un mínim i en (2, 4/3) un màxim, ja que és decreixent en ( , 0) i (2, ) i és creixent en (0, 2).

d) f(x) x3 3x

Df R, no té asímptotes de cap tipus, talla els eixos en

( , 0), (0, 0) i ( , 0). Té un màxim relatiu en ( 1, 2) i un mínim relatiu en el punt (1, 2), ja que és creixent en ( , 1) i (1, ), i decreixent en ( 1, 1).

e) f(x)

Df R {2}, la recta x 2 és una asímptota vertical, no en té d’horitzontals i la recta y x 4 és una asímpto-ta obliqua. Passa pels punts ( 2, 0) i (0, 0), els valors

x 2 ± 2 anul·len la primera derivada, és creixent en ( , 2 2 ) i (2 2 , ), i decreixent en els

intervals (2 2 , 2) i (2, 2 2 ), per tant en el punt

x 2 2 la funció presenta un màxim i en x 2 2 un mínim.

f) f(x)

El domini és Df R { 2, 2}, les rectes x 2 i x 2 són asímptotes verticals, la recta y 0 és una asímptota horit-zontal. Té un màxim relatiu en el punt (0, 2) i no té cap mínim relatiu, ja que és creixent en els intervals ( , 2) i ( 2, 0), i és decreixent en (0, 2) i (2, ).

Page 55: MATEMÀTIQUES - · PDF fileMatemàtiques 2 · Batxillerat · Solucionari No és permesa la reproducció total o parcial d’aquest llibre, ni el seu tractament informàtic, ni la

55MATEMÀTIQUES 2 3

g) f(x) x2

Df R {0}, l’eix d’ordenades és una asímptota vertical, no en té d’horitzontals ni d’obliqües. Talla l’eix d’abscisses en

el punt ( , 0), en x 1 hi ha un punt estacionari, que resulta ser un mínim ja que és decreixent en ( , 0) i (0, 1) i creixent en (1, ).

h) f(x)

Df R { 1,1}, les rectes x 1 i x 1 són asímptotes ver-ticals, la recta y x és una asímptota obliqua. Té un màxim

i un mínim relatius en x i en x respectivament, el punt (0, 0) és un punt d’inflexió de tangent horitzontal,

ja que és creixent en ( , ) i ( , ) i decreixent en ( , 1), ( 1, 1) i (1, ).

i) f(x)

Df R { 4}, la recta x 4 és una asímptota vertical, no en té d’obliqües. Té una asímptota horitzontal quan x : y 0, f(x) 0.

Talla els eixos en l’origen, té un punt estacionari en x 2, com que és creixent en tot el seu domini, fa que en el punt ( 2, e 2) hi hagi un punt d’inflexió de tangent horitzontal.

j) f(x)

Df R {0}, l’eix OY és una asímptota vertical, no n’hi ha de cap més tipus. No talla els eixos en cap punt, f’(x) s’anul·la en x 1, en el punt ( 1, 4) hi ha un màxim i en (1, 4) un mínim, ja que és creixent en ( , 1) i (1, ), i decreixent en ( 1, 0) i (0, 1).

Page 56: MATEMÀTIQUES - · PDF fileMatemàtiques 2 · Batxillerat · Solucionari No és permesa la reproducció total o parcial d’aquest llibre, ni el seu tractament informàtic, ni la

56 SOLUCIONARI DEL LLIBRE DE L’ALUMNE3

16. A partir de la gràfica, dóna tota la informació possible de les funcions dels apartats a, d, f i h de l’activitat anterior.

a) Df R {2}, les rectes x 2 i y 0 són una asímptota ver-

tical i una asímptota horitzontal respectivament. Talla els eixos en els punts (3, 0) i (0, 3), té un màxim absolut en el punt (4, 1) i presenta un punt d’inflexió en x 5. És de-creixent en ( , 2) i (4, ), creixent en (2, 4), és convexa en ( , 2) i (2, 5) i còncava en (5, ). No és simètrica ni respecte de l’eix d’ordenades ni respecte de l’origen, el recorregut és Rf ( , 1).

d) Df R, talla l’eix d’abscisses en ( , 0), (0, 0) i ( , 0),

té un màxim relatiu en ( 1, 2) i un mínim relatiu en el punt (1, 2), el punt (0, 0) és un punt d’inflexió. És simè-trica respecte de l’origen, és creixent en ( , 1) i (1, ), decreixent en ( 1, 1), és convexa en ( , 0) i còncava en (0, ). Rf R no hi ha ni màxim ni mínim absoluts.

f) El domini és Df R { 2, 2}, les rectes x 2 i x 2 són

asímptotes verticals, la recta y 0 és una asímptota horit-zontal. Té un màxim relatiu en el punt (0, 2), no té cap mínim relatiu ni cap punt d’inflexió. És creixent en els inter-vals ( , 2) i ( 2, 0), i és decreixent en (0, 2) i (2, ), és còncava en ( , 2) i (2, ) i convexa en l’interval ( 2, 2). És simètrica respecte de l’eix d’ordenades, el recorregut és Rf R ( 2, 0], no hi ha cap màxim ni cap mínim abso-luts.

h) Df R { 1, 1}, les rectes x 1 i x 1 són asímptotes ver-

ticals, la recta y x és una asímptota obliqua. Té un màxim

i un mínim relatius en x i en x respectivament, el punt (0, 0) és un punt d’inflexió de tangent horitzontal, és

creixent en ( , ) i ( , ) i decreixent en ( , 1),

( 1, 1) i (1, ), és convexa en ( , 1) i (0, 1) i còncava en ( 1, 0) i (1, ). És simètrica respecte de l’origen, el re-corregut és Rf R, no té cap màxim ni cap mínim absoluts.

17. Calcula els intervals de creixement i de decreixement, els màxims i els mínims de la funció següent:

Després fes un esquema senzill de la gràfica.

És creixent en els intervals ( , 2 000) i (0, ), i decreixent en l’interval ( 2 000, 0). Els punts x 2 000 i x 0 són res-pectivament un màxim i un mínim.

18. Considera la funció següent: per a x 0.

Troba els valors de x tals que f’(x) 0. Després fes un es-quema senzill de la gràfica de f(x), i explica-ho.

f’(x) 0 1 ln 1 000x 0

ln 1 000x 1 1 000x e x e

1000

f ( e1000 )

1000e

En x e

1000 la funció passa de creixent a decreixent, per tant

hi ha un màxim en el punt ( e1000 ,

1000e )

19. Fes un esquema senzill de la gràfica de la funció f(x) ex e–x que posi en evidència els límits quan x i els possibles màxims i mínims. Explica raonadament tot el que fas.

(ex e–x)

f’(x) ex e–x, f’(x) 0 ex e–x 0

e2x 1 2x 0 x 0 f(0) 2

En el punt (0, 2) hi ha un mínim ja que la funció és decreixent en l’interval ( , 0) i creixent en (0, ).

20. Troba dos nombres positius que sumin 30 tals que la suma dels seus quadrats sigui mínima.

La funció que cal optimitzar és:

Page 57: MATEMÀTIQUES - · PDF fileMatemàtiques 2 · Batxillerat · Solucionari No és permesa la reproducció total o parcial d’aquest llibre, ni el seu tractament informàtic, ni la

57MATEMÀTIQUES 2 3

f(x) x2 (30 x)2 2x2 60x 900f’(x) 4x 60, f’(x) 0 x 15 és la solució, ja que minimitza la funció f(x).

21. Hem de construir un parterre en forma de sector circular amb perímetre de 20 m. Calcula el radi del sector per tal d’obtenir-lo d’àrea màxima.

2r x 20 m x 20 2r

f(r) 10r r2 tindrà un màximf’(r) 10 2r f’(r) 0 10 2r 0 r 5 cm

22. Quin perímetre mínim pot tenir un sector circular de 25 m2 d’àrea?

.

El perímetre mínin és de 20 m.

23. La suma de totes les arestes d’un prisma recte de base qua-drada és 36 cm. Calcula les dimensions del prisma perquè tingui volum màxim.

Considerem x el costat del quadrat de la base i y l’altura del prisma, tenim que:

8x 4y 36 y 9 2xV x2y f(x) x2(9 2x) 9x2 2x3 f’(x) 18x 6x2 f’(x) 0 x 0, x 3

Per a x 0 dóna volum mínim, i per a x 3 el dóna màxim, per tant les dimensions del prisma són x y 3 cm.

24. Es vol construir un recipient cilíndric, amb tapa, de volum 100 m3. Quines han de ser les seves dimensions perquè s’utilitzi la mínima quantitat de material?

r: radi de la base h: altura del cilindre

25. Entre tots els cilindres rectes de base circular i d’àrea total 6 m2, troba les dimensions del que té volum màxim i calcu-la aquest volum.

r: radi de la baseh: altura del cilindre

V r2h r2 r(3 r2) 3 r r3

f(r) 3 r r3 ƒ’(r) 3 3 r2

f’(r) 3 3 r2 0 r2 1 r 1 m

f’’(r) 6 r, f’’(1) 6 0 és un màxim.

V r2h 2 m3

El volum màxim és de 2 m3.

26. Troba les dimensions del triangle isòsceles d’àrea màxima, inscrit en una circumferència de radi 10 dm. Calcula aquesta àrea màxima.

Page 58: MATEMÀTIQUES - · PDF fileMatemàtiques 2 · Batxillerat · Solucionari No és permesa la reproducció total o parcial d’aquest llibre, ni el seu tractament informàtic, ni la

58 SOLUCIONARI DEL LLIBRE DE L’ALUMNE3

Emprant les variables x, y del dibuix, tenim:

base: b 2x 10 dm

altura: h 10 y 10

10 10 15 dm

S bh 10 · 15 75 dm2

L’àrea màxima és de 75 dm2.

27. Quina és l’àrea més gran que pot tenir un rectangle de cos-tats paral·lels als eixos de coordenades inscrit a l’el·lipse d’equació 4x2 y2 1?

u2

L’àrea màxima és de 1 u2.

28. Una persona transporta un vidre molt prim per un carrer en forma de L, de manera que una de les parts del carrer té 4 m d’amplada i l’altra, 3 m. Quina serà la longitud màxima que podrà tenir el vidre per poder passar-hi?

Emprant les variables x, del dibuix, tenim

l l1 l2 és la longitud del vidre

47,74º és un mínim per a l’amplada del carrer, per tant serà un màxim per a la longitud del vidre.

4,46 5,40 9,86 m 986 cm

La longitud màxima que pot tenir el vidre són 986 cm.

Page 59: MATEMÀTIQUES - · PDF fileMatemàtiques 2 · Batxillerat · Solucionari No és permesa la reproducció total o parcial d’aquest llibre, ni el seu tractament informàtic, ni la

59MATEMÀTIQUES 2 3

29. Considera una piràmide recta que té per base un hexàgon regular d’1 cm de costat. L’altura d’aquesta piràmide mesura també 1 cm. Digues a quina distància de la base s’ha de situar un punt P sobre l’altura per tal que la suma de les distàncies de P als vèrtexs de la piràmide sigui mínima.

és un mínim.

El punt P s’ha de situar a cm del centre de la base.

30. Un triangle isòsceles de perímetre 10 cm gira al voltant de la seva altura, i engendra un con. Calcula la base del triangle perquè el con generat tingui volum màxim, i determina’l.

2 2 1010 2

2x y x

ycm

V y h y x y

yy

y

13 3

310 2

2

2 2 2 2

2

2

2

3325 104 5y y

V yy y

y y

y y( ) 6

100 50

25 10 6100 50

25 1

3 4

4 5

2

00

0 2

1 0

0

y

V y y

V V y

V V

( )

( ) ( )

(

creix

(2,1) yyy

) decreixés un màxim.2

La base del cilindre és b = 2y = 4 cm.

V(2) = 9,37 cm3

31. Considera un dipòsit constituït per una semiesfera de radi r a la qual s’ha afegit un cilindre circular del mateix radi r i d’altura h, tal com s’indica en la figura. Calcula r i h de manera que l’àrea total de les parets i de la tapa sigui de 5 m2 i tingui volum màxim.

és un màxim.

32. Troba els punts de la gràfica de la funció y2 4x, tals que la distància al punt (4, 0) sigui mínima. Calcula aquesta distància.

Page 60: MATEMÀTIQUES - · PDF fileMatemàtiques 2 · Batxillerat · Solucionari No és permesa la reproducció total o parcial d’aquest llibre, ni el seu tractament informàtic, ni la

60 SOLUCIONARI DEL LLIBRE DE L’ALUMNE3

En x 2, f’(x) passa de negativa a positiva, per tant la funció f(x) passa de decreixent a creixent, en el punt x 2 la funció presenta un mínim.

Els punts solució del problema són P1(2, 2 ) i P2(2, 2 ).

d 2 4 2 16 2 32 u

33. Troba el punt de la paràbola y 2x2 que està més a prop del punt (9, 0).

y 2x2 P(x, 2x2) és un punt de la paràbola.

PQ

PQ

34. Calcula els punts de la gràfica de la funció següent

en què la tangent té pendent màxim.

g(x) f’(x) g’(x) f“(x)

g’(x) 0 6x2 2 0 x ± 1

3. En x

1

3 hi ha un

màxim de g(x), y f1

3

34

, el punt de la gràfica que dóna

la solució al problema és P1

3

34

, .

35. Entre totes les rectes que són tangents a la gràfica de la

funció f(x) tg x, on x està situada entre i , escriu

l’equació de la que té pendent mínim.

36. Considera un triangle rectangle, de vèrtexs (0, 0), (x, 0) i (x, y), amb x 0 i y 0, i amb el vèrtex (x, y) sobre l’el·lipse d’equació x2 2y2 2, tal com s’indica en la figura. Troba el punt (x, y) que fa que el triangle rectangle tingui àrea màxima.

Page 61: MATEMÀTIQUES - · PDF fileMatemàtiques 2 · Batxillerat · Solucionari No és permesa la reproducció total o parcial d’aquest llibre, ni el seu tractament informàtic, ni la

61MATEMÀTIQUES 2 3

37. La resistència de flexió d’una biga de secció rectangular és directament proporcional a la base i directament proporcio-nal, també, al quadrat de l’altura d’aquesta secció. Calcula les dimensions que ha de tenir la secció rectangular d’una biga fabricada a partir del tronc cilíndric d’un arbre que fa un metre de diàmetre per tal que tingui una resistència de flexió màxima.

x: basey: altura

x2 y2 1 y2 1 x2

R kxy2 kx(1 x2) kx kx3

f(x) kx kx3 f’(x) k 3kx2

és un màxim.

,

La base ha de ser de i l’altura de .

38. La trajectòria d’un projectil disparat per un canó d’ar-tilleria situat a l’origen de coordenades és la paràbola

f(x) k(1 tg2 )x2 (tg )x

on k és una constant positiva que depèn de les caracterís-tiques del canó, i és l’angle que formen l’eix de les x positives i el canó. L’angle se suposa que comprèn entre 0 i 90 graus, tal com indica el dibuix. Calcula l’angle per al qual la paràbola anterior talla l’eix de les x positives al més lluny possible de l’origen.

f(x) 0 k(1 tg2 )x2 x · tg 0

x[ k(1 tg2 )x tg ] 0

x 0 O(0, 0).

L’altre punt, el que interessa és:

k(1 tg2 )x tg 0 x és el que cal

optimitzar.

g’( ) 0 1 tg2 0 tg 1 45º

Avaluació

1. Considera la funció f(x) 4x x2

a) Calcula l’equació de les rectes tangents a la gràfica de f(x) en els punts d’abscisses x 0 i x 4.

La derivada de ƒ (x) és ƒ’(x) 4 2x. Els pendents de les

rectes tangents demanades són f’(0) 4 i f’(4) −4. La recta

tangent en el punt (0,0) té per equació y 4x i la recta tan-

gent en el punt (4,0) és y 4x 16.

b) Fes un gràfic dels elements del problema.

Page 62: MATEMÀTIQUES - · PDF fileMatemàtiques 2 · Batxillerat · Solucionari No és permesa la reproducció total o parcial d’aquest llibre, ni el seu tractament informàtic, ni la

62 SOLUCIONARI DEL LLIBRE DE L’ALUMNE3

2. (Curs 2004-05) Considera la funció ƒ(x) 3 x2 i un punt de la seva gràfica, M, situat en el primer quadrant (x ≥ 0, y ≥ 0). Si pel punt M tracem paral leles als eixos de coorde-nades, la seva intersecció amb OX i OY determina dos punts, A i B, respectivament.

a) Fes un gràfic dels elements del problema.

b) Troba les coordenades del punt M que fan que el rectangle OAMB tingui l’àrea màxima.

Sigui M(a, 3 a2), amb a ≥ 0. L’àrea del rectangle OAMB és S(a) a∙(3 a2) 3a a3. Per trobar-ne el mínim, resolem S’(a) 0, és a dir, 3 3a2 0. Tenim que a ±1. Ens quedem només amb la solució positiva, a 1. Estem en un màxim perquè S’’(1) 6 0. Així, el punt M demanat és (1, 2).

3. Donada la funció , calcula els valors de

a, b, c i d sabent que f(x) té una asímptota vertical en el punt d’abscissa x 1, que la recta y 3x 2 n’és asímptota obliqua, i que f(x) té un màxim relatiu en el punt d’abscissa x 0.

a 3, b 1, c 1 i d 1

x 1: asímptota vertical

x d 0 1 d 0 d 1

y 3x + 2: asímptota oblíqua

max bx c

x xa

mx bx

x

x

lim( )

lim

0

2

0

2

13 3

3 ccx

xb x c

x

b b

x13

31

2

3 2

0lim

( )

1

x 0: màxim relatiu f ’(0) 0

f xx x c

x( )

31

2

f xx x c

x( )

( )

3 6 1

1

2

2

f c c( )0 0 1 0 1

4. Per tal d’il luminar una taula circular d’un metre de radi, volem penjar del sostre de l’habitació un llum situat en la vertical del centre de la taula i que enfoqui cap avall. Digues a quina altura hem de situar aquest llum respecte a la taula per tal que els punts de la seva vora tinguin una il luminació màxima. Si designem com a L el llum, que se suposa puntual, i com a P un punt qualsevol de la taula, tal com indica el dibuix, la il luminació I del punt P és donada per:

I K

on K és una constant que depèn de les característiques del llum, d és la distància entre P i L, i a és l’angle entre PL i la vertical.

h m

L’altura ha de ser de m.

Page 63: MATEMÀTIQUES - · PDF fileMatemàtiques 2 · Batxillerat · Solucionari No és permesa la reproducció total o parcial d’aquest llibre, ni el seu tractament informàtic, ni la

63MATEMÀTIQUES 2 4

j Unitat 4. Primitives

Activitats

1. Escriu l’expressió general de les primitives de cadascuna de les funcions següents:

a) f(x) 3x2

F(x) x3 C

b) g(x) sin x

G(x) cos x C

c) h(x) 5

H(x) 5x C

d) k(x)

K(x) ln | x | C

2. Determina la funció primitiva de la funció:

f(x) cos x

la gràfica de la qual passi pel punt de coordenades ( /2, 4).

F(x) sin x C

F 4 4 sin C 4 1 C C 3

F(x) sin x 3

3. Sabem que la funció:

és una primitiva de la funció f(x). Quina és la funció f(x)?

4. Comprova que totes les primitives de la funció f(x) ln x són del tipus F(x) x (ln x 1) C.

F ’(x) ln x 1 x ln x 1 1 ln x f(x)

5. Si G1(x) i G2(x) són dues primitives d’una mateixa funció g(x), es poden tallar les seves gràfiques? Dibuixa la gràfica de la funció G1(x) sabent que passa pel punt (0, 4) si la gràfica de la funció G2(x) és la de la figura.

No es poden tallar, ja que les expressions algebraiques de les funcions G1(x) i G2(x) només es diferencien en una constant.

Page 64: MATEMÀTIQUES - · PDF fileMatemàtiques 2 · Batxillerat · Solucionari No és permesa la reproducció total o parcial d’aquest llibre, ni el seu tractament informàtic, ni la

64 SOLUCIONARI DEL LLIBRE DE L’ALUMNE4

6. Calcula la derivada de les funcions següents i escriu-ne des-prés les corresponents integrals indefinides:

a) f(x) tg x

b) g(x) 23x 5

c)

d) k(x) ln2 x

7. Troba la derivada de les funcions següents:

a) f(x) x 3x dx

f‘(x) x3x

b) g(x) cos2 x dx

g’(x) cos2 x

c) h(x) (tg x ln x) dx

h’(x) tg x ln x

d) k(x) x2 ex dx

k’(x) x2 ex

8. Un mòbil recorre una trajectòria rectilínia amb una acce-leració constant de 2 m/s2. Se sap que en el moment de començar a comptar el temps, v(0) 3 m/s i s(0) 5 m.

Troba les expressions de les funcions v v(t) i s s(t) cor-responents al seu moviment.

Cal que recordis:

derivant derivant

v(0) 3 m/s 3 2·0 C C 3 m/s

v(t) (2t 3) m/s

s(0) 5 m 5 02 3·0 C’ C’ 5 m

s(t) (t2 3t 5) m

9. Comprova que les derivades de les funcions següents:

F(x) , n R, n 1 i

són, respectivament, f(x) x n i g(x) ax.

10. Troba x 1 dx.

11. Determina la primitiva de la funció f(x) 1 tg2 x la gràfica

de la qual conté el punt ( , 3).

12. Calcula les primitives següents:

a)

b)

x x x xx

C x C3434

74

74

74

47

d d

c)

xx

xxx

x x xx

xx

C3

2

13

2

53

23

2323

3

2d d d d

Page 65: MATEMÀTIQUES - · PDF fileMatemàtiques 2 · Batxillerat · Solucionari No és permesa la reproducció total o parcial d’aquest llibre, ni el seu tractament informàtic, ni la

65MATEMÀTIQUES 2 4

d)

13. Calcula:

a)

2 1 2 1

132

23

1

2 212

232

x x x x x x

xC x

d d( )

( )( 2 3) C

b)

c)

d)

e)

f)

14. Troba la primitiva de la funció f(x) sin x cos x la gràfica de

la qual passa pel punt .

15. Justifica el motiu pel qual podem afirmar que no hi ha cap

primitiva de la funció f(x) que presenti màxims

ni mínims relatius en el seu domini.

Sigui F(x) una primitiva de f(x).

F

Per trobar els màxims i mínims relatius de F(x) cal resoldre l’equació F ’(x) 0. És senzill observar que aquesta equació no té solució.

16. Troba la primitiva de la funció f(x) sin x ecos x la gràfica de la qual talla l’eix d’abscisses en x .

17. Calcula:

a) 4x3 sin (x4 3) dx

b) dx

c) dx

d) dx

Page 66: MATEMÀTIQUES - · PDF fileMatemàtiques 2 · Batxillerat · Solucionari No és permesa la reproducció total o parcial d’aquest llibre, ni el seu tractament informàtic, ni la

66 SOLUCIONARI DEL LLIBRE DE L’ALUMNE4

e) dx

f) (tg2 x tg4 x) dx

18. Calcula:

a) (3x2 1) cos (x3 x) dx

b) dx

2

12 1

112

2 1

2

212

212

x

xx x x x

xC

d d( )

( )x C2

c) 3x2 sin x3 dx

d) dx

e) dx

f) dx

19. Determina les asímptotes de la funció:

F(x) dx sabent que F( 2) 2.

.

Asímptota vertical: la recta x 3.

Asímptota horitzontal: la recta y 3.

20. Calcula:

a) (2x3 3x2 5x 1) dx

b) dx

c) (32x e4x 1) dx

e

e

e

d) dx

Page 67: MATEMÀTIQUES - · PDF fileMatemàtiques 2 · Batxillerat · Solucionari No és permesa la reproducció total o parcial d’aquest llibre, ni el seu tractament informàtic, ni la

67MATEMÀTIQUES 2 4

e) (2x 3)(2x 3) dx

f) dx

g) dx

h) dx

21. Sabem que la gràfica d’una funció passa pel punt P(1, 4) i que el pendent de la recta tangent en qualsevol punt d’aquesta gràfica s’expressa mitjançant m(x) 2x2 3x 5. Determina l’expressió algèbrica d’aquesta funció.

22. Troba la primitiva de la funció f(x) que s’anul·la quan x 2.

23. Calcula tg2 x dx.

Et suggerim que apliquis l’estratègia següent:

tg2 x 1 tg2 x 1

24. Calcula:

a) 5cos (3x 2) dx

b) dx

c) dx

d) dx

e) dx

f) dx

g) dx

Page 68: MATEMÀTIQUES - · PDF fileMatemàtiques 2 · Batxillerat · Solucionari No és permesa la reproducció total o parcial d’aquest llibre, ni el seu tractament informàtic, ni la

68 SOLUCIONARI DEL LLIBRE DE L’ALUMNE4

h) dx

i) dx

j) dx

25. Calcula:

a) x sin x dx

b) e2x sin x dx

c) ln x dx

d) x ln x dx

e) 2x x dx

f) arcsin x dx

Page 69: MATEMÀTIQUES - · PDF fileMatemàtiques 2 · Batxillerat · Solucionari No és permesa la reproducció total o parcial d’aquest llibre, ni el seu tractament informàtic, ni la

69MATEMÀTIQUES 2 4

g) (x 2) e3x dx

h) dx

i) (3x 2) cos x dx

j) dx

26. Ja has vist que, de vegades, cal aplicar en més d’una ocasió el mètode d’integració per parts. Et caldrà fer-ho en el càlcul de les primitives següents:

a) x2 e5x dx

b) x3 sin x dx

Page 70: MATEMÀTIQUES - · PDF fileMatemàtiques 2 · Batxillerat · Solucionari No és permesa la reproducció total o parcial d’aquest llibre, ni el seu tractament informàtic, ni la

70 SOLUCIONARI DEL LLIBRE DE L’ALUMNE4

c) (x2 4) · 3x dx

d) x2 cos x dx

e) dx

f) (1 x2) 23x dx

Page 71: MATEMÀTIQUES - · PDF fileMatemàtiques 2 · Batxillerat · Solucionari No és permesa la reproducció total o parcial d’aquest llibre, ni el seu tractament informàtic, ni la

71MATEMÀTIQUES 2 4

27. a) Resol l’equació F’(x) 0 si F(x) ex x(2 x) dx.

Les solucions són x1 0 i x2 2.

b) Calcula la primitiva de la funció f(x) ex x(2 x) la grà-fica de la qual passa per l’origen de coordenades.

e d e dx xx x x x x x( ) ( )2 22

28. Calcula amb els canvis de variable indicats:

a) dx amb x 4t.

b) dx amb t.

29. Aplicant el canvi de variable sin x t, calcula:

cos3 x dx

Si tens en compte la igualtat següent:

cos3 x cos2 x cos x (1 sin2 x) cos x

la pots calcular sense canvi de variable. Fes-ho.

30. Calcula la integral dx utilitzant el canvi de varia-ble x sin t o x cos t. Arribaràs a una integral del tipus:

cos2 t dt o sin2 t dt

respectivament. Et caldrà fer ús de les identitats trigonomè-triques:

cos2 t o sin2 t

Page 72: MATEMÀTIQUES - · PDF fileMatemàtiques 2 · Batxillerat · Solucionari No és permesa la reproducció total o parcial d’aquest llibre, ni el seu tractament informàtic, ni la

72 SOLUCIONARI DEL LLIBRE DE L’ALUMNE4

31. La integral dx és quasi immediata. Calcula-la.

Comprova que arribes al mateix resultat aplicant-hi el canvi de variable x2 3 t.

32. Calcula:

a)

b)

c)

d)

Page 73: MATEMÀTIQUES - · PDF fileMatemàtiques 2 · Batxillerat · Solucionari No és permesa la reproducció total o parcial d’aquest llibre, ni el seu tractament informàtic, ni la

73MATEMÀTIQUES 2 4

e)

f)

33. Calcula fent el canvi de variable x t6.

t t

t t t t

tt t

tt

3

3 2 2

2

2

1

1

11

34. Calcula:

a)

x x

x x xxx

2

2

5

5 555 25

25

b)

x x x

x x x

xx x

x

3 2

3 2

2

2

4 2

2 22 42 4

4 4

c)

Page 74: MATEMÀTIQUES - · PDF fileMatemàtiques 2 · Batxillerat · Solucionari No és permesa la reproducció total o parcial d’aquest llibre, ni el seu tractament informàtic, ni la

74 SOLUCIONARI DEL LLIBRE DE L’ALUMNE4

A 2, C 2

d)

x x

x

2 2

2

9 9

9 118

Activitats finals

1. Determina la funció f(x) sabent que la funció F(x) x2 ex 2 n’és una primitiva.

2. Quina és la primitiva de la funció f(x) 2x 5 que verifica la condició F(1) 9? I la que verifica F 1(3) 3?

3. Dos companys obtenen resultats diferents en el càlcul de les primitives d’una mateixa funció. El primer obté:

i el segon,

Indica raonadament quin dels dos ha arribat a la resposta correcta.

El segon, ja que si , es compleix que

4. Calcula:

a) (2x3 3x2 1) dx

b) ( 3·3x 4 cos x) dx

c)

Page 75: MATEMÀTIQUES - · PDF fileMatemàtiques 2 · Batxillerat · Solucionari No és permesa la reproducció total o parcial d’aquest llibre, ni el seu tractament informàtic, ni la

75MATEMÀTIQUES 2 4

d)

e) (4 tg2 x) dx

f)

5. Calcula les integrals quasi immediates següents:

a) cos 5x dx

b)

c) 5 sin4 x cos x dx

d)

e)

3 7

3 21 3 212121 147

147

2

2

x x

x x xxx

f) x3 sin (x4 ) dx

6. Calcula les integrals quasi immediates següents:

a) 5tg x (1 tg2 x) dx

b)

c)

d)

e)

f)

Page 76: MATEMÀTIQUES - · PDF fileMatemàtiques 2 · Batxillerat · Solucionari No és permesa la reproducció total o parcial d’aquest llibre, ni el seu tractament informàtic, ni la

76 SOLUCIONARI DEL LLIBRE DE L’ALUMNE4

7. Troba la primitiva de la funció:

que verifica la condició .

8. Troba l’expressió de la funció F(x) la gràfica de la qual passa pel punt (1, 1) sabent que el pendent de la recta tangent en qualsevol punt ve donat per la funció m(x) 3x2 6x 4.

9. Considera la funció .

Determina’n les asímptotes, sabent que F(0) 3.

4x 16 0 x 4

Asímptota vertical x 4.

Asímptota horitzonal y .

10. Calcula per parts les integrals següents:

a) x2 sin 3x dx

b) cos (ln x) dx

c) x3 ln x dx

Page 77: MATEMÀTIQUES - · PDF fileMatemàtiques 2 · Batxillerat · Solucionari No és permesa la reproducció total o parcial d’aquest llibre, ni el seu tractament informàtic, ni la

77MATEMÀTIQUES 2 4

d) e4x cos 4x dx

e) (x 1) 5x dx

f) ln2 x dx

11. Comprova que:

(x2 2x 1) ex dx (x2 4x 3) ex C

12. Calcula les integrals següents, fent ús en cada cas del canvi de variable indicat:

a) dx, x 2 sin t

b) dx, x 3t

c) dx, t

Page 78: MATEMÀTIQUES - · PDF fileMatemàtiques 2 · Batxillerat · Solucionari No és permesa la reproducció total o parcial d’aquest llibre, ni el seu tractament informàtic, ni la

78 SOLUCIONARI DEL LLIBRE DE L’ALUMNE4

t t

t

2 2

2

1

1 11

d) dx, x 6t

13. Calcula la integral dx mitjançant el canvi de va-

riable . Aquesta integral, però, és quasi immediata.

Calcula-la també sense fer canvi de variable.

14. Troba la primitiva de la funció , la gràfica de la

qual passa pel punt (2, 2).

x xx

2 11 13

15. Calcula les integrals següents:

a)

x x

x

2 2

2

1

1 11

Page 79: MATEMÀTIQUES - · PDF fileMatemàtiques 2 · Batxillerat · Solucionari No és permesa la reproducció total o parcial d’aquest llibre, ni el seu tractament informàtic, ni la

79MATEMÀTIQUES 2 4

b)

c)

d)

x x x

x xx

2 2

2

1 4

4 14 1

e)

17

17

18

1156

7

2( )( )

ln ln ln

x x xx

x x x C

d

f)

3 5 3 13 1 1

4

x xx

Page 80: MATEMÀTIQUES - · PDF fileMatemàtiques 2 · Batxillerat · Solucionari No és permesa la reproducció total o parcial d’aquest llibre, ni el seu tractament informàtic, ni la

80 SOLUCIONARI DEL LLIBRE DE L’ALUMNE4

16. Troba una primitiva de la funció següent:

Suggeriment: descompon la fracció en suma de dues frac-cions del mateix denominador.

Com que ens demanen una primitiva, fem, per exemple, C 0.

17. Troba la primitiva de la funció , la gràfica

de la qual té per asímptota horitzontal la recta y 2.

18. Determina la primitiva de la funció , la gràfica

de la qual conté el punt (4, 0). Anomena F(x) aquesta fun-

ció i calcula F(x) i F(x).

Dibuixa de manera aproximada la gràfica de la funció F(x).

19. Calcula les integrals següents:

a) (tg5 x tg7 x) dx

b) dx

x xcos2

c)

d)

e)

f)

Page 81: MATEMÀTIQUES - · PDF fileMatemàtiques 2 · Batxillerat · Solucionari No és permesa la reproducció total o parcial d’aquest llibre, ni el seu tractament informàtic, ni la

81MATEMÀTIQUES 2 4

x x

x x x

xx

4 2

4 2 2

2

2

1

1

11

g)

h)

20. Troba l’expressió algèbrica de la funció F(x) que verifica les condicions següents:

a) F’(x) 2x 6

b) La gràfica de la funció F(x) presenta un mínim en el punt d’ordenada 1.

Mínim

21. Calcula sin2 x dx i cos2 x dx a partir de les igualtats següents:

sin2 x cos2 x 1 i cos 2x cos2 x sin2 x

22. Calcula

Indicació: multiplica primer numerador i denominador per l’expressió conjugada del denominador.

23. Calcula

Indicació: multiplica primer numerador i denominador per 1 sin x.

24. Aplicant el mètode d’integració per parts, calcula cos2 x dx. Cal que tinguis en compte que cos2 x cos x cos x i que

sin2 x 1 cos2 x. Compara el resultat amb el que has obtingut a l’activitat 21.

Page 82: MATEMÀTIQUES - · PDF fileMatemàtiques 2 · Batxillerat · Solucionari No és permesa la reproducció total o parcial d’aquest llibre, ni el seu tractament informàtic, ni la

82 SOLUCIONARI DEL LLIBRE DE L’ALUMNE4

25. Un mòbil es desplaça sobre l’eix OX de manera que la seva acceleració ve donada per l’equació següent:

a 2t 1 m/s2

Si per a t 0 es verifica v(0) 2 m/s i x(0) 10 m, troba les expressions de les funcions velocitat v v(t) i posició x x(t) corresponents a aquest mòbil.

v t t x t t C

v C

v t t t

( ) ( )

( )

( )

2 1

0 2 2

2

2

d

22

23 2

2

0 10

23 2

ms

dx t t t xt t

t C

x

( ) ( )

( ) CC

x t t t t

10

13

12

2 103 2( ) m

26. Un mòbil descriu un moviment vibratori harmònic simple l’acce-leració del qual s’expressa per l’equació a 36 cos 3t cm/s2.

Si a l’instant inicial es verifica v(0) 0 cm/s i x(0) 4 cm, troba les expressions de les funcions velocitat v v(t) i posició x x(t) d’aquest mòbil.

v t t t t t t C

v

( ) cos cos sin

(

36 3 12 3 3 12 3

0

d d

))

( ) sin

( ) sin

0 0

12 3

12 3 4

C

v t t

x t t t

cms

d 3 3 4 3

0 4 4 4 0

sin cos

( )

(

t t t C

x C C

x t

d

) cos4 3t cm

Avaluació

1. Donada ƒ(x) (2x 1) ex2 x, determina la funció g(x) tal que g’(x) ƒ(x) (és a dir, una primitiva de ƒ(x)) i que el seu gràfic passa pel punt (0, 2).

Les funcions primitives de ƒ(x) (2x 1) e(x2 x) són de la forma g(x) e(x2 x) C, on C és qualsevol constant. Si es demana que el seu gràfic passi pel punt (0,2) cal que 2 g(0) e0 C; d’on es dedueix que C 1 i g(x) e(x2 x) 1.

2. Donada la funció ƒ(x) cos x cos3 x:

a) Troba la seva integral indefinida. Indicació: recorda que sin2 x cos2 x 1.

b) Quina és la primitiva de ƒ(x) que passa pel punt ?

Si la primitiva ha de passar pel punt

La primitiva demanada és, doncs,

3. Donada la funció f(x)

a) Calcula la integral f x x( )d

b) Troba la primitiva F(x) de ƒ(x) que compleixi F(1) 1

4. Calcula les primitives següents:

a)

b)

f x x g x x f x g xx

( ) ( ) cos ( ) ( )sin

i i3 13

3

x x xx x

x x

x x

cos( )sin

sin

sin

33

313

3

33

19

d d

cos3x C

c)

Page 83: MATEMÀTIQUES - · PDF fileMatemàtiques 2 · Batxillerat · Solucionari No és permesa la reproducció total o parcial d’aquest llibre, ni el seu tractament informàtic, ni la

83MATEMÀTIQUES 2 5

j Unitat 5. La integral

Activitats

1. Representa gràficament la funció f(x) x2 8x en l’inter-val [0, 4]. Calcula les sumes inferior i superior per estimar l’àrea sota la corba en aquest interval. Pots prendre n 8.

f(x) x2 8x

Una estimació és la mitjana d’aquests valors:

2. Calcula, en l’interval [0, 4], l’àrea sota la gràfica de la fun-ció f(x) x 5. Fes-ne una gràfica i justifica per què pots calcular exactament aquesta àrea.

f(x) x 5

u2

3. Considera la funció:

de l’exemple 1. Fes una partició de l’interval [0, 3] en 12 sub-intervals. Calcula la suma de les àrees inferior i superior.

Comprova que l’estimació feta abans és correcta.

8,08

La mitjana de les dues sumes és: A 7,515 u2.

4. Representa gràficament la funció següent: f(x) ex. Ex-pressa la integral definida de f(x) en l’interval [0, 1]. En-

cara que no sabem calcular la integral , coincideix

aquesta integral amb l’àrea entre la corba i l’eix OX en el mateix interval? Raona la teva resposta.

f(x) ex

f(x) ex

i l’àrea no coincideix amb la integral.

Page 84: MATEMÀTIQUES - · PDF fileMatemàtiques 2 · Batxillerat · Solucionari No és permesa la reproducció total o parcial d’aquest llibre, ni el seu tractament informàtic, ni la

84 SOLUCIONARI DEL LLIBRE DE L’ALUMNE5

5. Utilitza les propietats lineals c) i d) de la integral definida per expressar

0

com a suma d’integrals.

0

0 0 0

6. L’expressió no és una integral, encara que ho sembli.

Explica el perquè d’aquesta afirmació.

La funció no és contínua a [ 1, 1] i, per tant, no

existeix la integral en aquest interval.

7. Raona la certesa o no de cadascuna de les sumes següents:

a)

La igualtat és falsa. Cada una de les integrals sumants és 0 i la suma diferent de 0.

b)

La igualtat és certa. El primer sumant és 0, però els altres dos corresponen a intervals consecutius com el de la integral suma.

8. Troba la derivada de cadascuna de les funcions següents:

La derivada de cada funció és la funció a integrar. N’hi ha prou de canviar la t per x.

9. Calcula les integrals següents:

a)

21867

b)

c)

La primitiva per parts dues vegades:

d)

e)

f)

g)

Page 85: MATEMÀTIQUES - · PDF fileMatemàtiques 2 · Batxillerat · Solucionari No és permesa la reproducció total o parcial d’aquest llibre, ni el seu tractament informàtic, ni la

85MATEMÀTIQUES 2 5

h)

Cal dividir el numerador pel denominador:

10. Calcula i l’àrea sota la corba de la funció

f(x) cos x dx en l’interval [0, ]. Coincideixen els dos re-sultats? Raona la teva resposta.

No coincideixen els dos resultats ja que en l’interval [0, ] la funció f(x) cos x canvia de signe.

11. Expressa i calcula l’àrea entre la gràfica de la funció f(x) (x 1) (x2 x 6) i l’eix OX.

La funció talla l’eix 0X en els punts x 2, x 1, x 3

u

12. Calcula l’àrea entre la gràfica de la funció f(x) x3 3x i l’eix OX.

La funció talla l’eix 0X en els punts:

i presenta simetria imparella i l’àrea es pot expressar:

13. Calcula l’àrea de la regió compresa entre les gràfiques de les funcions f(x) x2 i g(x) x2 8.

Punts d’intersecció: f(x) g(x) x2 x2 8 x 2

14. Representa gràficament les funcions f(x) x2 4x i

g(x) . Calcula l’àrea de la regió compresa entre les dues

gràfiques.

Punts d’intersecció:

x

x

0

52

Page 86: MATEMÀTIQUES - · PDF fileMatemàtiques 2 · Batxillerat · Solucionari No és permesa la reproducció total o parcial d’aquest llibre, ni el seu tractament informàtic, ni la

86 SOLUCIONARI DEL LLIBRE DE L’ALUMNE5

15. Representa gràficament la funció f(x) ln x en l’interval [1, e]. Calcula l’àrea sota aquesta corba en aquest interval.

16. La funció f(x) x2 x4 presenta simetria parella en la seva gràfica. Pots comprovar-ho. Calcula l’àrea sota aquesta corba i l’eix OX. Pots fer-ho calculant només una integral? Fes-ho així i t’estalviaràs càlculs.

f(x) talla l’eix 0X en els punts: x 1, x 0 i x 1. Per la sime-tria parella de la funció l’àrea es pot expressar per:

17. Calcula l’àrea ombrejada de la figura. Les funcions represen-tades són:

g(x) x2 2x

Intersecció de les gràfiques:

Cal calcular també l’àrea en l’interval

18. Calcula el volum que genera la paràbola y x2 en girar a l’en-torn de l’eix OX en l’interval [0, 3]. Fes-ne la representació gràfica.

19. Dibuixa la gràfica de la funció f(x) sin x en l’interval [0, ]. Calcula el volum del cos que genera en girar a l’entorn de l’eix de les abscisses.

20. Considera la recta d’equació y 2x en l’interval [0, 2]. Si gira a l’entorn de l’eix de les abscisses, quin cos genera? I si consideres que ho fa en l’interval [1, 3], de quin cos es tracta? Calcula el volum de cadascun d’aquests cossos.

La funció y 2x en l’interval [0,2] genera un con i en l’interval [1, 3] un tronc de con.

Volum del con:

Volum del tronc de con:

21. Expressa i calcula el volum del cos generat per una circum-ferència de centre (3, 0) i radi 5.

El cos generat és una esfera i el seu volum només depén del radi. Podem situar la circumferència centrada a l’origen de co-ordenades:

Page 87: MATEMÀTIQUES - · PDF fileMatemàtiques 2 · Batxillerat · Solucionari No és permesa la reproducció total o parcial d’aquest llibre, ni el seu tractament informàtic, ni la

87MATEMÀTIQUES 2 5

V x x xx

2 25 2 253

5003

2

0

53

0

5

3( )d u

Activitats finals

1. Calcula la derivada de F(x) .

F ’(x) 0 ja que F(x) és una constant.

2. Calcula F’(1) si F(x) .

F ’(1) 0. En substituir x per 1, F és una constant.

3. Tenim una funció y f(x) de la qual l’única cosa que sa-bem és que la seva gràfica és aproximadament la que s’in-dica a la figura. Fes un esquema senzill de la gràfica de la

funció . Raona molt detalladament la

resposta.

g’(x) f(x). La gràfica de la figura de l’enunciat és la de g’(x) i la que donem és la de g(x). En els punts x 0, x 2 i x 4 hi ha extrems relatius.

4. Calcula les integrals següents:

a)

b)

c)

d)

5. Calcula: e . Per trobar una primitiva és necessari que apliquis el mètode d’integració per parts dues vegades.

6. Troba una primitiva de la funció

i calcula la integral d’aquesta funció en l’interval [3, 5].

Cal dividir el numerador pel denominador:

Page 88: MATEMÀTIQUES - · PDF fileMatemàtiques 2 · Batxillerat · Solucionari No és permesa la reproducció total o parcial d’aquest llibre, ni el seu tractament informàtic, ni la

88 SOLUCIONARI DEL LLIBRE DE L’ALUMNE5

7. Calcula dx utilitzant el canvi de variable: x 2 sin t.

La primitiva:

x t x t1 222

0 0

8. Fent el canvi de variable u(x) ex, calcula la integral:

9. Calcula l’àrea compresa entre la gràfica de la funció

f(x) , l’eix de les abscisses i les ordenades corresponents

a x 1 i x 2.

10. En una comarca un riu adopta la forma de la funció

f(x) x3 x2 x i és tallat per un camí que té la direcció

positiva de l’eix OX. Prenent com a unitat el km, calcula el valor del camp comprès entre el riu i el camí si el preu és de 300 l’hectàrea.

Preu:

11. Calcula l’àrea limitada per la gràfica de la funció següent y x2 3x i l’eix OX.

Les interseccions amb l’eix 0X són: x 0 i x 3

12. Calcula l’àrea determinada per la funció f(x) x3 x i l’eix de les abscisses.

Punts d’intersecció amb l’eix 0X:

x3 x 0 x 0, x 1

La funció presenta simetria imparella.

13. Troba els punts de tall amb l’eix de les abscisses de la funció f(x) x3 2x2 5x 6 i expressa i calcula l’àrea compresa entre la gràfica de la funció i aquest eix.

Punts de tall: f(x) 0 x 2, x 1 i x 3

Page 89: MATEMÀTIQUES - · PDF fileMatemàtiques 2 · Batxillerat · Solucionari No és permesa la reproducció total o parcial d’aquest llibre, ni el seu tractament informàtic, ni la

89MATEMÀTIQUES 2 5

14. Representa gràficament la funció f(x) (x 2) ex en l’in-terval [2, 3]. Calcula l’àrea sota aquesta corba en aquest interval.

4 3 2 1 1 2 3 4

3

2

1

1

2

3

4

e

La primitiva per parts:

L’àrea és

15. Fes la gràfica de la funció f(x) sin x en l’interval [0, 2 ] i calcula l’àrea compresa entre aquesta gràfica i les ordenades corresponents a x i x .

16. Calcula l’àrea entre la corba i les rectes x 0,

x i l’eix OX.

17. Considera la funció f(x) ln x. Calcula l’àrea sota aquesta corba en l’interval [1, 2]. Pots fer el mateix en l’interval [0, 2]? Raona’n la resposta.

No es pot calcular l’àrea en l’interval [0,2] perquè la funció f(x) ln x no és contínua a x 0.

18. Calcula l’àrea entre la gràfica de la funció f(x) 2x ln x en l’interval [a, 1]. Pot ser a un nombre negatiu? I a 0? Per què?

Cal fer la primitiva per parts:

Cal que a > 0 ja que no existeixen el logaritme de a < 0 i de a 0.

19. Representa gràficament la funció f(x) sin x en l’interval

[ , ]. Raona quin és el valor de la integral

sense calcular-la.

–3 –2 –1 1 2 3 4

–3

–2

–1

1

2

El valor de la integral: , ja que f(x) sinx és una

funció que presenta simetria imparella en aquest interval.

20. Demostra que no és necessari calcular la primitiva de la in-tegral següent:

Estudia la simetria que pot presentar la gràfica de la funció que s’ha d’integrar.

La funció presenta simetria imparella en l’interval [ 1, 1], és a dir,

f(x) < 0 a [ 1, 0) i f(x) > 0 a (0, 1]

i, per tant, la integral és 0.

Page 90: MATEMÀTIQUES - · PDF fileMatemàtiques 2 · Batxillerat · Solucionari No és permesa la reproducció total o parcial d’aquest llibre, ni el seu tractament informàtic, ni la

90 SOLUCIONARI DEL LLIBRE DE L’ALUMNE5

21. Demostra que l’àrea d’un cercle de radi r és donada per A r2. Per fer-ho considera una circumferència centrada a l’origen de radi r de la qual només tindràs en compte la se-micircumferència positiva. Calcula l’àrea sota aquesta corba i tindràs l’àrea del semicercle de radi r.

Circumferència:

Àrea del semicercle:

Canvi de variable:

Àrea del cercle:

22. Determina els valors de a, b i c en el polinomi

P(x) ax2 bx c

si verifica P(1) 4, P ’(1) 8 i P(2) 15P(0) 0. Repre-senta la funció i calcula l’àrea compresa entre la corba i l’eix OX.

P(1) a b c 4 P ’(x) 2ax b

P ’(1) 2a b 8

P(2) 15P(0) 4a 2b c 15c 0

En resoldre el sistema s’obté: a 3, b 2 i c 1

P(x) 3x2 2x 1 és una paràbola que talla l’eix 0X en els punts

A x x dx x x x( )3 2 13227

21

13 3 2

1

13 2u

23. Troba l’àrea de la zona limitada per les funcions f(x) x3 i g(x) 2x. Fes-ne una gràfica.

L’àrea està formada per dues regions iguals:

24. Troba l’àrea compresa entre les funcions

Punts d’intersecció:

25. Representa les funcions y sin x i y cos x en una mateixa gràfica en l’interval . Calcula l’àrea compresa entre les dues funcions en aquest interval.

Page 91: MATEMÀTIQUES - · PDF fileMatemàtiques 2 · Batxillerat · Solucionari No és permesa la reproducció total o parcial d’aquest llibre, ni el seu tractament informàtic, ni la

91MATEMÀTIQUES 2 5

Les dues funcions es tallen en donant dues regions

d’àrees A1 i A2:

26. Representa les funcions y ex i y e x en una mateixa gràfica. Calcula l’àrea limitada per les dues corbes i la recta x 1.

e e 1 2 1,086 u2

27. Calcula l’àrea del recinte limitat per les dues paràboles d’equacions:

y x2 2x i y x2 4x

Interseccions:

28. Troba el valor de a 0 si sabem que l’àrea entre la parà-bola y x2 ax i la recta y x 0 és 36 u2.

Interseccions de les dues gràfiques:

x2 ax x x2 ax x 0 i x1 0 i x2 (1 a)

29. Calcula l’àrea del recinte limitat pels gràfics de les dues fun-cions següents: f(x) x3 2x, g(x) x2, quan considerem només valors de x 0.

f(x) g(x) x3 2x x2 x1 0, x2 2 i x3 1

En la regió negativa és l’interval: [ 1, 0]

d

Page 92: MATEMÀTIQUES - · PDF fileMatemàtiques 2 · Batxillerat · Solucionari No és permesa la reproducció total o parcial d’aquest llibre, ni el seu tractament informàtic, ni la

92 SOLUCIONARI DEL LLIBRE DE L’ALUMNE5

30. Considera un recinte tancat limitat per la paràbola d’equació y x2 1 i la recta horitzontal d’equació y a, on a és un número més petit que 1. Determina el valor de a per tal que

l’àrea d’aquest recinte valgui u2.

Interseccions:

31. Calcula el volum de l’el·lipsoide generat per l’el·lipse

d’equació 1 en girar a l’entorn del seu eix ma-jor.

En l’equació de l’el·lipse

32. Calcula el volum generat per la funció f(x) x3 1 en girar a l’entorn de l’eix OX en l’interval [0, 2].

33. Considera la corba d’equació 4x2 y2 1. Defineix la funció y f(x) associada a aquesta funció. Quin és el seu domini? En aquest domini, calcula el volum del cos que genera en girar a l’entorn de l’eix OX.

Avaluació

1. Calcula

Fent el canvi immediat t lnx la integral es converteix en

que val .

2. Considera la regió S del pla limitada per la paràbola y 3x2 i la recta y 3 representada en l’esquema següent:

Siguin A i B els punts d’intersecció de la recta i la paràbola, i T el triangle que té per vèrtexs A, B i l’origen de coordenades (0, 0). Calcula l’àrea de la regió que resulta quan es treu el triangle T a la regió S.

Els punts A, B d’intersecció entre la recta i la paràbola són:

Calculem l’àrea d’intersecció entre la paràbola i la recta entre els punts x 1 i x 1. Com que la superfície és simètrica respecte l’eix d’ordenades, calculem només la meitat de l’àrea:

y

x

A B

O

3

Page 93: MATEMÀTIQUES - · PDF fileMatemàtiques 2 · Batxillerat · Solucionari No és permesa la reproducció total o parcial d’aquest llibre, ni el seu tractament informàtic, ni la

93MATEMÀTIQUES 2 5

L’àrea total de la intersecció, per tant, és S 2 2 4 u2.

L’àrea del triangle que formen els punts A, B i l’origen de coor-

denades és .

Per tant, l’àrea de la regió que resulta quan es treu el triangle T a la regió S és .

3. Considera la funció ƒ(x) de la figura definida a l’interval [0, 2].

a) Calcula la funció derivada ƒ’(x) a l’interval (0, 2)

El pendent del primer tram, entre 0 i 1, és 1. El pendent del segon tram, entre 1 i 2, és 1/2. Per tant,

b) Hi ha algun punt de (0, 2) en el qual ƒ’(x) no existeixi?

Evidentment, ƒ’(x) no existeix per a x 1, perquè en el punt (1, 1) la gràfica de ƒ(x) no té tangent. Alternativament es pot argumentar que en el punt x 1, la derivada de ƒ(x) per l’esquerra és 1 i per la dreta 1/2.

c) Calcula ƒ(x) dx.

Raona totes les respostes.

La integral demanada es pot calcular com la suma de les àrees del triangle per sobre de (0, 1) i del trapezi per sobre de (1, 2):

u2

o bé es pot calcular obtenint l’expressió analítica de ƒ(x):

4. Considera la funció ƒ(x)

a) Calcula el valor de m per tal que l’àrea del recinte limitat per la gràfica de la funció, l’eix OX i les rectes x 0 i x 2 sigui de 10 u2.

La funció es manté positiva per sobre de l’interval [0,2] per-què per a x ≥ 0 i m ≥ 0 forçosament .

Alternativament es pot raonar que ƒ(x) es manté positiva a l’interval [0,2] perquè ƒ(0) 1 i ƒ’(x) 3x2 2mx 0 per

a x [0,2] (recordem que m ≥ 0).

En conseqüència, l’àrea del recinte que ens diu l’enunciat és:

b) Per a m 1, indica el punt o els punts on la recta tangent a la gràfica de la funció forma un angle de 45° amb el semieix positiu de OX.

Per a m 1 la funció és f(x) .

El pendent de la recta tangent que ens diuen ha de ser igual a

1. Per tant, el punt o punts que ens demanen han de ser aquells

on f’(x) . O sigui i .

Els punts en qüestió són ( 1, 1) i .

Page 94: MATEMÀTIQUES - · PDF fileMatemàtiques 2 · Batxillerat · Solucionari No és permesa la reproducció total o parcial d’aquest llibre, ni el seu tractament informàtic, ni la

94 SOLUCIONARI DEL LLIBRE DE L’ALUMNE6

j Bloc 2. Matrius i sistemes

j Unitat 6. Vectors a l’espai

Activitats

1. Indica en cada cas com identificaries a partir de les seves coordenades:

a) Un punt de l’eix X.

( , , ), , , .x x y z0 0 R

b) Un punt de l’eix Y.

( , , ), , , .0 0y x y z R

c) Un punt de l’eix Z.

( , , ), , , .0 0 z x y z R

d) Un punt del pla XY.

( , , ), , , .x y x y z0 R

e) Un punt del pla XZ.

( , , ), , , .x x y z0 0 R

f) Un punt del pla YZ.

(0, y, z) amb y i z R.

2. Dibuixa uns eixos de coordenades i representa-hi els punts següents:

A(5, 3, 2); B( 4, 6, 3); C(0, 0, 6); D(2, 5, 0)

3. Digues les coordenades dels punts següents:

a) El punt P que es troba a l’eix X a distància 3 de l’origen de coordenades en el sentit negatiu de l’esmentat eix.

P( 3, 0, 0)

b) El punt Q que es troba en el pla YZ a distància 5 de l’ori-gen de coordenades i tal que el vector OQ forma un angle de 30° amb el sentit positiu de l’eix Y.

c) El punt R situat en el pla XZ a distància 4 de l’origen i tal que la recta que passa per O i per R forma un angle de 60° amb el sentit positiu de l’eix Z.

4. Donat el vector v (2, 4, 3), dibuixa en uns eixos de coorde-nades el seu representant que té per origen el punt A(3, 2, 4). Quines són les coordenades de l’extrem d’aquest vector?

Si B(x, y, z), v AB (2, 4, 3) (x 3, y 2, z 4)

2 x 3 x 54 y 2 y 63 z 4 z 7

L’extrem del vector v és el punt B(5, 6, 7).

5. Determina els components cartesians i el mòdul de cadascun dels vectors següents:

a) AB amb A( 3, 1, 5) i B(2, 4, 1)

AB AB

Page 95: MATEMÀTIQUES - · PDF fileMatemàtiques 2 · Batxillerat · Solucionari No és permesa la reproducció total o parcial d’aquest llibre, ni el seu tractament informàtic, ni la

95MATEMÀTIQUES 2 6

b) CD amb C(0, 4, 1) i D( 3, 5, 6)

CDCD

c) EF amb E(1, 1, 3) i F( 2, 3, 1)

EFEF

d) GH amb G( 6, 10, 3) i H( 5, 8, 1)

GH GH

6. Representant el vector v (2, 6, 1) amb origen al punt M, s’obté el punt N(1, 0, 4). Determina les coordenades de M.

Anomenem M(x, y, z).

Si v MN , es verifica:

(2, 6, 1) (1 x, y, 4 z)

Per tant:2 1 x x 16 y y 61 4 z z 3

M( 1, 6, 3)

7. Quins són els components del vector nul? I el seu mòdul?

O

O

8. Els vectors PQ i RS són equipol·lents. Si P(0, 1, 3), Q(3, 4, 1) i S( 4, 2, 1), esbrina les coordenades del punt R.

Anomenem R(x, y, z)

PQ RS (3, 5, 2) ( 4 x, 2 y, 1 z) 3 4 x x 7 5 2 y y 7 2 1 z z 3

R( 7, 7, 3)

9. Els punts P, Q, R i S de l’activitat anterior determinen un paral·lelogram? Raona’n la resposta.

Sí, perquè si els vectors PQ i RS són equipol·lents, també ho són els vectors PR i QS . Per tant, unint mitjançant segments els punts P, Q, R i S s’obté un quadrilàter que té els costats iguals i paral·lels dos a dos.

10. Dos vectors oposats tenen el mateix mòdul, la mateixa direc-ció i sentits contraris. Quina relació s’estableix entre els com-ponents de dos vectors oposats? Determina els components i el mòdul del vector oposat del vector v ( 1, 3, 4).

Els components respectius de dos vectors oposats són nombres reals que també son oposats.

11. Donats els punts A(2, 4, 5) i B(4, 6, t), calcula els valors de t sabent que |AB| 3.

AB

AB

AB

L’activitat té dues solucions possibles.

12. Considera un punt P i el vector QR (2, 3, 5). Si les coor-denades de Q són (1, 2, 1) i el vector posició del punt P és equipol·lent al vector QR , troba les coordenades dels punts P i R.

Les coordenades del punt P són (2, 3, 5).

Anomenem (x, y, z) les coordenades del punt R. Es compleix la igualtat:

(2, 3, 5) (x 1, y 2, z 1)

2 x 1 x 33 y 2 y 1

5 z 1 z 4

Les coordenades del punt R són (3, 1, 4).

13. Donats els vectors a (2, 4, 5), b ( 5, 7, 1) i c ( 5, 2, 3), troba els components dels vectors:

a) a b c

a b c

b) 3a 2b 2c

a b c

c) ( a 3b 3c )

a b c

d) a (c b )a c b a c b

Page 96: MATEMÀTIQUES - · PDF fileMatemàtiques 2 · Batxillerat · Solucionari No és permesa la reproducció total o parcial d’aquest llibre, ni el seu tractament informàtic, ni la

96 SOLUCIONARI DEL LLIBRE DE L’ALUMNE6

14. Si a és un vector de l’espai i k, h R, demostra que es verifica:

a)

b) a a

a

a

15. Els vectors p ( 2, 3, 1) i q (4, 6, 2), tenen la mateixa direcció? I el mateix sentit? Quina relació hi ha entre els seus mòduls?

Tenen la mateixa direcció i sentit contrari, ja que es verifica que

q 2p , o, el que és el mateix, p q . Per tant:

qq p p

16. Determina el vector unitari en la direcció i sentit de cadas-cun dels vectors següents:

a)

b)

u

b

b

c)

u

c

c

d)

u

d

d

e)

e

e

u

f)

u

17. Si el vector v és unitari, quins valors pot

tenir v3?

v

18. Si s a b , es verifica s a b .

a) Interpreta geomètricament aquesta desigualtat.

Si i tenen direcció diferent, els segments que representen els vectors , i determinen un triangle. Per tant, qualse-vol dels costats d’aquest triangle ha de ser més petit que la suma dels altres dos costats. En particular, s a b .

b) Indica en quin cas particular es verifica la igualtat.

Si i tenen la mateixa direcció i el mateix sentit, es com-pleix la igualtat s a b .

19. Demostra que el conjunt de totes les funcions reals que s’anul·len en el punt x0, amb les operacions habituals de suma de funcions i producte d’un nombre real per una fun-ció, té estructura d’espai vectorial.

Si f, g F, (f g) (x0) f(x0) g(x0) 0 0 0 f g F.

Si f F i R, ( f)(x0) f(x0) · 0 0 f F.

L’element neutre 0(x) 0 pertany a F, ja que 0(x0) 0.

Si f F, existeix l’element oposat f F, ja que ( f)(x0) f(x0) 0 0.

20. Considera els vectors següents:

v (2, 1, 3) i w ( 1, 3, 2)

Troba el vector x V3 que verifica 3x 2w v .

Page 97: MATEMÀTIQUES - · PDF fileMatemàtiques 2 · Batxillerat · Solucionari No és permesa la reproducció total o parcial d’aquest llibre, ni el seu tractament informàtic, ni la

97MATEMÀTIQUES 2 6

21. Considera un vector no nul v V3. Prova que el conjunt de vectors de V3 de la forma k · v , k R amb les operacions suma i producte per un nombre real té estructura d’espai vectorial. No oblidis que (V3, , ·) ja és un espai vectorial. Interpreta geomètricament el conjunt de vectors estudiat.

a)

b)

El conjunt S està format per tots els vectors que tenen la mateixa direcció.

22. Troba els nombres reals 1 i 2 que verifiquen la condició 1(2, 3) 2( 1, 2) (0, 0).

23. Donats els vectors v 1 ( 3, 1, 5), v 2 (2, 5, 3) i v 3 (4, 0, 1), troba en cada cas el vector que resulta de fer-ne les combinacions lineals següents:

a) v 1 3v 2 v 3

b) 2v 1 v 2 3v 3

c) 2(v 1 2v 2) 4v 3

d) 3v 1 (v 2 2v 3)

24. Expressa el vector v (2, 4, 1) en combinació lineal dels vectors v 1 (1, 2, 3), v 2 (4, 1, 2) i v 3 (1, 0, 0).

La solució del sistema és 1 1, 2 2 i 3 9.

25. Donats els vectors {v1, v2, …, vn}, indica raonadament si són certes o falses les afirmacions següents:

a) Cada vector v i és combinació lineal de tots ells.

Cert, perquè, per exemple:

b) El vector 0 també és combinació lineal d’aquests vectors.

Cert, perquè:

26. Donats dos vectors no nuls de V3, quina condició geomètrica s’ha de verificar perquè un d’ells sigui combinació lineal de l’altre? Quina relació cal que hi hagi entre els respectius components d’aquests dos vectors? Passa el mateix amb dos vectors no nuls de V2?

Els dos vectors han de tenir la mateixa direcció, és a dir, s’han de situar sobre la mateixa recta o sobre rectes parel·leles.

Els components respectius d’aquests vectors han de ser proporcionals:

Amb dos vectors no nuls de V2 passa exactament el mateix.

27. Esbrina si són linealment dependents o linealment indepen-dents els conjunts de vectors següents:

a) a 1 (2, 1, 3) i a 2

linealment dependents.

b) b 1 ( 1, 0, 2), b 2 (2, 0, 4) i b 3 (3, 1, 5)

linealment dependents.

c) c 1 (1, 2, 4), c 2 (0, 2, 1) i c 3 ( 1, 3, 0)

El sistema no té solució linealment independents.

Page 98: MATEMÀTIQUES - · PDF fileMatemàtiques 2 · Batxillerat · Solucionari No és permesa la reproducció total o parcial d’aquest llibre, ni el seu tractament informàtic, ni la

98 SOLUCIONARI DEL LLIBRE DE L’ALUMNE6

d) d1 (1, 3), d2 (2, 1) i d3 ( 4, 9)

1 2; 2 3; linealment dependents.

e) e 1 (1, 0, 0), e 2 (0, 1, 0), e 3 (0, 0, 1) i e

4 (2, 3, 5)

1 2; 2 3; 3 5 linealment dependents.

28. Existeix algun valor de k que faci que els vectors v 1 (3, 2, 2) i v 2 (6, 4, k) siguin linealment dependents? Justifica la resposta.

No, perquè independentment del valor de k, .

29. Troba p perquè els vectors següents: u1 (1, 2, 3),

u2 (3, 0, 4) i u

3 (2, 1, p) siguin linealment depen-dents. Per a quins valors de p aquests mateixos vectors són linealment independents?

Si p vectors linealment dependents.

Si p ≠ vectors linealment independents.

30. Justifica que si A, B i C són tres punts de R3 i els vectors AB i BC són linealment dependents, aleshores aquests tres punts estan alineats. Esbrina si els punts A(3, 4, 1), B(2, 1, 4) i C(0, 5, 10) es troben sobre la mateixa recta.

Si els vectors AB i BC són linealment dependents, vol dir que tenen la mateixa direcció. Per tant, els punts A, B i C han d’estar alineats.

Es compleix que els punts A, B i C se situen en

la mateixa recta.

31. Sabem que els punts P, Q i R estan alineats. Si P(1, 2, 3) i Q(4, 1, 5), determina les coordenades x i y del punt R sabent que la seva coordenada z és 9.

Anomenem R(x, y, 9).

Les coordenades del punt R són (10, 7, 9).

32. Comprova que els vectors v 1 (1, 2, 1), v 2 (2, 1, 0) i v 3 ( 1, 3, 1) són linealment independents. Expressa el vector w (5, 2, 3) en combinació lineal dels vectors v 1, v 2 i v 3.

Per tant,

33. Donats els vectors v (6, 4, 2) i w ( 3, 2, 1):

a) Troba el valor de que verifica v · w.

b) Comprova després que |v | | | · |w|.

Efectivament, es compleix que .

Page 99: MATEMÀTIQUES - · PDF fileMatemàtiques 2 · Batxillerat · Solucionari No és permesa la reproducció total o parcial d’aquest llibre, ni el seu tractament informàtic, ni la

99MATEMÀTIQUES 2 6

34. Indica raonadament si són certes o falses les afirmacions següents:

a) En V2, qualsevol conjunt format per dos vectors lineal-ment independents és base.

Certa.

b) En V3, qualsevol conjunt format per dos vectors lineal-ment independents és base.

Falsa.

c) Si la dimensió d’un espai vectorial V (dim V) és n, n 1 vectors d’aquest espai sempre són linealment dependents.

Certa.

d) Tant en V2 com en V3, una recta que contingui l’origen és un subespai vectorial de dimensió 1.

Certa.

e) En V3, un pla és un subespai vectorial de dimensió 2.

Falsa.

35. Esbrina si els vectors v 1 (1, 0, 3), v 2 (2, 1, 1) i v 3 (0, 2, 3) són base de V3.

Si la resposta és afirmativa, troba els components del vector v (3, 2, 4) en aquesta base.

Els vectors v 1, v 2 i v 3 són base de V3, ja que són linealment independents.

Els components de v en la base són

.

36. Considera un vector v V2 els components del qual en la base B {v 1, v 2} són (3, 2). Es demanen els components d’aquest mateix vector en la base B’ {u 1, u 2}, sabent que:

v 1 4u 1 u 2 i v 2 u 1 3u 2

Els components de v en la base són (10,3).

37. Els components del vector v en la base formada pels vectors v 1 (1, 2, 1), v 2 (2, 1, 0) i v 3 ( 1, 3, 1) són (2, 3, 4).

a) Expressa el vector v en combinació lineal dels vectors v 1, v 2 i v 3.

b) Troba els components de v en la base canònica.

38. Per a quins valors de t els vectors u 1 (3, 4, t), u 2 (1, 1, 2) i u 3 (0, 2, 1) formen una base de V3?

Els tres vectors han de ser linealment independents.

1 , 2

Els vectors u1, u2 i u3 són base si .

39. Els punts A(1, 2, 1), B(0, 0, 1), C( 2, 1, 3) i D(1, 1, 4) són coplanaris?

Els punts A, B, C i D no són coplanaris, perquè, per exemple els

vectors AB , AC i AD són linealment independents.

El sistema no té solució els tres vectors són linealment in-dependents.

Page 100: MATEMÀTIQUES - · PDF fileMatemàtiques 2 · Batxillerat · Solucionari No és permesa la reproducció total o parcial d’aquest llibre, ni el seu tractament informàtic, ni la

100 SOLUCIONARI DEL LLIBRE DE L’ALUMNE6

40. En la base B {(1, 1, 2), (3, 1, 4), (5, 2, 0)}, els com-ponents d’un vector v són (2, 3, 0). Determina els compo-nents d’aquest mateix vector en la base canònica i en la base B’ {(1, 0, 2), (2, 3 1), ( 2, 1, 0)}.

La solució del sistema és .

Per tant, els components del vector v en la base B són

.

41. Ens diuen que els components d’un vector v en la base B {(1, 0, 0), (0, 2, 0), (0, 0, 1)} són (2, 1, 2) i que els components d’aquest mateix vector en la base B’ {(2, 1, 0), (0, 0, 1), ( 1, 1, 0)} són (1, 0, 2). És això possible? Per què?

En el primer, els components del vector v en la base canònica són:

I en el segon cas, són:

Això no és possible, ja que si fos així, obtindríem dues ternes de components diferents per al mateix vector v en la base canòni-ca, i això no té cap sentit.

42. Justifica cadascuna de les afirmacions següents:

a) Si el producte escalar de dos vectors és positiu, l’angle que formen és agut i si és negatiu, l’angle és obtús.

b) Si dos vectors tenen la mateixa direcció i sentit, el pro-ducte escalar d’aquests dos vectors és igual al producte dels seus mòduls.

c) Si dos vectors tenen la mateixa direcció i sentit contrari, el producte escalar d’aquests dos vectors és igual al pro-ducte dels seus mòduls amb signe negatiu.

43. Donat el punt V(3, 5, 7), calcula les projeccions ortogonals del vector v OV sobre els eixos de coordenades.

Sobre l’eix 0X, 3; sobre l’eix 0Y, 5; sobre l’eix 0Z, 7.

44. Considera el vector v (3, 4, 5). Calcula els angles que for-ma amb els sentits negatius dels tres eixos de coordenades.

Angle que forma el vector (3, 4, 5) amb el vector ( 1, 0, 0):

Angle que forma el vector (3, 4, 5) amb el vector (0, 1, 0):

Angle que forma el vector (3, 4, 5) amb el vector (0, 0, 1):

45. v i w són dos vectors de mòduls respectius 2 i 4. Sabent que formen un angle de 60°, calcula k perquè el vector v kw sigui perpendicular a v .

46. Sabem que els vectors v (2, 1) i w (w1, w2) són orto-gonals.

a) Quina condició verifiquen els components del vector w?

b) Troba el vector w sabent que el seu mòdul és 5 i analitza les solucions obtingudes.

Hi ha dues solucions que són dos vectors oposats:

47. Resol les mateixes qüestions de l’activitat anterior, amb els vectors v (1, 2, 1) i w (w1, w2, w3) V3 i |w| 5. Co-menta les diferències que has trobat en relació amb l’ac-tivitat anterior i fes-ne una interpretació geomètrica. Et solucionaria el problema si et diguessin que el vector w es troba en el pla YZ? Per què?

Si v i w són perpendiculars:

Page 101: MATEMÀTIQUES - · PDF fileMatemàtiques 2 · Batxillerat · Solucionari No és permesa la reproducció total o parcial d’aquest llibre, ni el seu tractament informàtic, ni la

101MATEMÀTIQUES 2 6

En aquest cas, no és possible determinar el vector w que ens demanen, ja que el sistema:

té infinites solucions. Hi ha, per tant, infinits vectors w de mòdul 5 que són perpendiculars al vector v .

Si w es troba en el pla YZ, w1 0 i, per tant, sí que es pot deter-minar el vector. De la mateixa manera que en l’activitat anterior, s’obtenen dos vectors oposats:

Activitats finals

1. Determina les coordenades dels punts següents:

a) El punt P que es troba a l’eix Z a distància 5 de l’origen de coordenades en el sentit negatiu d’aquest eix.

P(0, 0, 5)

b) El punt Q que es troba en el pla XY a distància 6 de l’ori-gen de coordenades i tal que el vector 0Q forma un angle de 210° amb el sentit positiu de l’eix X.

c) El punt R situat en el pla XZ a distància 4 de l’origen de coordenades i tal que les projeccions ortogonals del vec-tor 0R sobre els eixos X i Z siguin iguals. Hi ha més d’un punt que verifiqui aquestes condicions?

Hi ha 4 punts que verifiquen la condició de l’enunciat:

2. Els components del vector v són ( 2, 4, 3). Es demana:

a) Les coordenades del seu extrem B si se situa el seu origen en el punt A(4, 3 , 6).

Anomenem B(x, y, z)

v AB ( 2, 4, 3) (x 4, y 3, z 6)2 x 4 x 2

4 y 3 y 1

3 z 6 z 9B(2, 1, 9)

b) Les coordenades del seu origen C si té l’extrem en el punt D(0, 4, 2).

Anomenem C(x, y, z)v CD ( 2, 4, 3) ( x, 4 y, 2 z)2 x x 2

4 4 y y 03 2 z z 5C(2, 0, 5)

c) Els components del vector unitari que té la mateixa direc-ció que el vector v i sentit contrari.

3. Troba una expressió que et permeti calcular les coordenades del punt mitjà del segment d’extrems els punts A(a1, a2, a3) i B(b1, b2, b3).

Anomenem M(x, y, z) aquest punt. Es verifica: AM MB

4. Troba les coordenades de divisió del segment d’extrems els punts P(3, 6, 3) i Q(9, 6, 0) en tres parts iguals.

Anomenem R i S els punts les coordenades dels quals hem de determinar:

R(r1, r2, r3) i S(s1, s2, s3).

6 3r1 9 r1 512 3r2 18 r2 2

3 3r3 9 r3 2

6 27 3s1 s1 712 18 3s2 s2 2

3 3s3 s3 1

Els punts són R(5, 2, 2) i S(7, 2, 1).

Page 102: MATEMÀTIQUES - · PDF fileMatemàtiques 2 · Batxillerat · Solucionari No és permesa la reproducció total o parcial d’aquest llibre, ni el seu tractament informàtic, ni la

102 SOLUCIONARI DEL LLIBRE DE L’ALUMNE6

5. Els punts A(3, 1, 2), B(5, 6, 4) i C(0, 4, 2) són tres vèr-texs consecutius d’un paral·lelogram. Determina les coorde-nades del quart vèrtex D i indica raonadament de quin tipus de paral·lelogram es tracta. Calcula’n l’àrea.

S’ha de verificar que AB DC . Anomenem D(x, y, z)

(2, 5, 6) ( x, 4 y, 2 z)2 x x 25 4 y y 16 2 z z 8

Les coordenades del quart vèrtex són D( 2, 1, 8).Es tracta d’un rombe, ja que el paral·lelogram en estudi té els quatre costats iguals i els seus angles no són rectes.

Es verifica:

D’altra banda, AB AD ≠ 0 els vectors AB i AD no són per-pendiculars.

Les diagonals d’aquest rombe mesuren:

i la seva àrea és:

6. Donats els vectors v i kv , k R, estableix la relació que hi ha entre els seus mòduls.

7. Considera dos vectors no nuls v 1 i v 2 V3. Prova que el con-junt de vectors de la forma 1v 1 2v 2 amb 1 i 2 R, amb les operacions suma i producte per un nombre real, és un subespai vectorial de V3. Fes-ne la interpretació geomètrica considerant les dues possibilitats:

Anomenem S aquest conjunt de vectors.

a) Que v 1 i v 2 tinguin la mateixa direcció.

Si v 1 i v 2 tenen la mateixa direcció, qualsevol vector de la forma 1v 1 2v 2 també tindrà aquesta direcció. Els vectors considerats se situen sobre la mateixa recta o sobre rectes paral·leles.

b) Que tinguin diferent direcció.

Si v 1 i v 2 tenen diferent direcció, aquests dos vectors deter-minen un pla en què es troben tots els vectors de la forma

1v 1 2v 2.

8. Troba t perquè el vector v (3, 4, 1) sigui combinació li-neal dels vectors v 1 (1, 4, 2) i v 2 (5, 2, t). Per a quins valors de t aquests tres vectors formen una base de V3?

La solució del sistema és .

Perquè els tres vectors formin una base de V3 han de ser lineal-

ment independents, i això succeeix si t ≠ .

9. Expressa el vector v (3, 4) en combinació lineal dels vec-tors u 1 (1, 0), u 2 (0, 1) i u 3 (2, 0). Té solució única el sistema que resulta? Com són entre ells els vectors u 1, u 2 i u

3? Formen una base de V2?

Aquest sistema té infinites solucions. Són de la forma:

1 3 2 2; 2 4; 3 3 ( 3 R)

Per a cada valor de 3 s’obté una solució particular del sistema.

Els vectors u1, u2 i u3 són linealment dependents, per tant, no formen base de V2.

10. Esbrina si els vectors v 1 (3, 5, 1), v 2 (1, 2, 1) i v 3 (0, 1, 1) són base de V3. En cas afirmatiu, troba els components del vector v (1, 3, 5) en aquesta base.

Els tres vectors són base, ja que són linealment independents.

Les components de v en aquesta base són .

Page 103: MATEMÀTIQUES - · PDF fileMatemàtiques 2 · Batxillerat · Solucionari No és permesa la reproducció total o parcial d’aquest llibre, ni el seu tractament informàtic, ni la

103MATEMÀTIQUES 2 6

11. Els components de a en la base B {u 1, u 2, u 3} són (2, 1, 3). Determina els components del vector a en la base B’ {v 1, v 2, v 3} sabent que:

v 1 u1 u3, v 2 u1 u2 u3 i v 3 2u

1 u3

Cal expressar els vectors de la base B en combinació lineal dels vectors de la base B’.

12. Si a (1, 2, 1), b (3, 0, 4) i c ( 2, 5, 1), determina:

a) a (2b 3c )

b) (a b ) c

c) a (b c )

d) L’angle format per a i b .

13. Donats els vectors v 1 (3, 4, 5) i v 2 (1, 2, 3), calcula la projecció ortogonal de v 2 sobre v 1.

Anomenem OP aquesta projecció:

14. Sabem que els vectors següents:

a (1, 2, 1) i b (b1, b2, b3)

són perpendiculars. Si el vector b es troba situat en el pla YZ i és unitari, troba’n els components. Interpreta les solucions obtingudes.

El vector b és del tipus b (0, b2, b3).

El sistema:

te dues solucions:

Hi ha dues solucions, són els vectors oposats:

15. Les coordenades de dos vèrtexs consecutius d’un paral-lelogram són A(2, 3, 1) i B(0, 4, 3). Si el centre d’aquest paral·lelogram es localitza en el punt P( 1, 2, 2), determi-na les coordenades dels altres dos vèrtexs i la mesura dels seus angles.

Anomenem C(c1, c2, c3) i D(d1, d2, d3) aquests dos vèrtexs que no coneixem, de manera que els vèrtexs A, B, C i D són consecutius.

Aleshores:

P és el punt mitjà entre A i C C( 4, 1, 3)

P és el punt mitjà entre B i D D( 2, 8, 1)

c1 4; c2 1; c3 3

d1 2; d2 8; d3 1

Per tant, Â

Page 104: MATEMÀTIQUES - · PDF fileMatemàtiques 2 · Batxillerat · Solucionari No és permesa la reproducció total o parcial d’aquest llibre, ni el seu tractament informàtic, ni la

104 SOLUCIONARI DEL LLIBRE DE L’ALUMNE6

16. Demostra que el baricentre d’un triangle de vèrtexs els punts A(a1, a2, a3), B(b1, b2, b3) i C(c1, c2, c3) està situat en el punt G de coordenades:

Anomenem G(x, y, z) el baricentre. Si M és el punt mitjà del costat AB, es compleix:

Avaluació

1. Donats els vectors (1, 1, 4), (2, 1, 3) i (1, 0, 0).

a) Determina si són vectors linealment dependents o inde-pendents.

Expressem en combinació lineal dels altres dos:

El sistema no té solució, els vectors són linealment inden-pendents.

b) Calcula la relació que hi ha d’haver entre els valors de a i b per tal que el vector (a, 1, b) sigui combinació lineal de i .

Si sumem les dues primeres equacions del sistema, obtenim:

.

Si sumem la segona multiplicada per 4 amb la tercera, obte-

nim .

Per tant,

2. Considera els vectors de V 3:

a) Troba l’únic valor de k per al qual aquests vectors no són una base de V 3.

Tres vectors de R3 no són base si no són linealment inde-pendents.

v v v

k k

3 1 1 2 2

1 21 2 1 3 1 3 4 2( , , ) ( , , ) ( , , )1 3

1 2

2 1 3 3

3 4 3

1 2

1 2

1 2

k k

k

Al mateix resultat s’arriba imposant que el rang de la matriu formada pels tres vectors no sigui 3:

b) Per a un valor de k diferent del que has trobat en l’apartat a), quins són els components del vector en la base ?

Si és una base, el vector té clara-ment els components (1, 1, 1) .

3. Un segment d’origen en el punt A( 1, 4, 2) i extrem en el punt B està dividit en cinc parts iguals mitjançant els punts de divisió A1, A2, A3 i A4. Si sabem que A2(1, 0, 2), quines són les coordenades de B?

Page 105: MATEMÀTIQUES - · PDF fileMatemàtiques 2 · Batxillerat · Solucionari No és permesa la reproducció total o parcial d’aquest llibre, ni el seu tractament informàtic, ni la

105MATEMÀTIQUES 2 7

4. Considera els punts A(1, 1, 1), B(2, 0, 1) i C(x, 2x, 1).

a) Calcula el valor de x que fa que els punts estiguin alineats.

b) Calcula el valor de x per tal que els vectors i siguin perpendiculars.

Han de verificar que 0

(1, 1, 0) . (x 1, 2x 1, 0) 0

x 1 2x 1 0 x 0

c) Si x 3, calcula l’angle entre i .

Substituint per x 3:

0,39

d) Calcula els valors de x per tal que el vector sigui uni-tari.

j Unitat 7. Matrius i determinants

Activitats

1. Escriu les matrius següents:

a) A (aij) on i 1, 2, 3, 4; j 1, 2, 3 per a aij ij

.

b) B (bij) d’ordre (2, 4), sabent que bij ( 1)i ( 1)j.

c) C (cij) de tres files i tres columnes per a cij ji.

2. Indica els vectors que determinen cadascuna de les ma-trius de l’activitat anterior.

a)

b)

c)

3. Escriu una matriu que representi 2 vectors de V2, i una altra que representi 4 vectors de V3. Indica l’ordre de cadascuna.

Resposta oberta. Per exemple:

La matriu A és d’ordre (2, 2) i la matriu B d’ordre (3, 4).

4. Escriu una matriu A d’ordre (3, 4).

Resposta oberta. Per exemple:

Page 106: MATEMÀTIQUES - · PDF fileMatemàtiques 2 · Batxillerat · Solucionari No és permesa la reproducció total o parcial d’aquest llibre, ni el seu tractament informàtic, ni la

106 SOLUCIONARI DEL LLIBRE DE L’ALUMNE7

a) Troba’n la matriu oposada i la matriu transposada. Com-prova que la matriu oposada de la transposada és igual a la matriu transposada de l’oposada.

b) Comprova que t(tA) A.

5. Donades les matrius següents:

troba els valors de x, y, z i t sabent que A B.

(1) 2x 4y(2) 3z t(3) x y z(4) 2y 3x 4

De (1) i (4) x 2, y 1, substituint a (3) z 1, i substituint a (2) t 3.

6. Escriu una matriu quadrada d’ordre 3 que sigui triangular superior. Indica els vectors de V3 que defineixen la matriu. Calcula’n la traça.

Resposta oberta. Per exemple:

(2, 0, 0), (3, 1, 0) i ( 1, 2, 1)

tA 2 1 1 2

7. Escriu una matriu quadrada d’ordre 4 que sigui alhora si-mètrica i antisimètrica. Hi ha moltes matrius que tinguin aquesta característica?

La matriu quadrada nul·la d’ordre quatre. Només aquesta.

8. Qualsevol matriu diagonal és simètrica? I antisimètrica? Justifica’n les respostes.

Sí, ja que si A és una matriu diagonal tA A A és simètrica.

No, perquè si és una matriu diagonal, els elements de la diago-nal principal no són zero, i per tant no pot ser antisimètrica.

9. Per què han de ser zero els elements de la diagonal principal d’una matriu antisimètrica?

Perquè sigui una matriu antisimètrica, els elements de la dia-gonal principal han de verificar que aii aii, d’on s’obté que aii 0.

10. Tenim la matriu:

a) Com és aquesta matriu?

És una matriu simètrica.

b) Troba la matriu transposada de la matriu oposada.

c) Com són entre elles la matriu que has trobat en l’apartat anterior i la matriu inicial?

Són oposades.

d) Es pot enunciar, en aquest sentit, alguna propietat general?

En una matriu simètrica, la transposada de l’oposada coinci-deix amb la matriu oposada.

Page 107: MATEMÀTIQUES - · PDF fileMatemàtiques 2 · Batxillerat · Solucionari No és permesa la reproducció total o parcial d’aquest llibre, ni el seu tractament informàtic, ni la

107MATEMÀTIQUES 2 7

11. Donades les matrius:

a) Comprova les propietats de la suma i del producte de ma-trius.

Suma:

A (B C) (A B) C

A O O A A

essent O la matriu quadrada nul·la d’ordre tres.

A ( A) ( A) A O, essent

A

A B B A

Producte:

(AB)C A(BC)

A(B C) AB AC

(A B)C AC BC

b) Prenent els valors k 2 i h 3, comprova les propietats del producte d’un nombre per una matriu.

2(A B) 2A 2B

[2 ( 3)]A A

2[( 3)A] 6A

1A A

c) Amb el mateix valor de k de l’apartat anterior, comprova les propietats de la transposició de matrius.

t(A B) tA tB

t(2A) 2(tA)

t(AB) (tB)(tA)

t(A2) (tA)2

Page 108: MATEMÀTIQUES - · PDF fileMatemàtiques 2 · Batxillerat · Solucionari No és permesa la reproducció total o parcial d’aquest llibre, ni el seu tractament informàtic, ni la

108 SOLUCIONARI DEL LLIBRE DE L’ALUMNE7

12. Amb les matrius de l’activitat anterior, calcula:

a) 2A 3B 4C

b) AB C

c) 3A CB

d) C(A 2B)

e) A3

f) (BC)2

13. Es diu que dues matrius A i B commuten quan AB BA. Com-prova, amb matrius quadrades d’ordre 3, que:

Respostes obertes. Per exemple:

a) Una matriu diagonal commuta amb qualsevol matriu del seu mateix ordre.

A B

La propietat és falsa.

AB

BA

No són iguals.

b) Si I és la matriu unitat, IA AI A.

IA AI

14. Siguin A i B dues matrius quadrades d’ordre 2, tals que:

A (aij); a11 a22 1, a12 2, a21 0

B (bij); b11 b22 a12, b12 b11 1, b21 0

a) Escriu les dues matrius.

A B

b) Les matrius A i B commuten?

Sí, ja que AB BA

c) Comprova que (A B) (A B) A2 B2. Per què creus que passa això? Justifica’n la resposta.

(A B)(A B) A2 B2

Si commuten, vol dir que AB BA, d’on tenim que:

(A B)(A B) AA BA AB BB A2 BA BA B2 A2 B2

15. Demostra que si A i B no commuten, aleshores

(A B)2 A2 2AB B2

Si no commuten, tenim que AB BA, i per tant BA AB 2AB, d’on:

(A B)2 (A B)(A B) AA BA AB BB A2 BA AB B2

Page 109: MATEMÀTIQUES - · PDF fileMatemàtiques 2 · Batxillerat · Solucionari No és permesa la reproducció total o parcial d’aquest llibre, ni el seu tractament informàtic, ni la

109MATEMÀTIQUES 2 7

16. Siguin A, B i C matrius quadrades d’ordre 3 i no nul·les:

a) És possible que AB doni la matriu nul·la d’ordre 3?

Sí, ja que el producte de dues matrius no nul·les pot donar la matriu nul·la.

b) Si es verifica que AB AC, podem assegurar que B C?

No, AB AC AB AC 0 A(B C) 0 i pot ser que A 0 i B C 0, d’on B C.

17. Donades les matrius:

calcula els productes AB i (tB)·(tA).

AB (tB)(tA)

18. Calcula els determinants d’ordre 2 següents:

a)

b)

c)

d)

19. Considera les matrius:

Calcula |A|, |B|, |C| i |A B C|.

|A| 19, |B| 26, |C| 2, |A B C| 17.

20. Comprova les igualtats següents:

a) (t 1)(t 5)

t2 6t 5 (t 1)(t 5)

b) (y x)(z x)(z y)

yz2 x2z xy2 x2y xz2 y2z

(y x)(z x)(z y) yz2 xz2 x2z y2z xy2 x2y

21. Resol les equacions:

a) x3 3x2 0 x 0, x 3

b) 6x 4 10 x 1

22. Considera els vectors de V3, ( 1, 2, 1), (2, 3, 2) i (0, 1, 3); i calcula:

23. Considerant la matriu A de l’exemple 7, calcula |A| i |A*|. Quina relació s’estableix entre els dos determinants?

A A

1 1 3

2 0 2

1 1 4

2 6 2

7 7 0

2 8 2

, *

|A| 14, |A*| 196, |A*| |A|2

Page 110: MATEMÀTIQUES - · PDF fileMatemàtiques 2 · Batxillerat · Solucionari No és permesa la reproducció total o parcial d’aquest llibre, ni el seu tractament informàtic, ni la

110 SOLUCIONARI DEL LLIBRE DE L’ALUMNE7

24. Sigui A (aij) una matriu quadrada d’ordre 3, definida per aij i j 1. Troba la matriu A* i comprova que t(A*) (tA)*.

A*

t(A*) (tA)*

25. Sigui B la matriu definida pels vectors (1, 2) i ( 1, 1) de V2.

a) Troba B*.

b) Les matrius B i B* commuten? Justifica’n la resposta.

No, perquè BB* i B*B

c) Calcula |B|, |B*|, |BB*| i |B*B|.

|B| |B*| 3, |BB*| |B*B| 9

26. Calcula el determinant

a) Per la regla de Sarrus.

10 2 12 15 9

b) Desenvolupant-lo pels elements de la segona fila.

( 2) 3 · 4 5( 1) 2 12 5 9

c) Desenvolupant-lo pels elements de la tercera columna.

2( 7) 5( 1) 14 5 9

27. Desenvolupa, per la columna o la fila més adequada, els de-terminants:

a)

3a columna 135

b)

3a fila 20

c)

2a fila 16

28. Comprova amb determinants d’ordre 3 i utilitzant la regla de Sarrus la primera, segona i tercera propietats. Comprova també la sisena i la desena.

Resposta oberta. Per exemple:

a)

b)

c)

f)

g)

Page 111: MATEMÀTIQUES - · PDF fileMatemàtiques 2 · Batxillerat · Solucionari No és permesa la reproducció total o parcial d’aquest llibre, ni el seu tractament informàtic, ni la

111MATEMÀTIQUES 2 7

29. Esbrina, emprant determinants, la dependència o indepen-dència lineal dels vectors següents:

a) (3, 1, 2), ( 1, 1, 2), (0, 2, 3)

linealment independents.

b) ( 1, 1, 4), (2, 1, 1), ( 2, 0, 2 )

linealment dependents.

c) (1, 0, 1), ( 2, 2, 1), (1, 2, 2 )

linealment dependents.

d) (1, 1, 1), (1, 1, 1), ( 1, 1, 1 )

linealment dependents.

30. Utilitzant determinants, troba els valors de a, per tal que els vectors de V3 següents siguin linealment dependents:

a) (1, 1, a), (2, 3, 2), (5, a, 8)

b) (1, 3, 2), (2, 1, a), (2, 13, 13)

31. Calcula el determinant següent, calculant només un deter-minant d’ordre 2:

Resposta oberta. Per exemple, deixant la 1a fila igual i canviant les altres dues per f2’ f2 f1 i f3’ f3 f1 i desenvolupant-lo pels elements de la segona columna nova queda:

32. Troba el rang de les matrius següents:

a)

b)

c)

d)

rang A 2, rang B rang C rang D 3

33. Considera la matriu .

Troba els valors de a per als quals rang A 3.

|A| 0 a2 3a 2 0 a 1, a 2

34. Sigui A una matriu quadrada regular. Demostra:

a) Si AX B, aleshores X A 1B

b) Si XA C, aleshores X CA 1

XA C (XA)A 1 CA 1 X(AA 1) CA 1 XI CA 1 X CA 1

c) Aplica-ho a les matrius:

Page 112: MATEMÀTIQUES - · PDF fileMatemàtiques 2 · Batxillerat · Solucionari No és permesa la reproducció total o parcial d’aquest llibre, ni el seu tractament informàtic, ni la

112 SOLUCIONARI DEL LLIBRE DE L’ALUMNE7

X A B1

513

1413

813

113

813

113

X CA 1

513

313

1113

413

1613

713

35. De les matrius:

a) Indica’n una que tingui inversa.

La matriu A.

b) Troba aquesta inversa. Fes-ne la comprovació.

A 1

1 0

12

14

AA A A1 1 1 0

0 1

36. Amb les matrius de l’activitat anterior, troba una matriu X tal que AX B C.

X A C B1

4 0 4

74

174

( )

37. Troba la inversa de la matriu:

i comprova-ho.

Calcula |A 1| i compara-ho amb |A|. Què observes?

|A 1|

38. Troba una matriu B, tal que BA (1 2 0), essent A la matriu de l’activitat anterior.

B

1011

611

911

, ,

39. Comprova que la inversa de

és A 1

Demostra que |A 1| |A| 1

40. Amb les matrius A3 1 20 1 11 1 2

i B0 0 10 1 01 0 0

, comprova:

(BA) 1 (A 1)(B 1) i (AB) 1 (B 1)(A 1)

Page 113: MATEMÀTIQUES - · PDF fileMatemàtiques 2 · Batxillerat · Solucionari No és permesa la reproducció total o parcial d’aquest llibre, ni el seu tractament informàtic, ni la

113MATEMÀTIQUES 2 7

Activitats finals

1. Donades les matrius:

troba, si existeix, la matriu X:

a) 2A B X C

X C B 2A

b) X 3B A C

X 2(A C 3B)

c) XB C

X C·B 1

d) AX B

X A 1·B

2. Sigui A troba una altra matriu quadrada B d’ordre 2 tal que AB 3A.

3. Calcula el determinant d’una matriu quadrada A d’ordre 3, definida per les condicions:

aij 3 (i j), aij 1 (i j)

4. Resol l’equació següent:

6x2 3x 0 x 0, x 1/2.

5. Demostra que les arrels del polinomi següent són 4, 8 i 12.

p(x) x3 112x 384 (x 4)(x 8)(x 12)

6. Calcula:

5 9

4 17

7. Troba les matrius inverses de:

Fes-ne la comprovació.

Page 114: MATEMÀTIQUES - · PDF fileMatemàtiques 2 · Batxillerat · Solucionari No és permesa la reproducció total o parcial d’aquest llibre, ni el seu tractament informàtic, ni la

114 SOLUCIONARI DEL LLIBRE DE L’ALUMNE7

C 1

112

56

712

16

23

16

16

13

16

AA 1 A 1A BB 1 B 1B CC 1 C 1C

8. De les matrius:

indica’n una que tingui inversa i troba-la.

Troba una matriu X tal que XA B 2C.

La matriu A, A 1 ;

X (2C B)A 1

9. Les matrius:

commuten? Justifica’n la resposta.

Sí, ja que AB BA

10. Decideix, segons els valors de k, el rang de la matriu:

k2 5k 6 0 k 2, k 3

Si k 2 i k 3, el rang és 3; i si k 2 o k 3, aleshores el rang és 2.

11. Troba el rang de les matrius següents:

rang A 3, rang B 2, rang C 2 i rang D 3.

12. Troba An si:

Si n és parell An I, si n és senar An A.

Avaluació

1. Un magatzem de rodes de vehicles de diferents tipus té l’es-toc de components (en centenars d’unitats) donat per la taula següent:

Pneumàtics Embellidors Llantes

Utilitaris 3,1 0,3 2,1

Berlines 1,6 1,1 0,6

Tot terrenys 0,9 0 0,2

La quantitat de quilos de matèria necessària per a cada com-ponent és:

Acer Cautxú

Pneumàtics 0,1 4,6

Embellidors 1 0,05

Tot Llantes 5 0

a) Calcula el total d’acer acumulat al magatzem.

1 646 kg d’acer.

b) Calcula el total de cautxú acumulat al magatzem.

2 583 kg de cautxú.

Page 115: MATEMÀTIQUES - · PDF fileMatemàtiques 2 · Batxillerat · Solucionari No és permesa la reproducció total o parcial d’aquest llibre, ni el seu tractament informàtic, ni la

115MATEMÀTIQUES 2 8

2. Calcula els valors de t per als quals rang A 2, essent:

At

4 1 02 3 03 2

Rang A 2?

12 2 10 0 rang A 2

Rang A 3?

12t 2t 10t 10t 0 t 0

Si t 0 rang A 2.Si t 0 rang A 3.

3. Considera les matrius:

Troba una matriu X que compleixi A · X A B.

La matriu X només pot ser quadrada d’ordre 2.

El problema es pot resoldre directament, fent

i efectuant A . X A i igualant a B.

S’obté el sistema d’equacions:

També es pot resoldre directament l’equació matricial:

AX A B AX B A X A-1 (B A).

La inversa de A és A 1

Ara, X A 1 (B A)

4. a)Mitjançant el càlcul de determinants, dedueix la depen-dència o independència lineal dels vectors:

(1, 3, 2), ( 3, 1, 1) i (1, 1, 1).

D ( , , ) 0

són linealment dependents.

b) Aplicant determinants, calcula els valors de m per tal que els vectors (m, 3, 2), (2, 0, m) i ( 3, 1, 1) siguin lineal-ment independents.

D m2 9m 10

m2 9m 10 0 m 1, m 10

m 1, m 10 són linealment independents.

j Unitat 8. Sistemes d’equacions

Activitats

1. Esbrina si (0, 1, 2) és la solució d’algun dels sistemes se-güents:

Substituim x 0, y 1 i z 2 en cada una de les equacions de cada sistema. Si es verifiquen les tres alhora, és solució:

a)

No

b)

c)

d)

No

2. Aplica les propietats de l’equivalència de sistemes fins a ar-ribar-ne a un d’esglaonat, per trobar la solució dels sistemes següents:

a)

Page 116: MATEMÀTIQUES - · PDF fileMatemàtiques 2 · Batxillerat · Solucionari No és permesa la reproducció total o parcial d’aquest llibre, ni el seu tractament informàtic, ni la

116 SOLUCIONARI DEL LLIBRE DE L’ALUMNE8

x y z

y

z

x

y

z

0

5 1

3 3

4515

1

b)

x y z

x z

z

x

y

z

2

1

9 9

0

1

1

3. Aplica el mètode de Gauss per resoldre, si és possible, els sistemes següents. Explica en cada cas de quin tipus de sis-tema es tracta.

a)

Posem la segona equació en primer lloc i dividim la tercera per 2 per aplicar Gauss:

el sistema és incompatible.

b)

Posar la segona equació en primer lloc. Es pot esquematitzar el procés representant les files de les successives matrius: F1, F2, F3 F1, F ’2 F2 3F1, F ’3 F3 2F1

F1, F ’2, F3’’ F ’3 2F ’2 Compatible determinat: (5, 7, 6)

c)

Posar la segona equació en primer lloc.

F1, F2, F3 F1, F ’2 F2 3F1, F ’3 F3 2F1

F1, F ’2, F3’’ F ’3 2F ’2 Compatible determinat: ( 3, 3, 0).

d)

F1, F2, F3 F1, F ’2 F2 F1, F ’3 F3 F1

F1, F ’2, F3’’ F ’3 F ’2

Compatible determinat: , , .

4. Si en un sistema de tres equacions i tres incògnites subs-titueixes una equació per la que resulta de sumar les tres, obtens un sistema equivalent? Raona la teva resposta.

Sí. La suma de les tres equacions és una combinació lineal d’elles i el sistema que en resulta és equivalent al donat.

5. Escriu dues equacions lineals amb tres incògnites. Afegeix una tercera equació de manera que el sistema format per aquesta i les altres dues sigui compatible indeterminat.

Construïm un sistema de manera que la tercera equació sigui combinació lineal de les dues primeres:

D’aquesta manera hem aconseguit un sistema compatible inde-terminat. Ara canviem el terme independent de la tercera equa-ció, i així obtenim un sistema incompatible.

6. Resol els sistemes següents en cas que siguin compatibles:

a)

Posar la segona equació en primer lloc:

F1, F2, F3 F1, F ’2 F2 3F1, F ’3 F3 F1 Compatible inde-

terminat ja que F ’2 F ’3. Les solucions expressades en funció de z, són:

x , y , z.

b)

Posar la tercera equació en primer lloc:

F1, F2, F3 F1, F ’2 F2 2F1, F ’3 F3 3F1 F1, F ’2, F3’’ F ’3 F ’2

Compatible determinat: , , 1

Page 117: MATEMÀTIQUES - · PDF fileMatemàtiques 2 · Batxillerat · Solucionari No és permesa la reproducció total o parcial d’aquest llibre, ni el seu tractament informàtic, ni la

117MATEMÀTIQUES 2 8

c)

Per reducció. Compatible determinat: x 37 i y 26

d)

Per reducció. Compatible determinat: x i y

7. Aplica el teorema de Rouché als sistemes següents i troba la solució dels que siguin compatibles.

a)

|M| 4, rang M rang M’ 3, per tant el sistema és compa-tible determinat. Es pot resoldre per Gauss: (5, 2, 4).

b)

|M| 11, rang M rang M’ 3, per tant el sistema és compatible

determinat. Es pot resoldre per Gauss: , , .

c)

|M| 63, rang M rang M’ 3, per tant el sistema és com-patible determinat. Es pot resoldre per Gauss: (5, 4, 1)

d)

|M| 12, rang M rang M’ 3, per tant el sistema és compati-

ble determinat. Es pot resoldre per Gauss: , , .

8. Comprova que aquests sistemes són incompatibles:

a)

|M| 0. Hi ha un determinant de segon ordre diferent de 0, per tant, rang M 2 i rang M’ 3 ja que el determinant que

s’obté en orlar amb la columna de termes independents és diferent de 0. El sistema és incompatible.

b)

rang M 2 i rang M’ 3. Igual que l’apartat anterior.

9. Contesta veritat o fals a cadascuna de les afirmacions següents:

a) Un sistema de tres equacions i tres incògnites és sempre compatible determinat.

És fals. Depèn dels rangs de les matrius M i M’.

b) Un sistema compatible indeterminat té només dues solucions.

És fals. Si és indeterminat té infinites solucions.

c) La matriu del sistema de dues equacions i tres incògnites és de rang 3.

És fals. Com a màxim és de rang 2, ja que només es pot con-siderar un determinant de segon ordre.

d) Un sistema és incompatible quan té més equacions que incògnites.

És fals. Que sigui incompatible només depèn dels rangs de les matrius.

e) En un sistema compatible determinat de tres equacions amb tres incògnites, la matriu ampliada és de rang 3.

És veritat. Coincideix amb el rang de la matriu.

10. Escriu els sistemes següents en forma matricial i troba’n la solució, si són compatibles, utilitzant la matriu inversa:

a)

|M| 45, per tant existeix la matriu inversa i podem trobar la solució:

Solució: (2, 1, 0)

Page 118: MATEMÀTIQUES - · PDF fileMatemàtiques 2 · Batxillerat · Solucionari No és permesa la reproducció total o parcial d’aquest llibre, ni el seu tractament informàtic, ni la

118 SOLUCIONARI DEL LLIBRE DE L’ALUMNE8

b)

|M| 13, per tant existeix la matriu inversa i podem trobar la solució:

La solució és (1, 2, 3).

11. Escriu un sistema de dues equacions amb tres incògnites. Posa’l en forma matricial. Té inversa, la matriu del siste-ma? És un sistema compatible i determinat? Raona les te-ves respostes.

Per exemple: En notació matricial:

La matriu M és d’ordre (2, 3), no és quadrada i no té inversa. No es pot resoldre el sistema per aquest mètode. El sistema és compatible indeterminat. Les dues matrius són de rang 2.

12. Escriu en forma matricial el sistema:

El pots resoldre utilitzant la matriu inversa? Raona la teva resposta.

i |M| 0, per tant, M no té inversa i no es pot resoldre el sistema per aquest mètode.

13. Resol el sistema següent en forma matricial:

La solució és: x i y .

14. Comprova que els sistemes següents són de Cramer i troba’n la solució utilitzant aquest mètode:

Per comprovar si els sistemes següents són de Cramer, calculem el determinant de la matriu del sistema. Si és diferent de 0, es pot aplicar aquest mètode. Calculem els determinants correspo-nents a cada incògnita, tot i que per trobar la tercera incògnita, també podem substitur les altres dues en una de les equacions.

a)

4, x 12, y 4 i z 4; x 3; y 1;

z 1. La solució és (3, 1, 1).

b)

8, x 40, y 24 i z 16. La solució és: (5, 3, 2).

c)

5, x 5, y 0 i z 5. La solució és: ( 1, 0, 1).

d)

2, x 5, y 3 i z 0. La solució és: , , 0 .

e)

11, x 96 i y 10. La solució és: , .

Page 119: MATEMÀTIQUES - · PDF fileMatemàtiques 2 · Batxillerat · Solucionari No és permesa la reproducció total o parcial d’aquest llibre, ni el seu tractament informàtic, ni la

119MATEMÀTIQUES 2 8

f)

1, x 5 i y 3. La solució és: (5, 3).

15. Considera el sistema següent i mira si el pots resoldre pel mètode de Cramer.

Si es considera el sistema

es pot resoldre per Cramer ja que 7 ≠ 0.

La solució és: z, z, z .

16. Raona, i resol en cas que sigui possible, els sistemes se-güents pel mètode de Cramer.

a)

26. Es pot resoldre per Cramer. x y z 0 per tenir una columna de zeros. És compatible determinat i la solució és la trivial: (0, 0, 0).

b)

0 . Es pot considerar el sistema:

Compatible indeterminat i la solució: ( , , ).

17. Resol els sistemes homogenis següents:

a)

6, x y z 0.

Compatible determinat: (0, 0, 0)

b)

0.

Compatible indeterminat: ( , , )

c)

2, x y z 0.

Compatible determinat: (0, 0, 0)

d)

0.

Compatible indeterminat: ( , 4 , )

e)

24, x y z 0.

Compatible determinat: (0, 0, 0).

18. Estudia la compatibilitat dels sistemes següents i resol els que siguin compatibles.

a)

|M| 0. Rang M rang M’ 2 i el sistema és compatible inde-terminat i es pot resoldre per Gauss. Les solucions es poden expressar: (9 4 , , 3 3 ).

b)

|M| 0. Rang de M 2 i rang M’ 2. El sistema és compatible indeterminat.

Solucions 3

719 3

7, ,

c)

Page 120: MATEMÀTIQUES - · PDF fileMatemàtiques 2 · Batxillerat · Solucionari No és permesa la reproducció total o parcial d’aquest llibre, ni el seu tractament informàtic, ni la

120 SOLUCIONARI DEL LLIBRE DE L’ALUMNE8

|M| 12. Rang M rang M’ 3. El sistema és compatible determinat. Es pot resoldre per Cramer perquè

|M| 12; x 36; y 24 i z 24.

La solució és ( 3, 2, 2)

d)

|M| |M’| 0. Rang M rang M’ 2 i el sistema és compati-ble indeterminat i es pot resoldre per Gauss. Les solucions es poden expressar: ( 5 7 , 3 2 , ).

e)

|M| 3. Rang M rang M’ 3. El sistema és compatible determinat. La solució és: (2, 2, 3).

f)

|M| 5. Rang M rang M’ 2. El sistema és compatible deter-minat. La solució és: ( 2, 3).

19. Discuteix els sistemes següents segons els valors del parà-metre k:

a)

Si fem |M| 0 k 3. Per a aquest valor, rang M rang M’ 2 i el sistema és compatible indeterminat. Per a k ≠ 3 és compatible determinat.

b)

Si fem |M| 0 k 2. Per a aquest valor, rang M rang M’ 2 i el sistema és compatible indeterminat. Per a k ≠ 2 és compatible determinat i té la solució trivial.

c)

Si fem |M| 0 k 0. Per a k 0 el sistema és incompatible ja que rang M 2 i rang M’ 3. Per a k ≠ 0 és compatible determinat.

d)

Si fem |M | 0 k 6. Per a k 6 el sistema és in-compatible ja que rang M 2 i rang M’ 3. Per a k ≠ 6 és compatible determinat.

e)

Si fem |M| 0 k 8. Per a k 8 el sistema és incompatible ja que rang M 2 i rang M’ 3. Per a k ≠ 8 és compatible determinat.

f)

Només serà compatible determinat si rang M rang M’ 2. Això implica que |M’| 0 k 2 ± . Només és compa-tible determinat per a aquests dos valors de k. Per a valors diferents d’aquests és incompatible.

20. En una granja hi ha porcs, vaques i cavalls, en total 54 ani-

mals. El nombre de vaques representa del nombre de

porcs, i el de cavalls, del de vaques. Quants animals hi ha

de cada classe a la granja?

x: nombre de porcs; y: nombre de vaques; z: nombre de cavalls.

Es pot resoldre per substitució. La solució és 24 porcs, 18 va-ques i 12 cavalls.

21. La suma de les edats de tres persones és 100 anys. Troba l’edat de cadascuna d’elles si saps que la del mig té 10 anys més que la més petita, i que la més gran té tants anys com les altres dues juntes.

x: edat més gran; y: edat del mitjà; z: edat més petita.

Es pot resoldre per substitució. La solució és 50, 30 i 20 anys, respectivament.

Page 121: MATEMÀTIQUES - · PDF fileMatemàtiques 2 · Batxillerat · Solucionari No és permesa la reproducció total o parcial d’aquest llibre, ni el seu tractament informàtic, ni la

121MATEMÀTIQUES 2 8

22. En un nombre de tres xifres la suma d’aquestes és 10. La xifra de les desenes és 3 i quan s’inverteix l’ordre d’aquestes xifres, s’obté un altre nombre que excedeix el primer en 495 unitats. Troba aquest nombre.

xyz 100x 10y z

El nombre és 136.

23. L’edat d’en Pere és el doble de l’edat de la Maria. Fa 7 anys la suma de les edats era igual a l’edat actual d’en Pere. Troba les dues edats.

x: edat d’en Pere, y: edat de Maria

Solució: 28 i 14 anys, respectivament.

24. Les edats d’una nena, el seu pare i la seva àvia sumen 100 anys. Calcula aquestes edats sabent que la diferència entre l’edat del pare i la de la seva filla és la meitat de l’edat de l’àvia i que 14 vegades l’edat de la nena és el doble de l’edat del pare.

x: edat de la nena; y: edat del pare; z: edat de l’àvia.

Solució: 5, 35 i 60 anys, respectivament.

25. D’un nombre de tres xifres sabem que:

a) Sumant la xifra de les centenes amb la de les unitats s’obté la xifra de les desenes.

b) Les tres xifres sumen 10.

c) Si s’inverteix l’ordre de les xifres, s’obté un altre nombre 297 unitats major.

Calcula el nombre.

xyz 100x 10y z

El nombre és 154.

Activitats finals

1. Estudia la compatibilitat dels sistemes següents i resol els que siguin compatibles.

a)

|M| 23 rang M rang M’ 3 i el sistema és compatible

determinat. La solució: (1, 2, 3).

b)

|M| 18 rang M rang M’ 3 i el sistema és compatible

determinat. La solució: (3, 3, 3).

c)

|M| 2 rang M rang M’ 3 i el sistema és compatible

determinat. La solució: (5, 7, 1).

d)

|M| 14 rang M rang M’ 3 i el sistema és compatible

determinat. La solució: (1, 5, 9).

e)

|M| 30 rang M rang M’ 3 i el sistema és compatible

determinat. La solució: 4, 6, .

f)

|M| 63 rang M rang M’ 3 i el sistema és compatible

determinat. La solució: , 4, 2 .

g)

|M| 95 rang M rang M’ 3 i el sistema és compatible

determinat. La solució: (2, 5, 3).

Page 122: MATEMÀTIQUES - · PDF fileMatemàtiques 2 · Batxillerat · Solucionari No és permesa la reproducció total o parcial d’aquest llibre, ni el seu tractament informàtic, ni la

122 SOLUCIONARI DEL LLIBRE DE L’ALUMNE8

h)

|M| 27 rang M rang M’ 3 i el sistema és compatible determinat; es pot resoldre per reducció o Gauss. La solució:

, ,

i)

|M| 0 rang M rang M’ 2 i el sistema és compatible indeterminat.

La solució: ( 2 , , )

j) El sistema següent està escrit en la forma matricial: AX B. Per resoldre’l cal trobar la matriu de les incògni-tes, la qual cosa implica el càlcul de la matriu inversa A 1 si existeix. També es pot resoldre multiplicant les matrius i escrivint el sistema en la forma més usual i resoldre’l per qualsevol dels mètodes.

Multipliquem les matrius i obtenim el sistema:

rang M rang M ‘ 3 compatible determinat.

Per Cramer: x 1; y 7 i z 6

2. Raona per què tots els sistemes següents són compatibles. Expressa la solució d’aquells que siguin indeterminats.

a)

b)

c)

Són sistemes homogenis i tenen com a mínim la solució tri-vial. c) és compatible indeterminat ja que |M| 0. Les solu-cions: (0, , ).

3. En una granja hi ha 1300 caps de bestiar distribuïts en tres corrals de manera que la relació entre el nombre d’animals

del primer corral i el del segon és i la relació entre el

nombre d’animals del segon i tercer és . Calcula quants animals hi ha en cada corral.

x: nombre de caps en el primer corral; y: nombre de caps en el segon; z: nombre de caps en el tercer.

Per substitució es resol fàcilment. La solució és 475, 450 i 375 caps de bestiar, respectivament.

4. Un constructor compra tres parcel·les i paga 150 €/m2, 180 €/m2 i 200 €/m2, respectivament. Calcula la superfície de cada una sabent que entre les tres fan 1870 m2, que el preu total de l’operació és de 336000 € i que el preu de la tercera parcel·la representa les tres quartes parts del preu de les altres dues juntes.

x: m2 de la 1a parcel·la, y: m2 de la 2a parcel·la, z: m2 de la 3a parcel·la

La solució és 500 m2, 650 m2 i 720 m2.

5. Considera el sistema:

Troba els valors de m pels quals el sistema no és de Cramer. Resol el sistema per aquest mètode quan m 1.

No és de Cramer si 0 m 3 i m 1.

Per a m 1, 8 i el sistema és compatible determinat.

La solució és , , 4 .

Page 123: MATEMÀTIQUES - · PDF fileMatemàtiques 2 · Batxillerat · Solucionari No és permesa la reproducció total o parcial d’aquest llibre, ni el seu tractament informàtic, ni la

123MATEMÀTIQUES 2 8

6. Discuteix el sistema següent segons el valor del paràmetre m. Expressa la solució general pel valor de m que el faci compatible indeterminat.

Fer |M| 0 m1 1 i m2 2

Per a m 1 rang M 1 i rang M ‘ 2 sistema incompatible.

Per a m 2 rang M rang M ‘ 2 sistema compatible indeterminat.

Es per resoldre per Gauss i s’obté:

7. Discuteix el sistema següent segons els valors de t i prova de resoldre’l quan sigui compatible:

Per a t 4, rang M rang M’ 2 i el sistema és compatible indeterminat.

Les solucions: , , .

Per a t ≠ 4 és incompatible ja que rang M 2 i rang M’ 3.

8. Un antiquari compra tres peces d’art per 2 milions d’euros. Confia a vendre-les amb uns guanys del 20%, del 50% i del 25%, respectivament, amb la qual cosa obtindria un bene-fici de 0,6 milions. Però en una subhasta ha aconseguit uns guanys del 80%, del 90% i del 85%, respectivament, fet que li ha representat un benefici de 1,7 milions. A quin preu va comprar cada peça?

x: preu de la 1a peça (milions €)y: preu de la 2a peça (milions €)z: preu de la 3a peça (milions €)

Solució: 0,5, 0,5 i 1 milions, respectivament.

9. Analitza’n la compatibilitat i resol el sistema següent. De-mostra que hi ha infinites solucions que tenen els tres va-lors de les incògnites positius.

Compatible indeterminat ja que rang M rang M’ 2. Una ex-

pressió de les solucions és: , , . Cal resoldre

les inequacions que donen les solucions en nombres positius. Per als valors de tals que 0 1/4 les infinites solucions són positives.

10. Per a quin valor de k el sistema és compatible? Troba’n la solució.

Resolem el sistema format per les 3 primeres equacions:rang M rang M ‘ 3 compatible determinat.

Per Cramer: 6; x 6, y 12, z 18

x 1; y 2; z 3

Substituim aquesta solució en la 4a equació:1 2 6 k

Per a k 3 el sistema és compatible.

11. Considera les equacions:

2x y z 0 3x 2y z 3

Escriu una tercera equació que, amb les dues anteriors, for-mi un sistema que sigui:

a) Compatible determinat.

Resoldre el sistema format per les dues equacions que és indeterminat i escriure una tercera equació que verifiqui una solució particular: 5x y 5z 9.

b) Compatible indeterminat.

Qualsevol combinació lineal de les dues equacions: 5x 3y 3.

c) Incompatible.

Com l’anterior però amb el terme independent diferent: 5x 3y 9.

12. Les tres xifres d’un nombre sumen 18. Si a aquest nombre li restem el que resulta d’invertir l’ordre de les seves xifres, s’obté 594. Troba aquest nombre si sabem que la xifra de les desenes és la mitjana aritmètica de les altres dues.

xyz 100x 10y z

Page 124: MATEMÀTIQUES - · PDF fileMatemàtiques 2 · Batxillerat · Solucionari No és permesa la reproducció total o parcial d’aquest llibre, ni el seu tractament informàtic, ni la

124 SOLUCIONARI DEL LLIBRE DE L’ALUMNE8

El nombre és 963.

13. Per la festa major, un noi va a tres espectacles diferents. El primer dia va dues vegades a l’espectacle X, una al Y i l’altra al Z, i es gasta 130 €. El segon dia va tres vegades al X i una al Y i es gasta 180 €. El tercer dia va un cop a cada espectacle i es gasta només 80 €. Quin era el preu de cada espectacle?

X, Y, Z: preus dels espectacles, respectivament.

Solució: X 50 €, Y 30 € i Z és gratis.

14. Troba l’edat de tres germans sabent que el triple de l’edat del primer menys el doble de l’edat del segon més l’edat del tercer fan 22 anys, l’edat del primer menys la del segon més el doble de la del tercer fan 8 anys, i el doble de la del primer més la del segon menys la del tercer fan 20 anys.

x, y, z: les tres edats.

Solució: 9, 3 i 1 anys, respectivament.

15. Troba la solució dels sistemes següents que siguin compatibles:

a)

|M| |M ‘| 0 Sistema compatible indeterminat ja que rang M rang M ‘ 2.

Prenem les dues primeres equacions:

y ; x 3 2 ; z 4 2x 3y 2

Solució: ( 3 2 , , 2 ).

b)

rang M 2 i rang M‘ 3 Sistema incompatible.

c)

rang M rang M‘ 2 Sistema compatible indeterminat.

Prenem les dues primeres equacions:

y , x 5 2 , z 3x 4 11 5

Solució: (5 2 , , 11 5 )

16. Discuteix els sistemes següents segons els valors del parà-metre :

a)

Si 3: rang M rang M‘ 1 sistema compatible indeterminat amb dos graus de llibertat.

Si ≠ 3 i ≠ 6 sistema compatible determinat.

Si 6: rang M rang M‘ 2 sistema compatible indeterminat amb un grau de llibertat.

b)

Per a 1 rang M rang M‘ 1 sistema compatible indeterminat amb dos graus de llibertat.

Per a 1 rang M 2, rang M‘ 3 sistema incompa-tible.

Per a ≠ 1 i ≠ 1 rang M rang M‘ 3 sistema com-patible determinat.

c)

|M| 0 1 1 i 2

Per a 1 1 rang M rang M‘ 2 sistema compatible indeterminat, amb un grau de llibertat.

Per a 2 rang M 2 i rang M‘ 3 sistema incom-patible.

Per a ≠ 1 i ≠ sistema compatible determinat.

Page 125: MATEMÀTIQUES - · PDF fileMatemàtiques 2 · Batxillerat · Solucionari No és permesa la reproducció total o parcial d’aquest llibre, ni el seu tractament informàtic, ni la

125MATEMÀTIQUES 2 8

Avaluació

1. Per a quin o quins valors del paràmetre real el sistema d’equacions seguient

és compatible i indeterminat? El determinant de la matriu de coeficients del sistema d’equa-cions val, per a cada , 10 14 4 2. Aquest determinant

només s’anul . la per a 1 o per a . En el cas que

1 surt un sistema incompatible, en el cas que el

sistema és compatible i indeterminat, en tots els altres casos el sistema és compatible determinat.

2. Donat el sistema d’equacions:

a) Afegeix-hi una equació lineal de manera que el sistema resultant sigui incompatible.

Per exemple 2x 3y z 0, que es contradiu amb la segona equació.

b) Afegeix-hi una equació lineal de manera que el sistema resultant sigui compatible indeterminat. Resol el sistema que s’obtingui.

Si afegim una equació que resulti, per exemple, de multiplicar la segona equació per 2: 4x 6y 2z 8. Aquesta equació no aporta cap informació nova. El sistema queda:

Per tant, x y 1 y 1 x

3x 2y z 5z 5 3x 2(1 x) 5 3x 2 2x 7 5x

La solució és (x, y, z) (x, 1 x, 7 5x) on x R.

3. Resol els sistemes següents:

a)

rang M 2 i rang M 3. És un sistema incompatible.

b)

Per Gauss:

És un sistema compatible indeterminat: (1, , 5 ).

4. (Curs 2003-04) En estudiar un sistema d’equacions lineal dependent del paràmetre k pel mètode de Gauss hem arribat a la matriu ampliada següent:

Discuteix el sistema en funció del paràmetre k.

Apliquem Rouché-Frobenius.Si l’únic determinant d’ordre 3 que podem extreure de la matriu és nul, rang A 2.

A (k 2) (k 1) 0 k 2; k 1

Per a k 2 i k 1 el rang A 2. Per a k 2 i k 1 el rang A 3.

Per a k 2 el rang A’ 3, el sistema és incompatible.

Per a k 1 el rang A’ 2, el sistema és compatible indetermi-nat.

Per a qualsevol altre valor el sistema és compatible determinat.

Page 126: MATEMÀTIQUES - · PDF fileMatemàtiques 2 · Batxillerat · Solucionari No és permesa la reproducció total o parcial d’aquest llibre, ni el seu tractament informàtic, ni la

126 SOLUCIONARI DEL LLIBRE DE L’ALUMNE9

j Bloc 3. Geometria

j Unitat 9. Equacions de rectes i plans

Activitats

1. Determina tres punts de la recta:

r: (x, y, z) (4, 2, 5) ( 3, 1, 2)

r: (x, y, z) (4, 2, 5) ( 3, 1, 2) 0 A(4, 2, 5) 1 B(1, 1, 3) 1 C(7, 3, 7)

2. Escriu les equacions paramètriques de la recta que passa pel punt P(3, 2, 4) i té la direcció del vector u (2, 3, 3).

3. Determina l’equació vectorial i les equacions contínues de la recta que passa per l’origen de coordenades i pel punt P( 1, 4, 2).

P v p OPO

( , , ) ( , , )( , , )

1 4 2 1 4 20 0 0

equació vectorial: (x, y, z) ( 1, 4, 2), R

equacions contínues:

4. Determina la recta que passa pels punts P(2, 0, 1) i Q(3, 1, 1), i comprova que conté el punt R(5, 3, 5).

5. Escriu l’equació vectorial de cadascuna de les rectes que de-terminen els eixos de coordenades.

L’eix OX: (x, y, z) (1, 0, 0)l’eix OY: (x, y, z) (0, 1, 0)l’eix OZ: (x, y, z) (0, 0, 1)

6. Un vector director d’una recta és u ( 2, 3, 1). Troba els components d’un altre vector director d’aquesta recta que sigui unitari.

7. Sigui u (3, 5, 2) un vector director d’una recta. Esbrina el valor que tenen v1 i v2 per tal que el vector v (v1, v2, 3) també sigui un vector director de la mateixa recta. Escriu el vector u com a combinació lineal del vector v .

8. Determina tres punts del pla següent:

: (x, y, z) (6, 1, 3) (2, 4, 5) μ(1, 0, 7)

(x, y, z) (6, 1, 3) (2, 4, 5) (1, 0, 7)

0 A(6, 1, 3)

0, 1 B(7, 1, 4)

1, 0 C(8, 5, 8)

9. Determina l’equació general del pla que té el vector u (2, 2, 5) com a vector orientador i que passa pels punts P(0, 2, 3) i Q(1, 1, 4).

Page 127: MATEMÀTIQUES - · PDF fileMatemàtiques 2 · Batxillerat · Solucionari No és permesa la reproducció total o parcial d’aquest llibre, ni el seu tractament informàtic, ni la

127MATEMÀTIQUES 2 9

10. Escriu les diferents equacions del pla que passa pel punt A(3, 1, 2) sabent que els vectors que determinen la seva orientació són u (2, 5, 3) i v (4, 1, 3).

Equació vectorial: (x, y, z) (3, 1, 2) (2, 5, 3) (4, 1, 3).

Equacions paramètriques:

Equació general: x y z 2 0

Equació canònica:

11. Demostra que el pla que té per equació general By Cz D 0 és paral·lel a l’eix OX.

By Cz D 0 y z

(x, y, z) (0, , 0) (1, 0, 0) (0, C, B), el vector e

1 (1, 0, 0) que determina la direcció de l’eix OX, és un vector

orientador del pla, per tant el pla és paral·lel a l’eix OX.

12. Comprova que les equacions vectorials següents:

(x, y, z) (1, 2, 3) (1, 1, 0) μ( 1, 0, 1)(x, y, z) (0, 0, 6) (0, 1, 1) (2, 1, 1)

són del mateix pla.

(x, y, z) (1, 2, 3) (1, 1, 0) ( 1, 0, 1)

(x, y, z) (0, 0, 6) (0, 1, 1) (2, 1, 1)

Com que dóna la mateixa equacio general, és el mateix pla.

13. Determina un punt i els vectors orientadors del pla que té per equació general:

x 3y 2z 5 0

14. Emprant l’equació general del pla, comprova que els punts P(1, 2, 1), Q(3, 1, 2), R(1, 1, 0) i S(0, 2, 1) són coplanaris.

S(0, 2, 1) 2 3 5 0 S és del pla que determinen els punts P, Q i R, per tant els quatre punts són coplanaris.

15. Troba dos vectors associats al pla següent:

3x 2y 2z 5 0 que siguin unitaris.

16. Troba l’equació general del pla que passa pel punt ( 3, 2, 1) i és perpendicular a una recta que té la direcció del vector u (0, 3, 1).

17. Donat el pla : 5x 3y 2z 8 0 i el punt P(3, 4, 2), troba l’equació de la recta que passa per P i és perpendicular al pla .

Page 128: MATEMÀTIQUES - · PDF fileMatemàtiques 2 · Batxillerat · Solucionari No és permesa la reproducció total o parcial d’aquest llibre, ni el seu tractament informàtic, ni la

128 SOLUCIONARI DEL LLIBRE DE L’ALUMNE9

18. Troba l’equació general i les equacions paramètriques del pla que passa pel punt (0, 0, 3) i és perpendicular a l’eix OZ.

P(0, 0, 3) 3 D 0 D 3

: z 3 0 equació general.

equacions paramètriques.

19. Donada la recta:

troba’n un punt i un vector director.

resolem el sistema:

i obtenim:

20. Escriu les equacions paramètriques i contínues de la recta de l’activitat anterior.

Equacions paramètriques:

Equacions contínues:

21. Troba un punt i un vector director de cadascuna de les rectes

s: 2x y 1 2z

P( 1, 0, 4), (1, 1, 0)

d’on s'obté:

Q (0, 1, 0), (1, 2, 1)

22. Escriu la recta r: (x, y, z) ( 3, 2, 4) (2, 2, 5) com a intersecció de dos plans.

Resposta oberta, per exemple:

23. Considera les dues equacions següents:

a) Digues per a quins valors de k aquestes dues equacions representen una recta.

Perquè determinin una recta, els dos plans no han de ser paral·lels. Considerant que són paral·les, tenim:

Per tant, si , els dos plans determinen una recta.

b) Per a k 1 és una recta? En cas afirmatiu, escriu-ne les equacions contínues.

k 112

sí, és una recta

Substituim per k 1:

obtenim:

no es poden escriure les equacions contí-nues perquè el 3r component del vector director és zero.

24. Donada la recta:

Page 129: MATEMÀTIQUES - · PDF fileMatemàtiques 2 · Batxillerat · Solucionari No és permesa la reproducció total o parcial d’aquest llibre, ni el seu tractament informàtic, ni la

129MATEMÀTIQUES 2 9

a) Determina’n un punt Q i un vector director u .

b) Escriu l’equació general del pla que passa pel punt

P( 4, 2, 3) i té per vectors orientadors u i PQ .

c) Comprova que és el mateix pla que hem obtingut a l’exemple 8.

Comparant les dues equacions generals, s’observa que efecti-vament és el mateix pla.

25. A partir de l’expressió del feix de plans, determina l’equació general del pla que passa per l’origen de coordenades i conté la recta següent:

x 1 z

escrivim el feix de plans:

(x z 1) (y 3z 2) 0

Imposem que passi per l’origen:

O(0, 0, 0) 2 0 2

2 (x z 1) (y 3z 2) 02(x z 1) y 3z 2 02x 2z 2 y 3z 2 0

: 2x y z 0

26. Escriu l’equació del feix de plans que contenen la recta que passa pels punts P(5, 7, 4) i Q(4, 3, 1). Comprova que el sistema definit per tres plans qualssevol del feix és compa-tible indeterminat amb un grau de llibertat i que la solució d’aquest sistema és la recta PQ.

Resposta oberta, per exemple:

(4x y 13) (5x z 21) 0

és el sistema.

Efectivament és un sistema compatible indeterminat amb un grau de llibertat, ja que rang M rang M’ 2.

La resolució del sistema dóna: (x, y, z) (5, 7, 4) (1, 4, 5) que és la recta que passa per P i Q.

Activitats finals

1. Els punts P(p1, p2, p3), Q(q1, q2, q3) i R(r1, r2, r3) defineixen un pla si no estan alineats. Justifica que la seva equació general es pot expressar així:

Considerem els vectors orientadors:

u PQ (q1 p1 , q2 p2 , q3 p3) i

v PR (r1 p1 , r2 p2 , r3 p3)

Amb el punt P(p1, p2, p3) i el punt X(x, y, z), que és un punt

qualsevol del pla, tenim el vector PX (x p1, y p2, z p3),

que és un vector del pla, per tant el vector PX és combinació

lineal dels vectors u i v , és a dir, els vectors PX , u i v són

linealment dependents, d’on tenim que D(PX , u , v ) 0.

2. Demostra que el pla d’equació Ax By 0 conté l’eix OZ.

Ax By 0 (x, y, z) ( B, A, 0) (0, 0, 1)

L’equació vectorial indica que el vector e 3 (0, 0, 1), que és el vector que determina l’eix OZ, és un vector orientador del pla, per tant el pla és paral·lel a l’eix OZ, a més el pla passa per l’origen, ja que D 0, aleshores el pla passa per l’eix OZ.

3. Escriu l’equació canònica i general del pla que passa pels punts P( 3, 0, 0), Q(0, 4, 0) i R(0, 0, 5). Indica’n dos vectors orientadors.

Equació canònica:

Page 130: MATEMÀTIQUES - · PDF fileMatemàtiques 2 · Batxillerat · Solucionari No és permesa la reproducció total o parcial d’aquest llibre, ni el seu tractament informàtic, ni la

130 SOLUCIONARI DEL LLIBRE DE L’ALUMNE9

Equació general: 20x 15y 12z 60 0

u PQ (3, 4, 0) i v PR (3, 0, 5)

4. Donades les equacions:

i R

a) Comprova que es tracta d’un pla.

De les equacions paramètriques es dedueix:

u (3, 3, 0) i v ( 3, 2, 2) que són dos rectes linealment independents, per tant determinen un pla.

b) Troba’n els punts de tall amb els eixos de coordenades.

P(3, 0, 0) de les equacions paramètriques, és el punt on talla l’eix OX.

Per trobar els altres dos punts:

talla l’eix OY.

talla l’eix OZ.

5. Justifica la certesa o la falsedat de cadascuna de les afirma-cions següents:

a) Els plans següents: Ax By Cz D 0 i (x, y, z) (x0, y0, z0) (A, B, C) μ(v1, v2, v3) són per-pendiculars.

Veritat. Un dels vectors orientadors del segon pla és el vector associat del primer.

b) La recta (x, y, z) (x0, y0, z0) (u1, u2, u3) i el pla Ax By Cz D 0 són perpendiculars si els vectors u (u1, u2, u3) i v (A, B, C) són linealment dependents.

Veritat. Si u i v són linealment dependents, vol dir que tenen la mateixa direcció, i per tant la recta té la direcció d’un vector perpendicular al pla, aleshores, la recta és per-pendicular al pla.

c) Si u és un vector director d’una recta; n , un vector associat a un pla i u · n 0, la recta i el pla són perpendiculars.

Fals. Si u · n 0, o bé la recta i el pla són paral·lels o el pla conté la recta.

d) Si A · B · C ≠ 0 i A · B · C · D 0, el pla: Ax By Cz D 0 talla els eixos de coordenades només a l’origen.

Veritat. Es dedueix que D 0, per tant el pla passa pel l’origen.

6. Donats els punts P(1, 2, 3), Q(1, 1, 1) i R(2, 0, 4), busca les equacions paramètriques de la recta que passa pel punt P i és perpendicular al pla que passa pels tres punts.

7. Determina els valors de a que fan que els dos plans d’equacions:

ax y z 3 0 i (a 2)x ay az 5

siguin paral·lels.

perquè siguin paral·leles:

d’on a2 a 2 a2 a 2 0 a1 2, a2 1

8. Troba l’equació canònica del pla que passa pel punt P( 3, 1, 4) i és perpendicular als plans:

2x 5y 3z 5 0 i 7x y 3z 21

són els vectors

orientadors.

x y z614

615

6111

1

Page 131: MATEMÀTIQUES - · PDF fileMatemàtiques 2 · Batxillerat · Solucionari No és permesa la reproducció total o parcial d’aquest llibre, ni el seu tractament informàtic, ni la

131MATEMÀTIQUES 2 9

9. Indica les condicions que han de complir en cada cas els coe ficients A, B, C i D de l’equació general del pla, de manera que aquest:

a) sigui paral·lel al pla determinat per OX i OZ.

b) talli els tres semieixos de coordenades positius en punts equidistants de l’origen.

Punt de tall amb l’eix OX: y z 0

Punt de tall amb l’eix OY: x z 0

Punt de tall amb l’eix OZ: x y 0

Per tant:

10. Escriu les equacions contínues de la recta que passa pel punt P(2, 3,5) i té la mateixa direcció que la recta:

Ho escrivim en forma de sistema:

resolent, s’obté:

Podem prendre com a vector director de la recta: u ( 12, 2, 5)

11. Troba l’equació general del pla que passa pel punt P( 4, 2, 3) i conté la recta d’equació:

(x, y, z) (1, 2, 2) (2, 1, 2)

P( 4, 2, 3)(x, y, z) (1, 2, 2) (2, 1, 2) Q(1, 2, 2), u (2, 1, 2)

PQ v

v

u

12. Troba l’equació general del pla que conté la recta:

considerant que un dels vectors que determina l’orientació del pla és un vector director de la recta:

13. Donats el pla : x y 2z 1 0 i la recta:

escriu l’equació vectorial del pla que és perpendicular al pla i conté la recta r.

escrivim el sistema i el resolem:

Page 132: MATEMÀTIQUES - · PDF fileMatemàtiques 2 · Batxillerat · Solucionari No és permesa la reproducció total o parcial d’aquest llibre, ni el seu tractament informàtic, ni la

132 SOLUCIONARI DEL LLIBRE DE L’ALUMNE9

14. Considera la recta r que passa pel punt de coordenades (1,1,2) i té el vector de components (1,1,1) com a vec-tor director. Considera el vector v de components (1,1,a) i digues si per a algun valor de a existeix un pla que conté r i és perpendicular a v . En cas afirmatiu, escriu l’equació cartesiana del pla.

r: (x, y, z) (1,1,2) (1,1,1)

han de ser ortogonals, per tant:

Per tant, tenim:

n v x y x D( , , )1 1 2 2 0

P(1, 1, 2) 1 1 4 D 0 D 2x y 2z 2 0 x y 2z 2

Avaluació

1. Determina l’equació del pla que conté a la recta: r:

i passa per l’origen de coordenades. De la recta tenim: i .

2. Considera els punts de l’espai A(1,1,2), B(0,1,1) i C(k,1,5).

a) Troba l’equació de la recta que passa per A i B.

Si fem servir A com a punt base i el vector com a vector director de la recta, la seva equació contínua és:

La general serà, doncs,

b) Per a quins valors de k els punts A, B i C formen un triangle?

Per no formar un triangle, els tres punts han d’estar alineats. Per tant, el punt C ha de pertànyer a la recta AB:

Així, la condició per tal que A, B i C formin un triangle és que .

3. Considera els plans d’equacions:

i

a) Hi ha algun valor del paràmetre a per al qual els plans i no tenen intersecció?

Dos plans són paral.lels si els coeficients dels vectors direc-tors són proporcionals. En aquest cas:

.

Això passa si a 2.

A més a més, si

els plans són coincidents.

Aquesta última condició no es compleix perquè

b) Calcula un vector director de la recta que s’obté quan es fa la intersecció de i per al valor del paràmetre a 0.

Per a a 0:

Resolem el sistema considerant z com un paràmetre:

L’equació vectorial de la recta és:

La recta passa pel punt P(7, 2,0) i té la direcció del vector

.

Page 133: MATEMÀTIQUES - · PDF fileMatemàtiques 2 · Batxillerat · Solucionari No és permesa la reproducció total o parcial d’aquest llibre, ni el seu tractament informàtic, ni la

133MATEMÀTIQUES 2 10

4. Un quadrat de l’espai té tres dels seus vèrtexs consecutius situats en els punts de coordenades enteres P (3, 2, 4), Q(a, 1, a 1)i R(2, 3, 0).

a) Tenint en compte que els vectors i han de ser perpendiculars, calcula el valor del nombre enter a.

QP a a

QR a a

QP

( , , )

( , , )

3 1 3

2 2 1

QR a a a a

a a

( , , ) ( , , )3 1 3 2 2 1 0

2 72 5 0104

1a ai

Com que ens demanen un valor enter a 1.

b) Calcula l’equació del pla que conté aquest quadrat.

Prenent el punt i els vectors i

, podem escriure l’equació vectorial del pla:

per passar a l’equació general:

c) Calcula el quart vèrtex d’aquest quadrat.

Hem de trobar el punt S(x, y, z).

El punt que compleix aquest sistema d’equacions és S(4, 4, 2).

d) Calcula l’àrea d’aquest quadrat.

L’àrea d’un quadrat és el quadrat del costat (l2). La longitud d’un costat és la distància entre dos dels seus vèrtex conse-cutius:

j Unitat 10. Posició relativa de rectes i plans

Activitats

1. Dóna la interpretació geomètrica de les solucions dels siste-mes següents:

a)

Resolem el sistema: rang M rang M’ 3 i el sistema és compatible determinat i la solució (12, 21, 7) correspon a les coordenades d’un punt. Els tres plans son concorrents en un punt.

b)

Rang M rang M’ 2 i el sistema és compatible indetermi-nat. Els tres plans tenen una recta en comú l’equació de la qual es pot donar amb el sistema format per les equacions de dos dels tres plans.

2. Dos plans són paral·lels coincidents o són secants. Conside-ra els plans d’equacions:

1: 2x 3y z 0

2: x ky 4z 2

Raona si poden ser paral·lels per a algun valor de k.

Condició de paral·lelisme: que no es verifica ja

que independement del valor de k: els dos plans no

poden ser paral·lels per a cap valor de k.

3. Determina m i n per tal que els dos plans siguin paral·lels.

1: 6x my 4z 9 2: 9x 3y nz n

Condició de paral·lelisme:

.

Per a aquests valors els dos plans són paral·lels no coincidents.

4. Estudia la posició relativa dels tres plans:

1: 2x y z 1 2: x 2y z 1 3: x y 2z 1

Page 134: MATEMÀTIQUES - · PDF fileMatemàtiques 2 · Batxillerat · Solucionari No és permesa la reproducció total o parcial d’aquest llibre, ni el seu tractament informàtic, ni la

134 SOLUCIONARI DEL LLIBRE DE L’ALUMNE10

Estudiem els rangs: rang M 2 i rang M’ 3. El sistema és incompatible, però els plans no són paral·lels, per tant, són secants dos a dos.

5. Considera els plans d’equacions: x y 2z 1 0 i x y 3z 0. Justifica que són secants. Escriu l’equació

d’un tercer pla que passi per la recta que determinen aquests dos. És únic aquest pla? Raona la teva resposta.

Els dos plans no són paral·lels ja que , per tant,

són secants. La recta intersecció és:

Qualsevol combinació lineal de les dues equacions és un pla que conté aquesta recta, per tant, aquest pla no és únic.

6. Troba la intersecció de la recta determinada pels punts P(1,2,1) i Q(1,6,0) i el pla d’equació:

x y 3z 2

Equació de la recta PQ:

Intersecció:

Punt: (1, 2, 1)

7. Estudia la posició relativa de la recta:

i el pla d’equació x 3y z 0.

Intersecció:

0 ≠ 2 indica que el sistema és incompatible.

La recta i el pla són paral·lels.

8. Determina quina és la posició relativa del pla d’equació 3x 2y z 3 i la recta d’equació:

Recta en paramètriques:

Intersecció amb el pla:

Són secants i el punt d’intersecció és

9. Considera la recta r:

i estudia la posició relativa respecte del pla:

: 2x 8y 5z 26 0

Podem considerar el sistema format per les tres equacions:

rang M rang M’ 3

El sistema és compatible determinat: la recta i el pla són secants.

10. Comprova que les rectes r i s són secants i escriu l’equació general del pla que les conté.

Les rectes no són paral·leles:

(4, 1, 0) (2, 3, 2)

Considerem aquests dos vectors i un tercer PQ , format amb un

punt de cada recta: PQ (4, 1, 0), que és el mateix que el vector director de la primera recta, per tant, els tres vectors estan en un mateix pla i són secants.

L’equació del pla és:

Page 135: MATEMÀTIQUES - · PDF fileMatemàtiques 2 · Batxillerat · Solucionari No és permesa la reproducció total o parcial d’aquest llibre, ni el seu tractament informàtic, ni la

135MATEMÀTIQUES 2 10

11. Determina la posició relativa de les rectes r i s en cada cas:

a)

No són paral·leles: (1, 3, 1) ≠ k (5, 4, 1)

PQ (5, 5, 0)

Det(u , v , PQ ) les rectes s’encreuen.

b)

vector PQ

Det(u , v , PQ )

les rectes s’encreuen.

12. Troba la posició relativa de les rectes r, determinada pels punts P(1, 1,0) i Q( 2,3,1), i s, paral·lela a l’eix OX i que passa pel punt R(0, 1, 3).

Vector director de la recta r: u ( 3,4,1).

Vector director de la recta s: v (1, 0, 0).

No són paral·leles.

Vector PR ( 1, 2, 3).

Det(u , v , PR ) s’encreuen.

13. Escriu l’equació del pla que determinen les rectes:

i la que té la direcció del vector u (6,2, 1) i passa per l’origen de coordenades.

Vector director de r:

v u .

Com que r no conté el punt (0, 0, 0), les rectes són paral·leles. Els dos vectors no poden ser orientadors del pla. En prenem un,

el u i el format per un punt de cada recta: .

Equació del pla:

14. Verifica que les rectes r i s són coplanàries.

Són paral·leles? Si no és així, troba el punt d’intersecció.

Vector director de r: v (0, 1, 1),

Vector director de s: u (1, 0, 1).

v ≠ ku i les rectes no són paral·leles.

Intersecció:

15. Considera els vectors u , v i PR de l’activitat 12 i estableix la relació que puguis trobar entre ells.

Els tres vectors són linealment independents.

16. Comprova que els tres plans que conformen els tres eixos de coordenades cartesianes són perpendiculars dos a dos.

Els tres plans tenen de vectors associats, respectivament, els tres eixos de coordenades: e

1 (1, 0, 0); e2 (0, 1, 0) i e3 (0, 0, 1)

Page 136: MATEMÀTIQUES - · PDF fileMatemàtiques 2 · Batxillerat · Solucionari No és permesa la reproducció total o parcial d’aquest llibre, ni el seu tractament informàtic, ni la

136 SOLUCIONARI DEL LLIBRE DE L’ALUMNE10

Dos a dos són perpendiculars tal com es comprova amb el pro-

ducte escalar: e

1 · e2 0;

e1 · e3 0 i e2 · e3 0

17. Troba k per tal que les rectes r i s siguin perpendiculars. Hi ha algun valor de k que faci que aquestes rectes siguin secants?

Condició de perpendicularitat: el producte escalar dels dos vec-tors directors ha de ser nul.

( 1, 3, 1) · (1, 1, k) 1 3 k 0 k 4

Condició de secants:

18. Considera el pla d’equacions paramètriques:

Troba l’equació de la recta perpendicular a que passa per l’origen de coordenades.

El vector associat al pla és el vector director de la recta perpen-dicular al pla.

Equació del pla:

n v ( 2, 2, 1)

Equació de la recta:

19. Troba l’equació del pla mitjaner del segment determinat pels punts P(1,3,0) i Q( 1,5, 2). Per pla mitjaner s’entén el perpendicular al segment pel seu punt mitjà.

Punt mitjà: (0, 4, 1); vector PQ ( 2, 2, 2) n Equació del pla: 2x 2y 2z D 0 conté el punt (0, 4, 1): 8 2 D 0 D 10 2x 2y 2z 10 0 x y z 5 0

20. Troba el peu de la perpendicular traçada pel punt P(1, 1,0) al pla d’equació:

x y 3z 2 El peu de la perpendicular és la intersecció de la recta i el pla: P’

recta:

Punt:

21. Troba la projecció ortogonal de l’origen de coordenades so-bre la recta d’equacions:

Pla perpendicular a la recta per l’origen i la seva intersecció amb la recta és la projecció ortogonal.

recta:

pla: x y z 0

Projecció de l’origen:

O’: (1, 0, 1)

22. Calcula les coordenades del punt simètric del punt P(5,0, 1), respecte de la recta r:

El punt P i el seu simètric P” determinen un segment en el que el punt mitjà P’ és la projecció de P sobre r. La projecció és el punt intersecció del pla perpendicular a r per P i la recta r: x 2y z D 0 per P(5, 0, 1) D 6

x 2y z 6 0

1 4 6 0 56

el simètric P ”

Page 137: MATEMÀTIQUES - · PDF fileMatemàtiques 2 · Batxillerat · Solucionari No és permesa la reproducció total o parcial d’aquest llibre, ni el seu tractament informàtic, ni la

137MATEMÀTIQUES 2 10

23. Donada la recta r: i el pla

: x 2y z 3 0, troba les equacions de la recta r’, pro-

jecció ortogonal de la recta r sobre .

La recta r’ és la intersecció del pla amb el pla perpendicular a ell que conté la recta r. Equació d’aquest pla:

2x y 4 0

r x yx y z

: 2 4 02 3 0

24. Troba l’equació del pla que passa pel punt intersecció dels plans:

1: x y 2z 5 0

2: 2x 3z 1 0

3: x 2y z 1

i és paral·lel al pla x y z 2 0.

Resoldre el sistema per trobar el punt:

Pla paral·lel:

25. Esbrina la posició relativa dels plans:

2x y z 2 0 x 3y z 1

Si es tallen, troba l’equació de la recta paral·lela a la seva intersecció, que passa pel punt P(2, 1, 1).

Els plans no són paral·lels, són secants:

Recta paral·lela per P(2, 1, 1) (x, y, z) (2, 1, 1) ( 4, 1, 7)

26. Considera el pla x y z 3 i la recta,

r :

Digues si la recta talla el pla en un punt, si està continguda en el pla, o bé si la recta i el pla són paral·lels.

La recta en paramètriques:

vector director:

vector associat al pla: u (1, 1, 1)

v · n 0 la recta i el pla són paral·lels.

27. Determina l’equació de la recta que passa pel punt P(1, 2, 0), és paral·lela al pla d’equació 3x y z 2 i talla la recta:

r :

El vector director v de la recta està determinat per P i un punt arbitrari de la recta donada:

La condició de paral·lelisme amb el pla és: n · v 0

Equació de la recta:

28. Determina quina és l’equació del pla que conté la recta

és paral·lel al vector d’extrems P(4, 0, 1)

i Q(0, 2, 1) i passa per P.

Els vectors orientadors del pla són el vector director de la recta: v (3, 2, 1) i el PQ ( 4, 2, 2), que passa per P.

Page 138: MATEMÀTIQUES - · PDF fileMatemàtiques 2 · Batxillerat · Solucionari No és permesa la reproducció total o parcial d’aquest llibre, ni el seu tractament informàtic, ni la

138 SOLUCIONARI DEL LLIBRE DE L’ALUMNE10

Equació del pla:

29. Considera les rectes:

on a és un nombre real. Comprova que les dues rectes es tallen per a qualsevol valor de a. Troba a, si és possible, per tal que les dues rectes siguin perpendiculars.

Trobem la intersecció de les dues rectes:

La solució del sistema indica que les rectes es tallen per a qual-sevol valor de a.

Les rectes seran perpendiculars si:

1 1 2 1 1 0

1 2

, , ( , , )a

a

Activitats finals

1. Discuteix la posició relativa dels plans:

segons els valors de a ≠ 0.

Els plans són paral·lels o secants. Condició de paral·lelisme:

a2 3a a a2 2a 0 a 0 i a 2

La solució és a 2, ja que una condició és a ≠ 0.Per a a 2 els dos plans són paral·lels.Per a a ≠ 2 els dos plans són secants.

2. Troba a i b per tal que els tres plans següents passin per una mateixa recta:

1: x 2y z 1 2: 2x y az 0 3: 3x 3y 2z b

El sistema format per les equacions dels tres plans ha de ser compatible indeterminat de rang 2. Per tant, els dos determi-nants de 3r ordre han de ser nuls:

3. Hi ha algun valor de k pel qual els quatre plans tinguin un punt en comú? Si és així, troba aquest punt.

1: x 2z 3 0 2: 3x y z 1 3: 2y z 2 0 4: x y kz 5 0

El sistema format pels quatre plans ha de ser compatible deter-minat, és a dir, de rang 3.

Resolem el sistema format per les tres primeres equacions:

i substituïm aquesta solució en la quarta equació per trobar k:1 2k 5 0 k 2

4. Estudia la posició relativa de la recta:

i el pla : 3x 2y z 3.

Recta en paramètriques:

Substituim en l’equació del pla:

3( 2 3 ) 2 · 2 (3 2 ) 3

La recta i el pla són secants.

El punt d’intersecció és: .

Page 139: MATEMÀTIQUES - · PDF fileMatemàtiques 2 · Batxillerat · Solucionari No és permesa la reproducció total o parcial d’aquest llibre, ni el seu tractament informàtic, ni la

139MATEMÀTIQUES 2 10

5. Calcula els valors de m i n per tal que el pla:

: nx my z 2 0

contingui la recta:

Es trien dos punts qualssevol de la recta, per exemple: P(0, 3, 4) i R(3, 3, 1) que han de ser del pla:

3m 4 2 0 m 23n 3m 1 2 0 n 3

6. Considera una recta

i el pla

: A’’x B’’y C’’z D’’ 0

Digues què significa geomètricament que el sistema que s’obté considerant les tres equacions sigui incompatible. Di-gues què significa geomètricament que aquest sistema sigui compatible determinat o indeterminat.

Si el sistema format per les tres equacions és incompatible, la recta i el pla són paral·lels. Si el sistema és compatible deter-minat, el pla i la recta són secants, tenen un punt en comú. Si el sistema és compatible indeterminat, la recta està continguda en el pla.

7. Considera la recta

i el pla

: 2x 5y az 2 0

Determina els valors de a i b per tal que:

Podem estudiar els rangs de les matrius del sistema format per les tres equacions:

a) r i siguin secants. Troba el punt d’intersecció.

rang M rang M’ 3

Per a qualsevol a ≠ 1 i b qualsevol, el sistema és compatible determinat i té de solució. Les coordenades del punt d’inter-secció seran diferents segons els valors que es donin a a i b.

b) r i siguin paral·lels.

rang M ≠ rang M’Per a a 1 rang M 2

Estudiem el determinant amb la columna de termes indepen-dents:

Si 13b 39 0 b 3 rang M’ 2

Si a 1 i b ≠ 3 el sistema és incompatible i la recta és paral-lela al pla.

c) La recta r estigui continguda en el pla .

Si a 1 i b 3, rang M rang M’ 2 i la recta està contin-guda en el pla.

8. Considera les rectes:

s

on a és una constant. Comprova que aquestes dues rectes són secants per a qualsevol valor de a i determina el valor de a per tal que siguin perpendiculars.

En considerar y 1 en la recta s, s’obté x 1 i z 1 per a qual-sevol valor de a. Les rectes són secants i es tallen en el punt (1, 1, 1).

Per ser perpendiculars: (1,0,1) · (2,1,a) 0 a 2

9. Estudia la posició relativa de les rectes r i s:

s

s

Els vectors directors: i u (1, 0, 1) no són pro-

porcionals, per tant, les rectes no són paral·leles.

Estudiem el determinant format pels vectors v , u i PP’ amb un punt de cada recta:

PP’

Page 140: MATEMÀTIQUES - · PDF fileMatemàtiques 2 · Batxillerat · Solucionari No és permesa la reproducció total o parcial d’aquest llibre, ni el seu tractament informàtic, ni la

140 SOLUCIONARI DEL LLIBRE DE L’ALUMNE10

2 112

12

014

1 1 0

12

0 les rectes s’encreuen.

10. a) Explica la relació que hi ha entre el vector associat a un pla i un vector director d’una recta perpendicular a aquest.

El vector associat a un pla i el vector director d’una recta perpendicular són linealment dependents.

b) Troba l’equació del pla que conté el punt (1,1,0) i és perpendicular a la recta

Equació del pla: x 5y 3z D 0

per a (1, 1, 0), x 5y 3z 6 0

11. Troba l’equació de la recta projecció ortogonal de:

sobre el pla

: 2x 2y z 6 0

La projecció ortogonal de r sobre el pla és la intersecció d’aquest pla amb el pla que conté la recta i és perpendicular al pla do-nat.

Equació del pla:

La recta projecció és:

12. Sigui el punt P1(1, 0, 1), P2 el punt simètric de P1 respecte

del pla d’equació x 2y 0 i P3 el simètric de P2 respecte

del pla d’equació x 2y z 1. Troba l’equació general del pla que determinen els punts P1, P2 i P3.

El pla cal que contingui un dels punts, per exemple P1, i que els vectors orientadors siguin, per exemple P1P2 i P2P3 .

P1P2 és proporcional al vector associat al primer pla: (1, 2, 0).

P2P3 és proporcional al vector associat al segon pla: (1, 2, 1).

Equació del pla:

13. Determina l’equació del pla que passa per P(1, 2, 1) i conté la recta intersecció dels plans:

: x 2y z 3 i el pla YZ

Recta intersecció:

P(1, 2, 1) i el punt de la recta (0, 0, 3) donen: u (1, 2, 2)

Equació del pla amb v , u i P:

14. Comprova que les rectes:

són paral·leles i escriu l’equació del pla que les conté.

Vector director de r: v (1, 1, 1). Vector director de la recta s: u (1, 1, 1). Per tant, són paral·leles.

Per a l’equació del pla cal un vector determinat per un punt de

cada recta: P(1, 0, 2) i P’(5, 2, 0) PP’ (4, 2, 2)

Equació del pla:

Page 141: MATEMÀTIQUES - · PDF fileMatemàtiques 2 · Batxillerat · Solucionari No és permesa la reproducció total o parcial d’aquest llibre, ni el seu tractament informàtic, ni la

141MATEMÀTIQUES 2 10

15. Escriu l’equació de la recta que passa per l’origen de coorde-nades i és paral·lela a la recta:

Cal trobar l’expressió de la recta en paramètriques tot resolent el sistema:

vector director: v (1, 9, 5)

Recta paral·lela per l’origen:

16. Sigui r la recta d’equacions:

troba l’equació cartesiana del pla que conté la recta r i és perpendicular al pla y 0.

Un dels vectors orientadors del pla és el vector associat del pla perpendicular: n (0, 1, 0). L’altre vector i el punt els trobem en la recta r.

v (1, 0, 1)

Equació del pla:

17. Troba les equacions de la recta que passa pel punt P( 1, 0, 0) i és paral·lela als plans:

1: 2x y z 1 0 i 2: x 3y z 5

La recta és paral·lela a la intersecció dels dos plans:

Equació de la recta paral·lela:

18. Escriu l’equació del pla perpendicular a la recta que passa pels punts P(2, 1, 3) i Q( 3, 1, 2) i que conté el punt mitjà del segment PQ.

Vector associat al pla: PQ ( 5, 2, 5)

Punt mitjà del segment PQ:

Equació del pla: 5x 2y 5z D 0

5x 2y 5z 0

19. Determina si la recta i el pla

3x 2y 5 0 són paral·lels. Es troba la recta continguda en el pla?

Si la recta és paral·lela al pla, el vector director v ha de ser

perpendicular al vector n associat al pla. Comprovem-ho:v · n (2, 3, 5) · (3, 2, 0) 6 6 0 Són paral·lels.

La recta està continguda en el pla si, en aquest cas, qualsevol

punt de la recta és del pla:

P(3, 1, 1) 3 · 3 2 · 1 5 16 ≠ 0 la recta no està

continguda en el pla.

20. Considera la recta

i la recta s: .

Comprova que les dues rectes s’encreuen.

Dóna les coordenades d’un punt P de r i un punt Q de s que verifiquin la condició que la recta PQ sigui la perpendicular comuna a r i a s.

Esbrinem el valor del determinant format pels vectors directors

v i u i el P1Q1 ( 2, 3, 5)

les rectes s’encreuen.

Punts arbitraris de cada recta:

PQ

Page 142: MATEMÀTIQUES - · PDF fileMatemàtiques 2 · Batxillerat · Solucionari No és permesa la reproducció total o parcial d’aquest llibre, ni el seu tractament informàtic, ni la

142 SOLUCIONARI DEL LLIBRE DE L’ALUMNE10

Condició de perpendicularitat:

Els punts són: P(3, 4, 6) i Q(3, 5, 5).

21. Donada la recta

determina a per tal que existeixi un pla que contingui aquesta recta i que sigui perpendicular al vector v (1, 1, 1). Escriu l’equació cartesiana d’aquest pla.

Qualsevol pla que sigui perpendicular al vector v és de la forma: x y z D 0.

Si restem les dues equacions que donen la recta r, obtenim: 2x 5y 13, independent de a. Substituïm en el pla qual-sevol punt que verifiqui aquesta condició i obtenim diferents plans paral·lels. Si substituim el punt (1, 3, 0) tenim el pla x y z 4 0.

22. Determina k per tal que existeixi un pla que contingui la recta

i que sigui perpendicular al vector v ( 6, 8, k).

El vector director de la recta cal que sigui perpendicular al ( 6, 8, k).

x y zx y z

x

y v2 6 12 2 3 6

5 3

292

39

,22

1

392

1 6 8

,

, , ( , , )

z

k 18 36 0 18k k

Equació del pla: 6x 8y 18z D 0 pel punt de la recta

(5, 2, 0): 6x 8y 18z 14 0 3x 4y 9z 7 0

23. Considera la recta

Digues si el punt P(6, 2, 2) es troba en la recta paral·lela a r que passa per l’origen de coordenades.

Recta paral·lela per l’origen:

El punt (6, 2, 2) no es troba en aquesta recta.

24. Considera l’equació:

(x y z)2 (3x y 2z 1)2 0

Raona i dóna una interpretació geomètrica dels punts P(x, y, z) que verifiquen aquesta equació. No cal desenvolu-par els quadrats.

L’equació consta d’una suma de dos quadrats que és 0. Només pot ser aquest valor si es verifica:

que equival a la intersecció de dos plans no paral·lels. Per tant, els punts P(x, y, z) són de la recta intersecció.

Avaluació

1. Determina per a quins valors del paràmetre a el pla és paral·lel a la recta

Si considerem el sistema d’equacions que té per equacions les del pla i la recta r:

la recta r és paral·lela al pla per als valors de a pels quals el sistema és incompatible. El determinant de la matriu de coefi-cients d’aquest sistema val 2a 2, que s’anul·la per a a 1. Quan a 1 el sistema és compatible determinat, mentre que quan a 1 el sistema resulta incompatible. Per tant, a 1 és l’únic cas per al qual la recta i el pla són paral·lels

2. Donats els punts de l’espai A(2,0,0), B(0,1,0) i C(0,0,3).

a) Determina l’equació del pla que els conté.

L’espai director del pla està determinat pels vectors

(2, 1, 0) i (2, 0, 3). L’equació del pla que passa per

A (2, 0, 0) i té aquest espai director és 3x 6y 2z 6 0.

Page 143: MATEMÀTIQUES - · PDF fileMatemàtiques 2 · Batxillerat · Solucionari No és permesa la reproducció total o parcial d’aquest llibre, ni el seu tractament informàtic, ni la

143MATEMÀTIQUES 2 10

b) Calcula l’equació de la recta r perpendicular al pla que passa per l’origen.

Un vector perpendicular al pla és (3,6,2). Les equacions

de la recta amb aquest vector director que passa per l’origen

són: .

3. Calcula el peu de la recta perpendicular a la recta (x, y, z) (1, 1, 1) (0, 1, 1)

traçada des del punt A(1, 0, 1).

Un punt P qualsevol de la recta serà . El vector AP serà, doncs, . Imposem que aquest vector si-

gui perpendicular a (0, 1, 1). Tindrem:

d’on

Per tant, .

4. Donats el pla d’equació x 2y 3z 1 0, la recta

r:

i el punt P(2, 1, 1).

Calcula:

a) Unes equacions de la recta que passa per P i és perpendi-cular a .

(1, 2, 3) és vector perpendicular al pla i serà director de la recta perpendicular a que, en passar per P(2, 1, 1) té les equacions:

b) L’equació del pla que passa per P i és perpendicular a la recta r.

Un vector director de r (d’equacions paramètriques x 2z 3, y z 4, z z) és (2, 1, 1), i serà perpendicular al pla, que tindrà una equació general del tipus 2x y z d. En passar per P (2, 1, 1)), necessàriament d 6 i l’equació d’aquest pla és 2x y z 6.

c) Unes equacions de la recta que passa per P i talla perpen-dicularment r.

Determinem un punt Q de r per tant, Q 2z 3, z 4, z) tal

que el vector (2z 5, z 3, z 1) sigui ortogonal al

vector director de r, (2, 1, 1). La condició d’ortogonalitat

entre i és 4z 10 z 3 z 1 0, que dóna

i .

La recta buscada passa per P(2, 1, 1) i té el vector director . Unes equacions per a ella són:

d) Unes equacions de la recta que passa per P, és paral·lela al pla i tal que el seu vector director és perpendicular al de r.

Un vector director (a, b, c) ha de ser ortogonal a (1, 2, 3)

i a (2, 1, 1). Es pot trobar com a una de les solucions no nul-

les de (a, b, c)·(1, 2, 3) 0 i (a, b, c)·(2, 1, 1) 0.

La recta passa per (2, 1, 1) i, per tant, admet les equacions:

Page 144: MATEMÀTIQUES - · PDF fileMatemàtiques 2 · Batxillerat · Solucionari No és permesa la reproducció total o parcial d’aquest llibre, ni el seu tractament informàtic, ni la

144 SOLUCIONARI DEL LLIBRE DE L’ALUMNE11

j Unitat 11. Distàncies i angles

Activitats

1. Calcula la distància entre els parells de punts següents:

a) P(1, 1, 3) i Q(4, 2, 0)

u

b) R(0, 2, 5) i S(1, 3, 2)

u

c) T(4, 2, 1) i U(1, 1, 2)

u

d) V(5, 3, 7) i W(6, 2, 8)

u

2. Si la distància entre els punts P(2, 1, 3) i Q(t, 1, 2) és 3, calcula el valor de t. Quants punts Q verifiquen aquesta con-dició?

d P Q PQ( , ) 3

t t t t2 4 0 4 0( ) tt

1

2

04

Hi ha dues solucions: Q1(0, 1, 2) i Q2(4, 1, 2).

3. Per a quin valor de t és mínima la distància entre els punts P i Q de l’activitat anterior? Quin és el valor d’aquesta dis-tància mínima?

Derivem aquesta expressió respecte de t i igualem a zero:

La distància és mínima per a t 2, i el valor d’aquesta distància mínima és:

u

4. Donat el triangle que té com a vèrtexs els punts A( 1, 2, 4), B(4, 1, 6) i C(0, 3, 4), troba les coordenades del baricentre G. Si M és el punt mitjà del costat AB, comprova que es veri-fica d(G, C) 2 · d(M, G).

Es compleix que:

5. Donats els punts P(3, 6, 9) i Q(6, 3, 6), troba les co-ordenades del punt R alineat amb P i Q de manera que es verifiqui:

PQ PR3

Quina és la relació que hi ha entre d(R, Q) i d(P, R)? Com-prova la teva resposta fent-ne el càlcul.

Anomenem R(x, y, z).

3 3x 9 12 3x x 49 3y 18 9 3y y 315 3z 27 12 3z z 4

Les coordenades del punt R són (4, 3, 4)

Es compleix que d(R, Q) 2 · d(P, R).

6. El triangle de vèrtexs els punts A(1, 2, 0), B(3, 2, 1) i C(1, 4, 0) és rectangle en A. Comprova-ho:

a) Mitjançant el producte escalar.

b) Aplicant el teorema de Pitàgores.

7. Calcula el perímetre del triangle que s’obté unint els punts de tall del pla 3x 4y 3z 12 0 amb els eixos de coor-denades.

Eix OX: (4, 0, 0) punt PEix OY: (0, 3, 0) punt QEix OZ: (0, 0, 4) punt R

Perímetre:

10 4 2 u

Page 145: MATEMÀTIQUES - · PDF fileMatemàtiques 2 · Batxillerat · Solucionari No és permesa la reproducció total o parcial d’aquest llibre, ni el seu tractament informàtic, ni la

145MATEMÀTIQUES 2 11

8. Calcula la distància del punt P(2, 3, 1) a la recta r : (x, y, z) (1, 2, 1) (2, 1, 3).

Equació del pla que conté el punt P i és perpendicular a r:

: 2x y 3z 4 0

Intersecció del pla amb la recta r: punt P’

x 1 2 ; y 2 ; z 1 3 2 (1 2 ) ( 2 ) 3 (1 3 ) 4 02 4 2 3 9 4 0

9. Determina la distància del punt P( 1, 3, 1) a la recta

Interpreta el resultat obtingut.

Les coordenades del punt P verifiquen cadascuna de les equa-cions dels dos plans que determinen la recta r, la qual cosa significa que el punt P pertany a la recta recta. Aleshores:

Si P r d(P, r) 0.

10. Donada la recta r: (x, y, z) (0, 1, 2) (2, 0, 1) i el punt P(1, 0, 1), troba una expressió que et doni la distàn-cia d entre aquest punt P i un punt Q qualsevol de la recta r en funció del paràmetre .

Calcula després d(P, r) buscant el valor de que faci mínima la funció d f( ).

Les coordenades d’un punt Q qualsevol són:

Q(2 , 1, 2, )

Derivem respecte de :

u

11. Donat el triangle que té com a vèrtexs els punts O(0, 0, 0), P(2, 5, 0) i Q(3, 2, 4), calcula’n l’altura relativa al vèrtex P. Quant val l’àrea d’aquest triangle?

L’altura relativa al vèrtex O és la distància entre el punt O i la recta que determinen els punt P i Q.

Anomenem r aquesta recta:r: (x, y, z) (2, 5, 0) (1, 3, 4)

Pla que conté O i és perpendicular a r:

Punt O ’ intersecció entre el pla i la recta r:

2 3(5 3 ) 4 · ( 4 ) 02 15 9 16 0

26 13

O O

hO

212

532

412

52

72

2, , , ,

d O r d O O( , ) ( , )904

32

10 u

Per a aquesta altura, la base del triangle és:

b PQ

Ab hO

26

2

2632 10

232

65 2

u

u

12. Calcula la distància del punt P(2, 3, 5) al pla

: x 2y 2z 7 0

u

13. Dedueix una expressió general que et permeti calcular la distància entre l’origen de coordenades i un pla del qual coneixes l’equació cartesiana.

14. El pla d’equació 2x 2y z D 0 es troba a distància 2 de l’origen de coordenades. Troba D i interpreta el nombre de solucions obtingudes.

Es verifica:

Page 146: MATEMÀTIQUES - · PDF fileMatemàtiques 2 · Batxillerat · Solucionari No és permesa la reproducció total o parcial d’aquest llibre, ni el seu tractament informàtic, ni la

146 SOLUCIONARI DEL LLIBRE DE L’ALUMNE11

Hi ha dos plans que compleixen la condició que estableix l’enun-ciat:

2x 2y z 6 0 i 2x 2y z 6 0

Evidentment, es tracta de dos plans paral·lels.

15. Quina és la distància entre el punt P(2, 0, 1) i el pla que conté la recta i el punt (5, 1, 0)?

L’equació del pla és:

xy

z

1 3 42 2 1

2 00

u

16. Calcula la distància entre els parells de rectes següents:

a) r: (x, y, z) (5, 1, 8) (1, 0, 2)

Les coordenades d’un punt genèric P de la recta r són de la forma:

P(5 , 1, 8 2 )

I les d’un punt genèric P’ de la recta s:

P’(2 3 , 2 , 1 4 ).

Per tant,

u

b) r: (x, y, z) (2, 1, 1)

Procedint de la mateixa manera que en el cas de l’apartat anterior, s’obté:

u

17. Comprova que la recta

r: (x, y, z) (2, 3, 0) (1, 0, 2)

és paral·lela al pla : 2x 5y z 3 0. Troba després la distància entre la recta r i el pla .

El vector director de la recta i el vector associat al pla han de ser perpendiculars. En efecte:

u

18. La distància entre els plans 1: 3x 2y 4z 12 0 i

2: 3x 2y 4z D 0 és 3. Calcula D i interpreta geomè-tricament el nombre de solucions obtingudes.

Considerem un punt qualsevol d’un dels dos plans, per exemple, el pla 1: P(0, 0, 3).

Donat un pla, hi ha dos plans paral·lels a aquest i, per tant, paral·lels entre ells que es troben a una distància determinada del pla inicial.

19. Calcula la distància entre els plans

1: 5x 4y 2z 3 0 i

2: (x, y, z) (1, 0, 2) (2, 1, 3) (0, 1, 2)

Analitza’n prèviament la posició relativa.

Els plans són paral·lels, ja que el vector associat a 1 és perpen-dicular a cadascun dels vectors directors de 2:

(5, 4, 2) · (2, 1, 3) 0(5, 4, 2) · (0, 1, 2) 0

d( 1 , 2) d( 2, 1) u

20. Determina la posició relativa de la recta

i el pla : x 2y z 2 0. Quina és la distància entre r i ?

Page 147: MATEMÀTIQUES - · PDF fileMatemàtiques 2 · Batxillerat · Solucionari No és permesa la reproducció total o parcial d’aquest llibre, ni el seu tractament informàtic, ni la

147MATEMÀTIQUES 2 11

La solució del sistema:

és x 2, y 4, z 4.

Això vol dir que la recta r i el pla es tallen en el punt P(2, 4, 4). Aleshores: d(r, ) 0.

21. Considera les rectes

i

s: (x, y, z) (0, 3, 4) ( 1, 2, 1).

Comprova que r i s s’encreuen. Troba l’equació del pla que conté s i és paral·lel a r i calcula la distància entre la recta r i aquest pla.

S'encreuen, ja que no tenen cap punt en comú i no són paral-leles.

Pla que conté s i és paral·lel a r:

:x

yz

1 23 2 14 1 3

0

: 6x z 4 2y 6 4z 16 3y 9 x 0

: 7x y 5z 17 0 7x y 5z 17 0

u

22. Troba l’angle format per les rectes

r: (x, y, z) (2 , 1 2 , ) is: (x, y, z) (1, 1, 2) μ(2, 3, 5)

El vector director de la recta r és i el vector direc-tor de la recta s, .

23. Calcula l’angle format pels plans

1: 4x 3y z 7 0

2: (x, y, z) (1, 0, 0) (2, 1, 0) (1, 3, 2)

Expressem el pla 2 en la forma Ax By Cz D 0

2

1 2 11 30 2

0 2 2 6 4 0:x

yz

x z z y

2x 4y 7z 2 0

Vector associat al pla 1: .

Vector associat al pla 2: .

24. Donat el pla : 3x y 2z 1 0 i la recta

calcula l’angle que formen.

Vector director de la recta: (2, 1, 4).

Vector associat al pla: (3, 1, 2).

25. Considera la mateixa recta i el mateix pla de l’activitat ante-rior.

a) Troba l’equació de la recta r ’, projecció ortogonal de la recta r sobre el pla .

Tingues en compte que la recta r ’ és la intersecció entre el pla i un altre pla ’ que conté r i és perpendicular a .

Equació del pla ’ que conté r i és perpendicular a .

:x

yz

2 2 31 1

1 4 20

2x 4 2z 2 12 y 3z 3 4y 4x 8 0

’: 2x 16y 5z 9 0 2x 16y 5z 9 0

Recta r ’, intersecció entre els plans i ’:

b) Calcula l’angle format per les rectes r i r ’ i comprova que és el mateix que l’angle format per la recta r i el pla que has trobat a l’activitat anterior.

Angle que formen les rectes r i r ’: .

Vector director recta r .

vector director recta r ’ .

Page 148: MATEMÀTIQUES - · PDF fileMatemàtiques 2 · Batxillerat · Solucionari No és permesa la reproducció total o parcial d’aquest llibre, ni el seu tractament informàtic, ni la

148 SOLUCIONARI DEL LLIBRE DE L’ALUMNE11

26. Donats els vectors (2, 4, 1) i (1, 3, 2), calcula’n el producte vectorial i comprova que el nou vector que has obtingut és perpendicular al vector i al vector .

u v

e

e

ee e e

1

2

3

1 2 3

2 1

4 3

1 25 5 10

27. Calcula l’angle format pels vectors ( 1, 3, 4) i (2, 0, 1) a partir de:

a) el seu producte escalar.

b) el seu producte vectorial.

v w

e

e

ee e e

1

2

3

1 2 3

1 2

3 0

4 13 8 6 e

e e e

2

1 2 33 9 6

28. Determina tots els vectors de mòdul 4 que són perpendicu-lars als vectors (3, 0, 1) i (1, 1, 0). Interpreta el nombre de solucions obtingudes.

r p q

e

e

ee e e

1

2

3

3 2 1

3 1

0 1

1 03 r ( , , )1 1 3

Vector unitari que té la mateixa direcció i sentit que el vector:

Vector unitari que té la mateixa direcció i sentit contrari que el vector:

Hi ha dos vectors que compleixen les condicions de l’enunciat de l’activitat:

Es tracta de dos vectors oposats i perpendiculars al pla que determinen els vectors .

29. Quant ha de mesurar l’angle que formen dos vectors per tal que el mòdul del seu producte vectorial sigui màxim?

90º. En aquest cas:

30. Donats els vectors (2, 0, 1) i (1, 1, 2), comprova que els vectors x i x tenen el mateix mòdul, la mateixa direcció i sentits contraris.

És senzill veure que es tracta de dos vectors oposats.

31. Considera el pla determinat pels punts P(1, 2, 1), Q(3, 2, 1) i R(0, 2, 3). Utilitza el producte vectorial per determinar-ne el vector associat i troba l’equació carte-siana d’aquest pla.

n

e

e

ee e e n

1

2

3

1 2 3

1 1

2 0

1 24 2 ( , , )4 1 2

Equació cartesiana del pla:

32. Donats els vectors (1, 3, 2), ( 1, 0, 2) i (2, 1, 3), comprova que es verifiquen les propietats del

producte vectorial abans esmentades.

a) Anticommutativa:

b) Distributiva respecte de la suma de vectors:

c) Producte per un nombre real:

33. Amb els mateixos vectors de l'activitat anterior, esbrina si es verifica la igualtat:

No es verifica, ja que:

34. Utilitza l’expressió en forma de determinant del producte vectorial per demostrar que si i són linealment depen-dents, .

Page 149: MATEMÀTIQUES - · PDF fileMatemàtiques 2 · Batxillerat · Solucionari No és permesa la reproducció total o parcial d’aquest llibre, ni el seu tractament informàtic, ni la

149MATEMÀTIQUES 2 11

Suposem que

u v v v

e v v

e v v

e v v

e1 1 1

2 2 2

3 3 3

11 1 1

2 2 2

3 3 3

0 0

v v

e v v

e v v

35. Raona la validesa de l’afirmació següent:

Si són dos vectors linealment independents, els vec-tors formen una base de V3.

L’afirmació és correcta, perquè en ser i dos vectors lineal-ment independents, el vector x és perpendicular al pla que generan aquests dos vectors.

Per tant, els vectors , i x també són linealment inde-pendents, i, en conseqüència, formen una base de V3.

36. Calcula l’àrea del paral·lelogram que es pot obtenir a partir dels vectors (1, 4, 3) i (3, 2, 4).

v w

e

e

e

1

2

3

1 3

4 2

3 4

37. Els punts A(0, 1, 0), B(2, 1, 3) i C( 1, 3, 2) són tres vèr-texs consecutius d’un paral·lelogram. Es demana:

a) Les coordenades del quart vèrtex D.

Anomenem D(x, y, z).

(2, 2, 3) ( 1 x, 3 y , 2 z)

2 1 x x 32 3 y y 53 2 z z 5

Les coordenades del quart vèrtex són D( 3, 5, 5).

b) L’àrea del paral·lelogram.

AB AD

e

e

e

1

2

3

2 3

2 4

3 5

c) La distància entre la recta r que conté els punts A i B, i la recta s que conté els punts C i D.

La distància d(r, s) coincideix amb l’altura del paral·lelogram ABCD. Si prenem com a base d’aquest paral·lelogram el cos-tat DC, es compleix:

A = base · altura A DC d r s( , )

s

38. Calcula, fent ús del producte vectorial, la distància entre el punt P(1, 0, 2) i la recta que conté el punt Q(2, 1, 3) i és paral·lela a la recta:

(x 1, y, z 2) (1, 0, 3)

u QP

e

e

e

1

2

3

1 1

0 1

3 5

d P ru QP

u( , )

74

10

37

5

1855

u

39. Calcula, fent ús del producte vectorial, la distància en-tre el punt P(2,0, 1) i la recta r determinada pels plans x y z 3 0 i x y 3z 5 0.

Equació de la recta

r:

z t, x 2t 1, y t 4

r: (x, y, z) (1, 4, 0) t(2, 1, 1)

u QP

e

e

e

1

2

3

2 1

1 4

1 1

d P ru QP

u( , )

99

6

33

2

662

u

Page 150: MATEMÀTIQUES - · PDF fileMatemàtiques 2 · Batxillerat · Solucionari No és permesa la reproducció total o parcial d’aquest llibre, ni el seu tractament informàtic, ni la

150 SOLUCIONARI DEL LLIBRE DE L’ALUMNE11

40. Determina la mesura de les tres altures del triangle que té com a vèrtexs els punts O(0,0,0), P(1,2,3) i Q( 2,1, 1).

Altura relativa al vèrtex 0: distància entre el punt 0 i la recta que passa pels punts P i Q.

PQ PO

e

e

e

1

2

3

3 1

1 2

4 3

78 u

Seguint el mateix procediment, s’obté:

altura relativa al vèrtex u

altura relativa al vèrtex u

41. Els punts R(3, 4, 2), S(1, 0, 1) i T(0, 2, 3), estan ali-neats? Si la resposta és no, troba l’àrea del triangle que determinen.

RS RT

e

e

e

1

2

3

2 3

4 2

3 1

42. Utilitza el producte vectorial per trobar un vector associat al pla que conté els punts M(1, 2, 1), N(3, 1, 1) i P(7, 7, 5). Interpreta el resultat que obtinguis.

Observem que , o també,

Per tant, . És així perquè els tres punts M, N i P estan alineats, i, en conseqüència, no determinen cap pla.

43. Donats els vectors (2, 4, 1), (1, 3, 1) i (0, 2, 3), calcula:

a)

b)

c)

d)

44. Els vectors , i verifiquen 2 . Pots calcular-ne el producte mixt sense especificar-ne l’ordre? Si la resposta és afirmativa, quant val? Justifica cadascuna de les respostes.

El producte mixt dels tres vectors , i és igual a zero, ja que es tracta de tres vectors que són linealment dependents. En calcu-lar el determinant, una fila seria combinació de les altres dues.

45. Sabem que el producte mixt de tres vectors és diferent de zero. Què pots afirmar respecte d’aquests vectors?

Que són linealment independents.

46. Esbrina, utilitzant el producte mixt, si els punts P(1, 2, 1), Q(3, 2, 1), R(0, 1, 3) i S( 3, 1, 0) són coplanaris.

Són coplanaris, ja que, per exemple,

i això vol dir que es tracta de tres vectors linealment depen-dents.

47. Utilitza el producte mixt per calcular el valor de t que fa que els vectors (1, 1, 3), (2, 3, 4) i (t, 2, 1) siguin linealment dependents. Expressa després el vector en combinació lineal dels vectors i .

v w ut

, , 01 21 3 23 4 1

0

3 12 4t 9t 2 8 0 5t 15 0 t 3

(3, 2, 1) 1 (1, 1, 3) 2 (2, 3, 4)

La solució del sistema és 1 2 1

Aleshores:

Page 151: MATEMÀTIQUES - · PDF fileMatemàtiques 2 · Batxillerat · Solucionari No és permesa la reproducció total o parcial d’aquest llibre, ni el seu tractament informàtic, ni la

151MATEMÀTIQUES 2 11

48. Calcula l’àrea total i el volum de la piràmide que té com a vèrtexs els punts P(2, 3, 1), Q(4, 1, 2), R(2, 3, 5) i S( 2, 1, 3).

L’àrea total de la piràmide és la suma de les àrees dels triangles que en determinen les cares:

At APQR APQS APRS AQRS

PQ PR

e

e

ee e

1

2

3

1 2

2 0

2 0

3 48 8

PQ PS

e

e

e

1

2

3

2 4

2 4

3 2

PR PS

e

e

ee

1

2

3

2

0 4

0 4

4 216 16ee1

PR PS PR PS( , , )16 16 0 16 2

QR QS

e

e

e

1

2

3

2 6

2 2

7 5

QR QS 8 29

A QR QS

A

QRS

t

12

12

8 29 4 29

4 2 12 8 2

2u

4 29

12 12 2 4 29

50 5

2

2

2

u

u

uAt ,

El volum de la piràmide és:

49. Troba el volum del paral·lelepípede format a partir dels vec-tors:

(4, 3, 5), (1, 0, 3) i ( 3, 5, 1)

u v w

V u

, ,

,

4 1 33 0 55 3 1

25 27 3 60 55

v w, 55 3u

50. a) Determina les coordenades dels vèrtexs de la piràmide limitada pels eixos de coordenades i el pla d’equació:

3x 4y 2z 12 0

x y 0 z 63x 4y 2z 12 0 y z 0 x 4 x z 0 y 3

Els vèrtexs de la piràmide són:O(0, 0, 0), P(4, 0, 0), Q(0, 3, 0) i R(0, 0, 6).

b) Calcula’n el volum.

OP OQ OR

V OP

, ,4 0 00 3 00 0 6

72

16

, ,OQ OR16

72 12 3u

51. Considera les rectes següents:

r: (x, y, z) (5, 1, 8) (1, 0, 2)

a) Comprova que s’encreuen.

S’encreuen, ja que no tenen cap punt en comú i no són paral-leles.

Page 152: MATEMÀTIQUES - · PDF fileMatemàtiques 2 · Batxillerat · Solucionari No és permesa la reproducció total o parcial d’aquest llibre, ni el seu tractament informàtic, ni la

152 SOLUCIONARI DEL LLIBRE DE L’ALUMNE11

b) Calcula de tres maneres diferents la distància que les separa.

PQ u v

u

3 1 33 0 19 2 4

9 18 12 6 9

v

e

e

ee e e

1

2

3

3 2 2

1 3

0 1

2 46 4 2ee1

d r sPQ u v

u v( , )

, , 93

3 u

Nota: aquesta activitat només s’ha resolt mitjançant el pro-cediment indicat en l’apartat.

52. Comprova que els punts A(1, 0, 0), B(2, 5, 3), C( 2, 4, 7) i D( 1, 2, 5) no són coplanaris. Calcula el volum de la piràmide que defineixen i troba la distància entre les arestes AB i CD.

No són coplanaris, ja que .

AB AC AD( , , ); ( , , ); ( , ,1 5 3 3 4 7 2 2 5)

AB AC AD, ,1 3 25 4 23 7 5

20 18 70 24 75 14 117

16

V AB AC AD, ,16

117392

3u

Equació de la recta que conté l’aresta AB:r: (x, y, z) (1, 0, 0) (1, 5, 3)

Equació de la recta que conté l’aresta CD:r ’: (x, y, z) ( 1, 2, 5) (1, 2, 12)

Les rectes r i r’ s’encreuen. Per qualsevol dels procediments ana-litzats en aquesta unitat s’obté:

u

53. Determina la posició relativa de les rectes:

rx y

z

s xy

z

:

:

12

11 3

23 1

Si les rectes r i s s’encreuen, utilitza el producte mixt per calcular la distància que les separa.

Les rectes r i s s’encreuen.

PQ u v, ,1 2 13 1 34 1 1

1 24 3 4 6 3 33

2 1

1 3

1 1

1

2

3

u v

e

e

e

54. Calcula l’àrea total i el volum del paral·lelepípede que es pot obtenir a partir dels vectors (2, 1, 0), (1, 2, 3) i x . De quin tipus de paral·lelepípede es tracta?

u v

e

e

e

1

2

3

2 1

1 2

0 3

u v ( , , )3 6 5

Anomenem u v ( , , )3 6 5 .

Es compleix que per tant, els vectors veri-fiquen:

a) Són perpendiculars dos a dos.

b) Tenen el mòdul diferent:

En conseqüència, el paral·lelepípede és un ortòedre.

Page 153: MATEMÀTIQUES - · PDF fileMatemàtiques 2 · Batxillerat · Solucionari No és permesa la reproducció total o parcial d’aquest llibre, ni el seu tractament informàtic, ni la

153MATEMÀTIQUES 2 11

55. Els punts O(0, 0, 0), P(2, 3, 1) i Q(1, 2, 3) són tres vèr-texs d’una piràmide. Quina és la condició analítica que han de verificar les coordenades (x, y, z) del quart vèrtex R per tal que el volum d’aquesta piràmide sigui 14 u3? Fes-ne la interpretació geomètrica.

V OP OQ OR V16

14, ,

OP OQ OR x y z( , ); ( , , ); ( , , )2 3 1 1 2 3

OP OQ ORxyz

z y x

, ,2 13 21 34 9 2 3 6 7 7 7x z y x y z

7x 7y 7z 84 x y z 12 0

7x 7y 7z 84 x y z 12 0

Es tracta de dos plans que són paral·lels entre ells i paral·lels al pla determinat pels punts O, P i Q. La distància entre cadascun d’aquests plans i el pla definit pels punts O, P i Q és precisament l’altura de la piràmide corresponent.

Activitats finals

1. Troba la distància del punt (3, 4, 5) a la recta d’equació

u QP

e

e

e

1

2

3

1 4

2 6

1 10

20 6 4 8 10 6

26

1 3 2 3 2 1e e e e e e

e1 2 314 2e e

u QP u QP

u

d P r

( , , )

( , )

26 14 2 876

6

876

6146 u

2. Es demana l’equació del pla que conté l’eix OX i dista 6 uni-tats del punt P(0, 10, 0).

Equació general del pla: Ax By Cz D 0.

El pla conté l’eix OX un dels punts del pla és l’origen de co-ordenades D 0.

Vector associat al pla és normal al vector , un dels vectors directors del pla.

Per tant, el pla que hem de trobar és de la forma By Cz 0.Com que el punt P(0, 10, 0) dista 6 unitats d’aquest pla, es compleix:

Hi ha dos plans que verifiquen les condicions establertes a l’enunciat d’aquesta activitat:

1: 3y 4z 0 i 2: 3y 4z 0

3. Determina un punt de la recta

r: (x, y, z) (1, 1, 2) (2, 3, 2)

que equidisti dels plans:

3x 4y 1 0 i 4x 3y 1 0

És única la solució?

Interpreta geomètricament el resultat que has obtingut.

Les coordenades d’un punt genèric P de la recta r són:

P(x, y, z) P(1 2 , 1 3 , 2 2 )

S’ha de verificar:

d(P, 1) (P, 2)

Page 154: MATEMÀTIQUES - · PDF fileMatemàtiques 2 · Batxillerat · Solucionari No és permesa la reproducció total o parcial d’aquest llibre, ni el seu tractament informàtic, ni la

154 SOLUCIONARI DEL LLIBRE DE L’ALUMNE11

Hi ha dos punt solució. Els plans 1 y 2 són perpendiculars i la

recta r té un punt d’intersecció amb cadascun d’ells.

4. Considera el punt P(1, 1, 3) i la recta:

r: (x, y, z) ( , 2 , 2 )

Troba:

a) L’equació del pla perpendicular a la recta r que conté el punt P.

L’equació general d’un pla perpendicular a la recta r és x y 2z D 0. Com que aquest pla ha de contenir el punt P(1,1,3), es compleix:

1 1 6 D 0 D 8 x y 2z 8 0

b) El punt intersecció d’aquest pla amb la recta r.

x ; y 2 ; z 2 ; x y 2z 8 0

El compliment simultani de les quatre equacions ens du a una altra equació que ens permet trobar el valor del paràmetre que correspon al punt d’intersecció entre la recta i el pla:

2 4 8 0 1 1 x 1, y 3, z 2

El punt és P’(1, 3, 2).

c) La distància del punt P a la recta r.

u

5. Donades les rectes:

r: x y zs: (x, y, z) (1, 1, 1) (2, 1, 1)

es demana:

a) L’equació de la recta perpendicular a r i a s.

Un punt P genèric de la recta r és P( , , ) i un punt P ’ genèric de la recta s:

P ’(1 2 , 1 , 1 )

El vector:

ha de ser perpendicular al vector al vector

Per tant:

Les dues igualtats anteriors ens condueixen al sistema:

la solució del qual és .

Per tant, tenim que:

P P PP37

37

37

97

67

67

67

, , ; , , ; , ,37

97

La recta perpendicular a les rectes r i s és la recta que conté els punt P i P ’ i té per equació:

b) La distància entre les rectes r i s.

u

6. Determina els vectors de mòdul 2 que són alhora perpendi-culars als vectors (2, 2, 3) i (3, 3, 2).

Si representem per un d’aquests vectors, s’ha de complir alhora:

(v1, v2, v3) · (2, 2, 3) 0 2v1 2v2 3v3 0

(v1, v2, v3) · (3, 3, 2) 0 3v1 3v2 2v3 0

La solució del sistema format per aquestes tres equacions és:

Hi ha dos vectors que verifiquen les condicions de l’enunciat de l’activitat:

7. Calcula l’angle que formen les rectes:

s: (x, y, z) (1, 2, 3) ( 3, 4, 5)

El vector director de la recta r és ( 3, 4, 5) i el de la recta s, ( 3, 4, 5).

Els dos vectos són perpendiculars 90º

8. Calcula l’àrea del triangle que té com a vèrtexs els punts O(0, 0, 0), P(1, 1, 0) i Q punt intersecció de la recta (x, y, z) ( 1 2 , 1 3 , 2 ) amb el pla XY.

Q: punt intersecció de la recta.

(x, y, z) ( 1 2 , 1 3 , 2 ) amb el pla z 0.2 0 2 Q(3, 5, 0)

Page 155: MATEMÀTIQUES - · PDF fileMatemàtiques 2 · Batxillerat · Solucionari No és permesa la reproducció total o parcial d’aquest llibre, ni el seu tractament informàtic, ni la

155MATEMÀTIQUES 2 11

OP OQ

e

e

ee e

1

2

3

3 3

1 3

1 5

0 05 3 22 3e

9. Troba l’angle que formen les rectes

r: x y z

Vector director recta r: ( 1, 1, 1).Vector director recta s: ( 1, 0, 1).

Com que , les dues rectes són perpendiculars 90º.

10. Troba els vèrtexs de la piràmide triangular que determinen els plans y 0, z 0, x y 0 i 3x 2y z 15 0 i cal-cula’n el volum.

Vèrtex O x y z 0 O(0, 0, 0)

Vèrtex A x 5, y z 0 A(5, 0, 0)

Vèrtex B x y 0, z 15 B(0, 0, 15)

Vèrtex C

x y 3, z 0 C(3, 3, 0)

El volum de la piràmide es calcula:

u v w, ,5 0 30 0 30 15 0

225

11. Anomenem pla paral·lel mitjà a dos plans paral·lels aquell pla els punts del qual equidisten d’aquests dos plans. Determina l’equació del pla paral·lel mitjà als plans 2x 2y z 4 0 i 2x 2y z 10 0.

L’equació del pla que ens demanen ha de ser del tipus 2x 2y z D 0.

Si P(x, y, z) és un punt d’aquest pla, s’ha de complir la igualtat:

d(P, 1) d(P, 2)

2x 2y z 4 2x 2y z 104x 4y 2z 6 0 2x 2y z 3 0

12. Troba la condició analítica que verifiquen tots els punts de 3 que disten 2 unitats del pla d’equació 3x y z 0.

Representem per P(x, y, z) un d’aquests punts.

3

112

x y z

Són punts que pertanyen a dos plans paral·lels al pla que ens donen i que es troben a distància 2 (equidisten) d’aquest pla, que és el pla paral·lel mitjà dels altres dos.

13. Determina l’angle format per:

a) La recta x y i el pla y 0.

Page 156: MATEMÀTIQUES - · PDF fileMatemàtiques 2 · Batxillerat · Solucionari No és permesa la reproducció total o parcial d’aquest llibre, ni el seu tractament informàtic, ni la

156 SOLUCIONARI DEL LLIBRE DE L’ALUMNE11

b) Els plans 3x 5 0 i 2x y z 2 0.

14. Calcula la distància del punt P(3, 5, 6):

a) A l’origen de coordenades.

u

b) A cadascun dels plans XY, XZ i YZ.

pla XY: 6 upla XZ: 5 upla YZ: 3 u

c) A cadascun dels tres eixos de coordenades.

15. a) Calcula l’àrea total i el volum del paral·lelepípede gene-rat pels vectors de la figura.

V 3 · 5 · 4 60 u3

Nota: cal observar que l’àrea i el volum d’aquest ortòedre es poden determinar de manera molt senzilla, ja que se’n conei-xen les tres dimensions.

b) Determina l’angle que formen les rectes OT i QS.

9 25 16 0 les rectes OT i SQ són per-pendiculars 90º.

16. Determina l’equació dels plans de vector associat (1, 2, 3) i que disten 3 unitats de l’origen de coordenades.

Els plans de vector associat (1, 2, 3) són de la forma:x 2y 3z D 0.

La distància de l’origen de coordenades a un d’aquests plans és 3. Aleshores:

Hi ha dos plans solució:

17. Calcula l’àrea total i el volum de la piràmide limitada pels plans x 0, y 0, z 0 i 2x 3y 4z 12 0.

Els vèrtexs de la piràmide són els punts:

O(0, 0, 0); A(6, 0, 0); B(0, 4, 0) i C(0, 0, 3)

Àrea de la piràmide:

AB AC

e

e

e

e

1

2

3

1

6 6

4 0

0 3

12 24e e3 218 12 18 24( , , )

A 276 29

227 3 29 2u

Volum de la piràmide:

Page 157: MATEMÀTIQUES - · PDF fileMatemàtiques 2 · Batxillerat · Solucionari No és permesa la reproducció total o parcial d’aquest llibre, ni el seu tractament informàtic, ni la

157MATEMÀTIQUES 2 11

18. Esbrina la longitud del segment projecció ortogonal del seg-ment que té com a extrems els punts A(2, 4, 1) i B(1, 2, 5) sobre el pla 2x y z 0.

Si representem per l la longitud del segment, es compleix:

on és l’angle que forma la recta que conté els punts A i B amb el pla 2x y z 0.

u

19. Quina és la condició que han de verificar els vectors asso-ciats a dos plans que són perpendiculars? Expressa-la analí-ticament utilitzant els seus components.

Els vectors associats també han de ser perpendiculars: n n 0

pla 1: Ax By Cz D 0 (A,B,C)pla 2: A’x B’y C’z D’ 0 (A’, B’, C’)

20. Troba l’equació de la recta projecció ortogonal de la recta (x, y, z) (1, 2, 4) (3, 2, 1) sobre el pla:

3x 2y z 2 0.

Determinem l’equació del pla que conté la recta: (x, y, z) (1, 2, 4) (3, 2, 1) i és perpendicular al pla 3x 2y z 2 0.

D’aquest pla, en coneixem el punt (1, 2, 4) i dos vectors directors: (3, 2, 1) i (3, 2, 1).

xyz

y z1 3 32 2 24 1 1

0 2 10 0

La recta que ens demanen és la recta intersecció dels plans 3x 2y z 2 0 i y 2z 10 0.

21. Donats els vectors (3, 2, 4) i (1, 4, 2), troba:

a) El seu producte vectorial.

u v

e

e

e

1

2

3

3 1

2 4

4 2

b) Un vector unitari perpendicular als dos vectors.

Hi ha dos vectors unitaris que compleixen aquesta condició.

El que té la mateixa direcció i el mateix sentit que el vector

:

El que té la mateixa direcció que el vector, però el sentit

contari :

c) L’àrea del paral·lelogram que es pot dibuixar a partir de i .

d) El volum del paral·lelepípede que s’obté a partir dels vectors , , i x .

22. Calcula l’angle que formen dues de les diagonals d’un cub. Per més comoditat, situa un dels vèrtexs del cub a l’origen de coordenades i considera’l d’aresta unitat.

Considerem la recta r que conté la diagonal determinada pels vèrtexs (0, 0, 0) i (1, 1, 1), i la recta s que conté la diagonal determinada pels vèrtexs (1, 0, 0) i (0, 1, 1).

Vector director de la recta r (1,1,1).Vector director de la recta s ( 1,1,1).

L’angle que formen les diagonals del cub verifica:

Page 158: MATEMÀTIQUES - · PDF fileMatemàtiques 2 · Batxillerat · Solucionari No és permesa la reproducció total o parcial d’aquest llibre, ni el seu tractament informàtic, ni la

158 SOLUCIONARI DEL LLIBRE DE L’ALUMNE11

23. Troba els punts de la recta que passa per A( 1, 0, 1) i B(1, 2, 3) que distin 3 unitats del punt C(2, 1, 1). Inter-preta geomètricament el nombre de solucions obtingudes.

L’equació de la recta r que conté els punts A i B és: (x, y, z) ( 1, 0, 1) (1, 1, 1) (x, y, z) ( 1 , , 1 )

La distància entre el punt C (2, 1, 1) i un punt P qualsevol de la recta r ve donada per l’expressió:

Per tant, per a un punt P de r que es troba a distància 3 del punt C, es compleix:

Com que el punt C no pertany a r, hi ha dos punts que verifiquen la condició de l’enunciat:

1 1 P1(0, 1, 2)

24. Determina la distància de la recta

a cadascun dels eixos de coordenades.

La recta r es pot expressar en la forma:

r: (x, y, z) (2, 2, 0) (1, 1, 1)

D’altra banda, l’eix OX té per equació:(x, y, z) (1, 0, 0)

La distància entre la recta r i l’eix OX es pot calcular mitjançant l’expressió:

Fent els càlculs, obtenim:

Amb el mateix procediment calculem les altres dues distàncies que ens demanen. Obtenim els resultats següents:

Distància entre la recta r i l’eix OY u

Distància entre la recta r i l’eix OZ u

25. Donat el triangle que té com a vèrtexs els punts A( 3, 4, 0), B(3, 6, 3) i C( 1, 2, 1), troba la mesura dels seus angles. Pertany l’origen de coordenades al pla que conté aquest tri-angle?

Pel mateix procediment obtenim:

Equació del pla que conté els punts A, B i C:

xy

zx z

3 6 24 2 2

3 10 2 3 0

no hi pertany, ja que D ≠ 0.

26. Donades les rectes

r: x 1 y 2 z 3s: (x, y, z) (1 , 2 , 3 2 )

localitza dos punts, un de cada recta, tals que la distància entre ells sigui mínima.

Un punt P genèric de la recta r és de la forma:P(1 ’, 2 ’, 3 ’)

i un punt P ’ genèric de la recta s té per coordenades:P ’(1 , 2 , 3 2 )

Perquè es verifiquin les condicions de l’enunciat cal localitzar P r i P ’ s tals que el vector sigui perpendicular a ca-dascun dels vectors directors (1,1,1) i ( 1,2,2) de les dues rectes.

Com que,

PP

PP u

( , , )2 2 2

0 3 3 2

0 9 3 4PP v

El sistema:

té per solucions

En conseqüència:

Page 159: MATEMÀTIQUES - · PDF fileMatemàtiques 2 · Batxillerat · Solucionari No és permesa la reproducció total o parcial d’aquest llibre, ni el seu tractament informàtic, ni la

159MATEMÀTIQUES 2 11

27. Els punts A(1, 2, 3), B(2, 4, 1) i C(3, 2, 3) són tres vèrtexs consecutius d’un paral·lelogram. Calcula’n l’àrea i la mesura dels angles que formen les seves diagonals. De quin tipus de paral·lelogram es tracta?

AB AC

e

e

e

1

2

3

1 2

2 0

4 0

Representem per D(x, y, z) el quart vèrtex del paral·lelogram.Es compleix:

1 3 x x 22 2 y y 04 3 z z 7

Per tant, D(2, 0, 7).

Angle que formen les diagonals:

0 les diagonals són perpendiculars 90º.

El paral·lelogram és un rombe, ja que:

té les diagonals perpendiculars.

els costats no són perpendiculars .

els costats són iguals .

28. Troba la distància del punt P(1, 2, 4) al pla que conté la recta i és paral·lel a la recta

s:

Equació vectorial de la recta s:(x, y, z) ( 1, 1, 0) (4, 2, 1)

Equació general del pla que ens demanen:

xy

zx y z

1 2 41 2

1 3 10 5 14 8 13 0

29. Determina l’angle que forma la recta

amb el pla que conté el punt (2, 1, 1) i la recta s: x 1 y z 2.

Equació general del pla :

xyz

x y2 1 11 1 11 1 3

0 1 0

Hem considerat com a vectors directors del pla el vector di-rector de la recta s i el vector determinat pels punts (2, 1, 1) i (1, 0, 2), ambdós pertanyents a .

On (1,3,2) és un vector director de la recta r i (1, 1, 0), el vector associat a .

30. Calcula la distància entre el punt P(1, 2, 1) i el punt P ’ simètric de P respecte del pla que conté el punt A(4, 1, 0) i és paral·lel al pla x 3y z 5 0.

Equació del pla que conté el punt (4, 1, 0) i és paral·lel al pla x 3y z 5 0:

Equació de la recta r que passa per P i és perpendicular a :

(x, y, z) (1, 2, 1) (1, 3, 1)(x, y, z) (1 , 2 3 , 1 )

Punt intersecció Q entre la recta r i el pla :

1 3 (2 3 ) ( 1) 1 0

El punt Q és punt mitjà entre P i P ’ . Aleshores:

31. Calcula l’àrea del triangle els vèrtexs del qual són els punts intersecció del pla 2x y 3z 6 amb els eixos de coorde-nades. Escriu l’equació de la recta que conté cadascun dels costats d’aquest triangle.

Vèrtexs del triangle:A(3, 0, 0) B(0, 6, 0) C(0, 0, 2)

Page 160: MATEMÀTIQUES - · PDF fileMatemàtiques 2 · Batxillerat · Solucionari No és permesa la reproducció total o parcial d’aquest llibre, ni el seu tractament informàtic, ni la

160 SOLUCIONARI DEL LLIBRE DE L’ALUMNE11

AB AC

e

e

ee e

1

2

3

1

3 3

6 0

0 212 18 33 26e

Recta que conté el costat AB:(x, y, z) (3, 0, 0) 1( 1, 2, 0)

Recta que conté el costat AC:(x, y, z) (3, 0, 0) 2( 3, 0, 2)

Recta que conté el costat BC:(x, y, z) (0, 0, 2) 3 (0, 3, 1)

Avaluació

1. Donat el punt P(1, 2, 3) i la recta

rx y

z:1

21

1

a) Troba el punt P ’, projecció ortogonal del punt P sobre la recta r.

Les coordenades d’un punt genèric de la recta r són de la forma: ( 1 2 , 1 , ). El vector ha de ser perpen-dicular al vector , vector director de la recta r. Per tant:

b) Calcula la distància del punt P a la recta r.

2. a) Troba l’equació del pla que conté el punt P(1, 3, 2) i la recta:

r x y zx z

: 2 06 0

Determinem l’equació vectorial de la recta r:z ; x 6 z x 6 y x z 2 6 2 4 2 r : (x, y, z) ( 6, 4, 0) ( 1, 2, 1)

Equació general del pla :

xyz

x y z1 1 73 2 72 1 2

0 11 9 7 30 0

b) Calcula la distància de l’origen de coordenades al pla .

u

3. a) Esbrina la posició relativa de les rectes:

r: (x 1, y, z 1) (1, 1, 2)

Les dues rectes s’encreuen, ja que no tenen cap punt en comú i no són paral·leles.

b) Calcula la distància entre les rectes r i s.

r: (x, y, z) (1, 0, 1) (1, 1, 2)

s: (x, y, z) (0, 1, 1) (2, 1, 3)

PQ u v

u v

e

, ,1 1 2

1 1 12 2 3

14

11 22

1 1

2 37 3 1 7 32

3

1 2 3e

ee e e ( , , )

u

4. Calcula l’àrea del triangle determinat pels punts d’intersecció del pla 3x 2y 4z 24 0 amb els eixos de coordenades.

Els punts de tall amb els eixos són:

A( 8, 0, 0) B(0, 12, 0) C(0, 0, 6)

Quin és el volum de la piràmide triangular definida pels punts anteriors i l’origen de coordenades?


Recommended